You are on page 1of 57

ESTONIAN FINNISH PHYSICS OLYMPIAD

PROBLEMS & SOLUTIONS (2003-2014)


ESTONIAN FINNISH PHYSICS OLYMPIAD
PROBLEMS & SOLUTIONS (2003)
Tallinn 2003 2) A wire is made of dierent alloys, its heat resis- I(mA) the brick velocity change during a semi-period is
tivity as a function of the coordinate along the negligible? Further we assume that this condi-
Complete solution of each subquestion gives two
wire is given in the attached graph. The cross- tion is satisfied.
points. You have five hours for solving. 10
section area of the wire is S = 1 mm2 , its length 2) The brick is kept in motion along x-axis by a
1. Volleyball (8 points) l = 4 cm. Find the heat flux through the wire, force Fx in such a way that the mean brick ve-
if one end of the wire is kept at the temperature locity is v. Sketch graphically the dependance
Consider a simplified model of a volleyball: a thin 100 C, and the other end at 0 C.
5
Fx (v).
spherical envelope filled with air. The envelope (Km/W) 3) The brick is kept in motion along (horizontal)
material is non-stretchable, easily foldable. The U(V) y-axis by a force Fy in such a way that the mean
excess pressure inside the ball p = 20 kPa, the 0 0,1 0,2 0,3 0,4 brick velocity is v. Find the dependance Fy (v).
ball radius R = 10 cm and mass m = 400 g (the 0,2
tunnel diode 4) Until now we have ignored the dependance
air mass inside the ball is negligible). You can Uin
(input-signal)
of the friction coecient on the sliding velocity.
neglect the dependence of the excess pressure on Further let us assume this dependance is given
the deformation of the ball. 0,1 E R Uout
by the graph below. The brick is kept in motion
(output-signal)
1) The ball is pressed between two parallel rigid along x-axis by a force Fx in such a way that the
plates, the distance between of which is 2R 2h x(cm) 1) Find the current in the circuit, if Uin + E = mean brick velocity is v. Sketch graphically the
(so that the height of the deformed segments is 0 1 2 3 4 0,08 V. dependance Fx (v) taking u = 34 w0 .
h = 1 cm). Find the force between the ball and a
2) Find the output voltage Uout 0 if Uin = 0 V.
plate N .
3) Find the output signal Uout Uout 0 if Uin = 1
2) Ball moves with velocity v0 = 2 m/s and hits 1 mV.
3. Gravitation (6 points)
a rigid wall. Find the maximal height of the de- 4) The input signal is given in the graph below. 0
formed segment hm . . .
1) Find the free falling acceleration g0 at the sur- Sketch the output signal as a function of time.
3) . . . and the collision time . I(mV)
face of such a spherical planet, which has mass 1
4) For small excess pressures, the ball can lose M and material density (in what follows, M
its spherical shape even in those points, which w
and are assumed to be constant). _ _

are not in touch with the wall. Which condition t (ms) w0 w0 w0
between the quantities p, R, m and h has to 2) Is it possible that at the surface of a non- 1 2 5) The brick is put on the surface, there are no
be satisfied in order to ensure that such a loss of spherical planet, there is a point with free falling external forces. What is the bricks terminal
sphericity is negligible? acceleration g > g0 ? Motivate your answer.
velocity v? Provide the answer as a function of
3) For which planet shape the maximum of the -1 u.
2. Heat flux (4 points) free falling acceleration is achieved? Answer can
Heat resistivity is equal to the ratio of the tem- be given in polar coordinates, the expression can 5. Vibration (10 points)
perature dierence between the end-points of a contain one unspecified constant. Consider a smooth horizontal surface, which is
wire of unit cross-section and unit length, and moved periodically back and forth along the ho-
the heat flux (unit: W) through this wire. rizontal x-axis: during the first semi-period ,
1) Microprocessor of power P = 90 W has a 4. Tunnel diode (8 points) the surface velocity is u, during the second semi-
water-cooling system. The chip and flowing wa- period u. A brick of mass m is put on that
ter are separated by a copper plate of thickness Tunnel diode is a semi-conductor device, similar surface; the friction coecient between the sur-
d = 5 mm and cross-section area s = 100 mm2 . to the ordinary diode, the volt-amper characte- face and brick is , free falling acceleration is g.
What is the temperature dierence between the ristic of which is given in the attached graph. The 1) The brick has initial x-directional velocity v.
processor and water? The copper heat resistivity circuit below describes a simple amplifier. The Which condition between the quantities g, , v,
is = 2,6 mmK/W. resistance R = 10 , battery voltage E = 0,25 V. and has to be satisfied in order to ensure that
6. Charged particle (12 points) case when the pulses are of variable polarity: for all the eects other than diraction. Estimate a
N S
the 2n-th pulse, the electric field is +E, and for the surface density of the light power in the cen- S N b
A particle of mass m and charge q is in a ho- c
the 2n+1st pulse E. Find the particles average ter of the star image w2 (in the focal plane of the
mogeneous magnetic field with induction B (the
velocity, (vectorially) averaged over the cyclotron telescope), due to the light arriving from the star.
vector is parallel to the z-axis). The characteris-
period. 5) Provide an expression for the ratio of the sur-
tic time of the system is the cyclotron period of
face densities of the light powers k in the midd-
the particle TB = 2m/Bq. The system is si- 7. Telescope (12 points)
le of the star image, and in a point farther away
tuated in between two parallel electrodes, which
As it is well known, a telescope makes it possib- from it.
can be used to create an homogeneous, parallel
le to see the stars in daylight. Let us study the 6) Is it possible to see a star in daylight using a
to the x-axis electric field E.
problem in more details. Consider a simplified telescope? Plain eye? Motivate yourself.
1) The particle is at rest. At the moment of time
model of the eye: a single lens with focal length
t = 0, the electric field E is switched on; after a 8. Experiment (12 points)
f = 4 cm and diameter d = 3 mm creating an
short time interval (  TB ), it is switched o,
image on screen (retina). The model of a telesco- Determination of attraction force between iron
again. What will be the trajectory of the particle?
pe is similar: a lens of focal length F = 2 m and plate and a permanent magnet as a function of
2) Let px and py denote the x- and y compo- diameter D = 20 cm creating an image in focal distance. Tools: iron plate, wooden brick, ruler,
nents of the momentum of the particle. Sketch plane (where eg. a film can be put). In your calcu- dynamometer, paper stripes.
the trajectory of the particle in (px , py )-plane lations, the following quantities can be used: the Attention! the permanent magnets are ve-
and depict the vectors of the momentum for the density of the light energy radiated from a unit ry strong, keep them far from credit cards etc.
moments of time tn = nTB /4 (n = 1, 2, 3 and Solar surface in unit time w (the light power Avoid also hitting them against each other and
0
4). surface density); the ratio of the star and Sun against the iron plate, because they are fragile
3) Consider the situation when the on-o switc- distances q = 4 105 (we assume that the star and can be broken.
hing of the electric field is done periodically, is identical to the Sun); Solar angular diameter 1) Determine the static and dynamic coecients
starting with t = 0, after equal time intervals 9 mrad. Remark: If the answer contains w0 of friction between the iron plate and a paper
t = TB /4. Sketch the particle trajectories in then numerical answer is not required. stripe. Draw the scheme of your set-up.
(px , py )- and (x, y)-planes. 1) Consider a sheet of paper, the normal of which 2) (4 points) Determine the attraction force be-
4) Let the period be short, t  TB (but still is directed towards the Sun. What is the sur- tween the iron plate and a magnet for those dis-
much longer than the duration of pulse, t  face density of the light power w1 arriving to the tances which allow direct usage of dynamometer.
). Show that after the n-th pulse (at the time sheet from the Sun? Draw the scheme of your set-up.
moment tn = nt), the momentum of the par- 2) Find the net power P2 of the light, which is 3) (4 points) Determine the attraction force be-
ticle can be represented as the sum of n vectors focused by the telescope into the image of the tween the iron plate and a magnet for smaller
pi , where all the component-vectors are equal star. distances. For that purpose, you can use the
in modulus (the modulus being independent of 3) Assume that blue sky is as bright as a sheet wooden brick sliding down an inclined plate and
n), and the neighboring vectors ( pi and pi+1 , of gray paper illuminated by Sun. You may as- hitting the magnet. You do not need to study the
i = 1, 2, . . .) have equal angles between them. sume that in the direction, perpendicular to the zero-distance (direct contact of magnet and iron
5) Consider the limit case t 0, so that sheet, the ratio of the light power scattered by plate) case. Draw the scheme of your set-up. De-
E /t Ek (Ek denotes the time-average of the paper into a 1-steradian space angle, to the pict all the measurement result graphically.
the elctric field). Sketch the particles trajectory net light power arriving to the sheet, is 0,1 4) Join two permanent magnets by a bridge ma-
in (px , py )-plane and express the particles mean (this corresponds to the dissipation of ca 70 % de of a piece of iron (a) as shown in figure. Put
velocity (vectorially; averaged over the cyclotron light energy in the gray paper). What is the sur- a stripe of paper (b) on the iron plate (c) and put
period) via the quantities Ek and B. face density of the light power in the focal plane the system of magnets upon it. Determine the
6) Let us return to the non-zero (but still small, of the telescope w3 , due to the blue sky? attraction force between the system of magnets
t  TB ) time-intervals. Let us consider the 4) While studying the star image, let us ignore and iron plate.
Solutions h(2Rh) < 4R2 . This condition is always satis- 4. Tunnel diode (8 pts) 2) F = 0, when |v| < u; F = mg, when
fied, no additional constraint is needed. Notice 1) For voltages below 0.08V, the graph is almost a |v| > u.
1. Volleyball (8 pts) that we considered only the worst case requiring straight line corresponding to a constant resis- 3) The x-component of the frictional force
1) F = pS, where S = r2 is the segment ba- the largest compensating force when the force of tance RD = 0.05 V/6.5 mA 7.7 . Hence cancels in average out, the y-component is left:

se surface. It is easy to see that r2 = (2R h)h, inertia is normal to the surface. Remark: The ca- I = (Uin + E)/(R + RD ) 4.5 mA. F = mgv/ v + u2 .
2

hence F = ph(2R h) 120 N. se of stretchable envelope is completely dierent, 2) The output voltage can be found graphically: 4) F = [(v + u) + (v u)]mg, if v > u and
2) During the collision the ball is deformed as sphericity disappears over all the surface (try to the diode voltage U (I) = E IR, hence, the F = [(u + v) (u v)]mg, if v < u (F > 0
shown in Figure: the envelope is not stretchable, press a balloon against a glass!). intersection point of the graph and the straight means that F and v are opposite to each other).
hence it retains the spherical shape (except whe- line U = E IR, gives us the diode current It is easy to see that by small values of v, the force
re in touch with the wall). Using the approxima- 2. Heat flux (4 pts) 6 mA; then, the output voltage IR = 60 mV starts linearly decreasing [with F (v = 0) = 0]
tion h  R we can neglect the term h2 in the (see the graph). (F < 0 implies that force and velocity are in
1) The heat flux P = T s/d, hence T =
expression for the force. Then, the force is pro- the same direction). At u = v, the graph exerts
P d/s 12 K.
portional to h, ie. the ball behaves as a spring of I(mA) a jump, F becomes positive, and starts dec-
2) By a constant heat flux P , the temperature
stiness k = 2Rp. According to the ener- reasing. The attached graph presents a sketch
change along the wire T = P x/S, whe-
gy conservation
 law mv 2 = 2Rph2 , hence of the eective friction coecient; the const-
re x is a displacement along the wire. Hence
h = v m/2Rp 11 mm. 10 ruction has been based on the lengths k1 =
the temperature drop t1 t2 = P S/S, where
(w0 /2) (w0 ), k2 = (w0 /4) (5w0 /4),
S is the surface under the graph. Thus, P =
and k3 = (0) (3w0 /2).
(t1 t2 )S/S. Using the graph we find S
5
50 Kcm2 /W and P 20 mW.

U(V)
3. Gravitation (6 pts)
0 0,1 0,2 0,3 0,4 1
1) g0 = M/R2 , where R can be found from the k3
k2
relationship 43 R3 = M . Hence, 3) One millivolt input shifts the line intersec- 0
ting the graph a little-bit sideward, but the shift k1
4 2/3
g0 = M ( ) . is so small that the graph can be approximated
3M
by a straight line. The cotangent of the slope of
2) Taking a piece of ground from a certain point w
that line gives us the dierential resistance of the
of the planet surface and carrying it into another ef _
diode, Rd = 16 . Then, a small change in w0
point, the free fall acceleration can be changed ekt w0
the input voltage U will lead to a current chan-
(the sign of the change depends on the direction
ge I given by the relationship (R + Rd )I =
of the transport).
U ; hence, I = U/(Rd + R). The output
3) This is the half of the harmonic 5) The rest position is unstable, if u < w0 : the
 oscillations 3) Let use the polar coordinates with the origin voltage change Uout = IR = RU/(Rd +
period, = m/2Rp = m/2Rp at the point where the free fall acceleration is to R), and the amplification factor Uout /U = particle obtains the (stable) velocity u. If u > w0 ,
18 ms. be maximized. Let the axis = 0 be given by the R/(Rd + R) 1.7. Consequently, the output the rest position is stable, and the particle ve-
4) Let us use the balls system of reference. The direction of the acceleration. Carrying a small voltage is 1.7 mV, and . . . locity remains 0.
envelope surface element dS is exerted by the piece of ground from a point (r1 , 1 ) to anot- 4) the output graph is exactly the same as the in-
force of inertia dFi = amdS/4R2 , where a = her point (r2 , 2 ) must keep the modulus of the put graph, except that it is vertically stretched by 6. Charged particle (12 pts)
ph(2R h)/m. Thus, dFi = ph(2R acceleration vector g constant, i.e. the vector of a factor of -1.7.
h)dS/4R2 . In order to keep the spherical sha- the small change must be perpendicular to the 1) The particle acquires the velocity v = Eq /m
pe, this force has to be compensated by the force vector g. Consequently, cos 1 /l12 = cos 2 /l22 , 5. Vibration (10 pts) and starts moving along a circle of radius R, with

due to the excess pressure dFr = pdS, hence hence l = l0 cos . 1) mg  v. mv 2 /R = Bvq, hence R = E /B.
px endpoint of the particles momentum lies on that tance. The image of this piece of paper has le, the accuracy of the results can be enhanced by
4 circle. Thus, averaged over the moments of time size s = SF 2 /L2 ; thus, w3 = P3 /s = subtracting this weight from N : F = Fd /Fp .
py 3 2nt, the average velocity is vx = Eq /2m. w1 ( 4 D2 /F 2 ) = w0 (D/4F )2 . 3) We use a similar set-up, except that smaller
1 For odd number of impulses, one has to add 4) The angular distance of the first diraction number of paper stripes is used (totaling up to
the lastly given momentum P = (Eq, 0); minimum (using the single slit approximation around 2 mm), and a steep slope of the plate.
2 hence, a similar circle is formed, except that the circle is actually not a slit) is /D. Hence, We let the brick slide down the slope and hit
2) center is shifted by P : the center coordinates are the bright circle radius can be estimated as on the magnet. We keep the falling height and
px
mvx = Eq /2, mvy = 0. Correspondingly, = F /D. Consequently, w2 = P2 / 2 = plate slope constant, and measure the sliding
averaged over the moments of time 2nt, the w0 (D2 /4qF )2 . path, which is covered by the papers and the
py average velocity is vx = +Eq /2m. Averaged 5) k = (w2 +w3 )/w3 = 1+()1 (D/q)2 magnet after having been hit by the brick. This
over all the moments of time, the final result is 4 (assuming 500 nm). path is inversely proportional to the attraction
vx = vy = 0. 6) k 1 1 (or k 1 > 1) means that the star force N . If this path turns out to be too short
can be easily seen (as is the case for the telesco- for an accurate measurement (for very small
3)
pe); k 1  1 means that the star cannot be distances between the magnet and the plate),
4) Let us consider the vectorial sum of the mo- seen (for the eye, k 1 1 104 ). several brick hits can be used. In that case,

P res
menta given to the particle in dierent mo- the single-hit path can be found as the measu-
ments of time. During the time interval t, all P1 8. Experiment (12 pts) red path, divided by the number of hits. The
the component-vectors are rotated by the angle P2
1) We incline the plate until sheet starts sli- constant of proportionality can be found by

s
2t/TB = Bq/m. Thus, with each impulse,

re
comparing the results of this and previous ques-
the static coecient is found as static =
ding:

P2n
P
a vector P with modulus P = Eq is added; the h/ l2 h2 , where h is height of the plate end- tion, for those distances, which are covered by
angle between the lastly added vector, and the point, and l the plate length. Now we push both measuring techniques. Reasonable measu-
previously added vector is = tBq/m. the sheet laying on the plate slightly, and find rement results are given in the attached graph.
5) All these vectors, when added according The figure represents the net moment Pres after the inclination angle, for which the sheet will sli- F /N/
 30
to the triangle rule, form a circle of radius 2n-th impulse, and also the net impulse Pres for de down with a constant velocity; we use again

R = P/ sin P/ = Ek m/B. another time moment 2n t. For an odd num-  the formula kinetic = h/ l2 h2 . The reaso- 25
px
ber of impulses, the pattern is exactly the same, nable numerical values are static 0.37 and 20
except that all the vectors have opposite direc- kinetic 0.29.
py 15
tion (because the lastly added component, the 2) We put several paper stripes on the plate, and
10
vertical vector, has opposite direction). the magnet on the top of them. We make a
loop of cord, put it around the magnet, and pull 5 d /mm/
7. Telescope (12 pts) it using the dynamometer sideward (sliding the 0
Hence, the average velocity vy = R/m = 1) The light flux density decreases inversely pro- whole system of paper and magnet). The attrac- 0 1 2 3 4 5 6
Ek /B, vx = 0. portionally to the square of the distance, the- tion force F N (where N is the reaction force)
6) Two subsequent momenta along x-axes result refore w1 = w0 Rp2 /L2p , where Rp is the solar is found as the ratio of the reading of the dyna- 4) The same technique as in the case of previous
in net moment along y-axes Py = P . The radius, and Lp the solar distance. Due to mometer Fd and the appropriate friction coef- question is applied, except that a larger number
sequence of such moment pairs form a (near- = 2Rp /Lp , we obtain w1 = w0 2 /4. ficient (depends, which reading is taken: either of hits has to be used ( 10 20). Reaso-
ly) circle (actually, regular equilateral polygon), 2) The previous result can be applied to the the maximal one, or the one corresponding to sli- nable result for d = 0.2 mm (one paper stripe)
composed of vectors (with modulus Py ), the star flux density, which is q 2 w1 ; hence P2 = ding), F Fd /. The distance d is measured is F 270 N. Note that the result is much lar-
angle between of which is 2 (see Fig.). The ra- 14 D2 w1 q 2 = w0 (D/4q)2 . in the number of paper stripes (one stripe had ger than the double result in the case of a single
dius of the circle is Py /2 = P/2 = 12 Eq , 3) The paper surface area S radiates towards a thickness of 0.2 mm). For large distances magnet; this is due to closing the ferromagnetic
and its center coordinates are mvx = Eq /2, the lens of the telescope the power P3 = (approximately d > 4 mm), the weight of the pa- loop of magnetic field lines.
mvy = 0. After an even number of impulses, the w1 S( 4 D2 /L2 ), where L is the telescope dis- per Fp stripes and magnet is no longer negligib-
ESTONIAN FINNISH PHYSICS OLYMPIAD
PROBLEMS & SOLUTIONS (2004)
1. Rubber fiber (12 pts) circular. Write down two independent equations the main optical axis and the straight line lay in
relating r(t), vr (t), and vt (t) to each other (using the same (x, y) surface. Hint: use the coordinate
Fibers made of elastic rubber can be stretched to
also the mass of the body m, together with the system, where the origin coincides with the cen-
lengths l, much longer than the length in unde-
parameters L, Vr , Vt , l0 , S0 , E0 ). (3.5 pts). ter of the lens and represent lines algebraically,
formed state l0 . For such rubbers, the net volume
6) Find the relationship between r(t) and vr (t) e.g. y = ax + b. Make use of the formula of thin
of the fiber remains constant.
(containing also the parameters m, L, Vr , Vt , l0 , lens f 1 = x1 x1 (x > 0 and x are the
1) Express the cross-sectional area S of such a fi- S0 , E0 ) assuming that |r| L, and find the pe- x-coordinates of a point and its image, respecti-
ber in a deformed state via its length l and initial riod T of small oscillations of r(t). Simplify the vely) (2 pts).
dimensions l0 , S0 (1 pt). expression of T for L l0 (3 pts). 2) In figure (a), draw the image of the given line
2) For small deformations of an elastic material, and indicate, which parts of the image are vir-
the stretching force F and deformation x are re- 2. Planets (6 pts)
tual, and which are real (2 pts).
lated to each other by the Hookes law F = k0 x, Two planets move along circular orbits around 5. 4th order ellipse (6 pts)
where the stiffness k0 = E0 S0 /l0 and E0 is the a star of mass M = 2.0 1030 kg; gravitational 4 4
4th order ellipse is defined by equation xa4 + yb4 =
Youngs modulus of the rubber. For non-small constant G = 6.67 1011 m3 /kg s2 . The de-
1, where a and b are the lengths of the half- axes.
(possibly large, l l0 ) deformations of elastic pendence of the angular distance between a pla-
Consider an homogeneous cylinder, the cross-
rubber, however, the Hookes law is substituted net and the star on time, as seen from the other
section of which is 4th order ellipse. The po-
by a non-linear law, F (l) = a + bl (breaking planet, is depicted in figure. 3) Photographer wants to take a photo of a field sition of the cylinder is measured by the angle
of this law at very large values of l will not be of flowers. In order to get image where all the 0 /2 between the vertical direction and
studied here). Express the constants a and b in flowers (both the close and far ones) are sharp, a longer half-axes, see figure.
terms of l0 , S0 , and E0 (2 pt). he has to use a lens with tilt-shift (TS) capabi-
3) Suppose such a fiber is stretched by some lities (either an ordinary camera with TS lens,
force up to the length l. A small change F of or a large-format camera, where the entire lens
the stretching force results in a small change in compartment can be freely positioned). The
the length l l. Express F in terms of l, l0 , field of flowers (which extends effectively to in-
S0 , E0 , and l (1 pt). finity) and the image of its distant edge, together
4) Suppose a small body is fixed to an one end with the image plane are depicted in figure (b).
of the fiber and the system is put into rotation Reconstruct the position of the lens, the focal
around the other end of the fiber. In the case of length of which is provided as a scale (2 pts).
a circular motion of the body, express the length
of the fiber l via l0 , S0 , E0 , and the kinetic energy
1) What is the ratio of the radii of the planets k
of the body K (the kinetic energy of the fiber and
(2 pts)?
gravity can be neglected). (1.5 pts) 1) What are the equilibrium positions of the cy-
2) Determine the value of the unit on the vertical
5) Let us analyse a slightly non-circular motion linder laying on an horizontal surface (3.5 pts)?
axis (or express it in terms of k, if you were unab-
of the body. Let us describe the motion of the 2) Sketch on graph the net torque of gravity and
le to find it) (2 pts).
system by the length change of the fiber r(t) = surface reaction forces with respect to the con-
3) What are the orbital radii of the planets, if the
l(t) l(0), the radial vr (t) and tangential vt (t) tact point of the cylinder and surface as a func-
unit on the horizontal axis equals to one year
velocities of the body (the components respecti- 4. Transparent film (6 pts) tion of (0 /2). For the axis of torque,
(2 pts)?
vely parallel and perpendicular to the fiber). The A thick glass plate is coated by a thin transparent you do not need to indicate any quantitative sca-
initial values of these quantities are designated 3. Tilt-shift lens (6 pts) film. The transmission spectrum of the system is le (1.3 pts).
as L l(0), Vr vr (0), and Vt vt (0). The 1) Show that an image of a straight line created depicted in graph (light falls normal to the plate). 3) Which equilibrium positions are stable and
values L and Vt are chosen so that if the initial by a thin lens is also a straight line. Consider The refractive index of the film n 1.3. What is which are not? Motivate your answer (1.2 pts).
radial velocity were zero, the motion would be two-dimensional geometry only, i.e. assume that the thickness of the film d?
6. Magnets (6 pts) 7. Passive air-cooling (9 pts) contain also the parameters defined above) (2
Certain type of magnetic toys are made up of Consider a passive cooling system depicted in pts).
ferromagnetic spheres and permanent magnets figure. Cold air (at normal conditions: p0 = 5) What is the temperature T of the outflowing
of cylindrical shape. These building blocks can 105 Pa, T0 = 293 K) flows over the heat sink of a air? In your calculations, you may use approxi-
be used to build, for instance, a tetrahedron, see chip of power dissipation P = 100 W, into a ver- mation T T0 T0 (2 pts)?
figure (letter N marks the northern end of a tical pipe of length L = 1 m and cross-sectional
magnet). Assume that all these permanent mag- area S = 25 cm2 . After passing the pipe, air
nets are identical and each of them alone can enters the ambient room. Assume that the air 8. Loop of wire (7 pts)
create a magnetic flux (assuming the both inside the pipe becomes well mixed; neglect the
Consider a rectangular loop of wire with dimen-
ends of the magnet are in contact with a U- viscous and turbulent friction of air inside the pi-
sions a = 0.03 m and b = 1.0 m, one side
shaped large piece of ferromagnetic material, so pe and heat sink. Air can be considered as an
of which is parallel to another long straight wi-
that a closed ferromagnetic contour is formed). ideal gas with adiabatic exponent = 1.4 and
re carrying current I0 = 1000 A, at distance
Assume also that due to high magnetic perme- molar mass = 29 g/mol.
l = 0, 01 m, see figure. The magnetic inductance
ability of the material of the building blocks, all of such current is plotted as a function of the dis-
the magnetic field lines are constrained inside of tance from the wire in attached graph. The Oh-
them (i.e. in the surrounding medium, the mag- mic resistance of the loop is R = 1, 0 , the in- 9. Experiment (15 pts)
netic inductance B = 0). ductance is negligible.
The black box contains a nonlinear element
(active resistance) and a capacitor, connected
sequentially. Find the capacitance C of the ca-
pacitor (5 pts) and the V I characteristic of
the nonlinear element (6 pts). Note that (a) the
I0 electrolytic capacitor accepts only one polarity
of charge (indicated by the colors of the output

b
wires of the black box); (b) the V I characte-
ristic cannot be expected to be symmetric with
l a respect to I = 0. However, you are requested
b to study the range I > 0 corresponding to the
b discharge of the capacitor. Tabulate your measu-
1) Express heat capacitance at constant pressure rement data and draw appropriate graphs (4 pts).
cp via quantites and R (1 pt). Experimental equipment: batteries, wires, multi-
1) Let us designated the fluxes in each perma- 2) Find a relationship between the outflowing air meter, stopwatch, graphic paper.
nent magnet (magnets AF in figure) by A 1) Calculate the magnetic flux through the
density and temperature T (the relationship
F . Write down equation relating A , B , and loop (2 pts).
may contain also the parameters defined above)
C to each other (and possibly to ) (1 pt). (2 pts). 2) At a certain moment of time, the current in
2) Write down equation relating A , B , and 3) Find a relationship between the air flow ve- the long wire is switched off. What is the net
F to each other (and possibly to ) (1 pt). locity in the pipe v and outflowing air density charge Q flowing through a fixed cross-section
3) Find the ratio F /C (1 pt). (the relationship may contain also the parame- of the wire of the loop (3 pts)?
4) Find the magnetic fluxes in each permanent ters defined above) (2 pts). 3) What is the net momentum p given to the
magnet (2 pts). 4) Express the power disspation P in terms of loop during the switch-off of the current (express
5) Which of the magnets is the most difficult one the air flow velocity v, the outflowing air tem- it in terms of Q and the given quantities, if you
to remove? Motivate your answer (1 pts). perature T , and density (the relationship may were unable to calculate Q) (2 pts)?
1. Rubber fiber (12 pts) and maximum, t1 1.2, and between neighbou- perpendicular to the tangent at that point. In or-
1) Volume conservation: Sl = S0 l0 , hence S = ring minima t2 4.6 (in graph units), respecti- der to find the tangent, let us differentiate the el-
3 3

S0 l0 /l. vely. = tt12 = 2 arcsin


2
k
0.261, hence lipse formula: 4 xa4 dx + 4 yb4 dy = 0, hence, with
1 3 4
2) At the limit of small deformations, F (l) = a+ k = sin[( 2 )] 1.47 1.5. dx = 1, dy = xy3 ab 4 , a tangent vector is ~ =
b 2 2
l a bx/l0 = k0 x, hence E0 S0 /l0 = b/l0 , 2) For the maximal angular displacement m ,
[1, ( xy )3 ( ab )4 ]. The vectors are perpendicular,
hence b = E0 S0 l0 (1 pt). Besides, at l = l0 , sin = k = sin[( 1 )], hence = if the scalar product is zero: x y( xy )3 ( ab )4 , i.e.
m 2 m 2) First we notice that the distance of the image
F = 0 (0.5 pts), hence a + bl = 0 and a = E0 S0 ( 1 ) = 0.75 rad 3.6 units. Therefore, the y b 2 y b 2
2 of the far end of the field (let us designate it by A) x = ( a ) = = arctan x = ( a ) .
(1 pt). unit is m /( 21 ) 4.8. 2) Around each zero changes sign. At = 0,
l0 l from the focal plane equals to the focal length f .
3) F dF dl dl = E0 S0 l2 . small increase in will result in a torque trying
So, the lens plane must touch the circle of radius
4) Newton II law: E0 S0 (1 ll0 ) = 2K/l, hence 3) If the angular velocities of the planets are 1 to return to the initial position, i.e. the torque
and 2 , the seeming angular velocity (as seen f , drawn around A, see figure. Next we notice
l = l0 + E2K 0 SO
. becomes negative. So, the graph looks like the
that there is one such ray connecting far end of
5) Conservation of angular momentum: lvt = from the system, where both star and the obser- one below.
the field and its image, which does not refract
LVt , hence vt = Vt L+r L
(1.5 pts). Conservation ver planet are at rest) is = 1 2 . From
the Newton II law, GM ri2 = i2 ri , where the one passing through the enter of the lens, see
of energy: 2 (vt + vr ) + E0 S0 (r l0 ln r+L
m 2 2
L )=
m 2 2 i = 1, 2 q and ri is the planets orbital radius. figure.
2 (Vt + Vr ) (2 pts).
6) Substituting vt from the angular momen- So, i = GM ri3 and = GM (r21.5

tum conservation law into the energy equation r1.5 ) = GM r1.5 (1 k 1.5 ). Finally, the
1 1
we obtain m 2 L 2 2
2 [Vt ( L+r ) + vr ] + E0 S0 (r square of the observed period T = (2/) =
2 2
l0 ln r+L
L ) = m
2 (V t
2
+ V r
2
) (0.4 pts). Furt- 2 3
4 r1 /GM (1 k 1.5 2 2
) and r1 = [T GM (1
her we make use of condition |r| L and k 1.5 )2 /42 ]1/3 . Using T 4.6 years 1.45
L 2
substitute ( L+r ) 1 2 Lr + 3( Lr )2 (0.4 108 s, we arrive at r1 2.5 1011 m; correspon-
3) If the derivative of the graph at equilibrium
pts), ln L Lr 12 ( Lr )2 (0.4 pts). Line- dingly, r2 = kr1 3.7 1011 m.
r+L
point is negative, the position is stable; otherwi-
ar in r terms cancel out due to the condition
se it is unstable. = 0 and = /2 are stable,
E0 S0 (1 lL0 ) = mVt2 /L (0.4 pts). So, we arrive
= ( ab )2 is unstable.
at m 2 r 2 2 1 r 2
2 [3Vt ( L ) + vr ] + 2 E0 S0 l0 ( L ) = 2 Vr
m 2 4. Transparent film (6 pts)
(0.4 pts). This is the energy conservation law for The short-wavelength oscillations on the graph
a pendulum consisting of a spring with effective 3. Tilt-shift lens (6 pts) are due to the diffraction on the film, there-
stiffness keff = (3mVt2 + E0 S0 l0 )L2 and of a fore the local maximum condition is 2dn =
x f
body with effective
p mass meff = m (0.5 pts). So, From f 1 = x1 x1 we obtain x = x +f . N = cN/. So, 2dn = cN and 2dn( +
T = 2L/ 3Vt2 + E0 S0 l0 m1 2L/ 3Vt Since the ray passing through the centre of a lens ) = c(N + 1), hence 2dn = c and d =
(0.3+0.2 pts). without refraction, from similar triangles we ob- c/2n. In order to measure the distance be-
tain the relationship between the y-coordinates tween two maxima more precisely, we take a
2. Planets (6 pts)
of the image: y = y xx = xy +f f
. Substituting longer frequency interval , e.g. = 80 THz
1) First method: determine the tangents to the into y = ax + b we result in y f = a x f + b,

x +f x +f and count the number of maxima between them,


graph at the points where the curve crosses the x
hence y = ax + b( f + 1) = x (a + fb ) + b,
m 34. Consequently, = /m
horizontal axis, a1 2.8 and a2 16 (in
k which defines also a straight line. 2.35 THz, and d 50 m
graph units), respectively. Then, a2 = 1+k
k 5. 4th order ellipse (6 pts)
and a2 = 1k . The graph units will cancel out 1) First we notice that the line, its image, and
from the ratio of these to tangents, = aa12 = lens plane intersect in one point, because the 1) There are trivial positions = 0 and =
1+k 1
1k 5.7, hence k = 1+ 1.4. image of that point of the line which lays at the /2. Besides, there is a position between these
Second method (more precise): determine lens plane, coincides with itself. Now, it is easy to two. At the equilibrium, the vector from the ori-
the distances between neighbouring minimum construct the image, see the figure. gin to the touching point ~r = (x, y) has to be
6. Magnets (6 pts) process is by constant pressure, otherwise the- 0 I/2l and B1 = 0 I/2(l + a), we end
1) Each permanent magnet can be considered re would be huge acceleration due to pressure up with dp = Rb 2
0I a
l(l+a) d. Using the re-
as a solenoidal molecular current at the surface drop). sult of first question, d = b a+l
2 ln l dI, i.e.
0

3) Different air densities inside and outside the b0 2 a a+l


of the magnets. Suppose that each magnet has dp = ( 2 ) Rl(l+a) ln l IdI. Finally, p =
net surface current I. Consider triangular con- pipe give rise to small residual (as compared to a(b0 I0 )2
ln a+l
8 2 Rl(l+a) l 2.08 10
6
kgm/s2 .
tour going through the interiors of the magnets the static pressure distribution inside the pipe)
The same result could have been obtained
A, B, C. According to the circulation theorem pressure difference between the open ends of the
using the graph and approach used in the alter-
for that contour, the circulation BA l + BB l + pipe, p = gL. This pressure difference is
native solution of the question 1.
BC l is proportional to the overall molecular cur- responsible for the acceleration of the air, from
rent through that contour: BA l + BB l + BC l = zero, up to the velocity of the air flow v. The mo- 9. Experiment (15 pts)
3kI. Here, BA designates magnetic inductance mentum balance for small time interval yields
The idea: take readings of discharge current, as
inside the magnet A; BB and BC are defined Sp = (Sv )v, hence (0 )gL = v 2 . . a function of time. The surface area under the
analogously. For a single magnet attached to a Here, the cold air density 0 = p0 /RT0. Final-
graph is the outflown charge Q. Taking the rea-
p0
massive U-shaped ferromagnetic, the circulation ly, ( RT 0
)gL = v 2 . dings of voltage U0 and U1 at the beginning and

theorem yields B0 l = kI (where B0 is the mag- 4) Heat flux: P = 1 R(T T0 )Sv/. at the end of discharge, we obtain Q/C = U0
netic inductance inside the magnet; the cont- 5) From the result of question 2, we obtain U1 , i.e. C = Q/(U0 U1 ). As for V-I characteris-

ribution to the circulation inside a massive U- = T T . From the result of question
tic, interrupt from time to time discharge, take
v2
shaped ferromagnetic can be neglected, because 3, = gL . Substituting these values in- reading of discharge current I just before inter-
the magnetic field there is much smaller than in- to the equation obtained for question 4, P = ruption, measure voltage U , and continue disc-
side the magnet). So, BA + BB + BC = 3B0 v3 harging. Collect enough data to draw V-I charac-
1 R gL T S/. Using the gas equation, this
and A + B + C = 3. simplifies to v = 1 S p0 . So, T = T0 [1 + teristic.
3 gL P

2) There are no sources of magnetic field lines gL P 2/3


( 1 S p0 ) /gL] 322 K.
(and hence of the flux), so A = B + F .
3) Due to symmetry, F /C = 1. 8. Loop of wire (7 pts)
4) Upon using symmetry, F = C and D =
1) At the distance r from the current I0 , the
B . From the circulation theorem for the triang-
magnetic induction B = 2r 0 I0
. Then, the
le CDE, C + E B = . From the no- R l+a
flux through the contour = l Bbdx =
flux-source condition for the vertex with mag- R l+a b0 I0 b0 I0 a+l
nets E, C, B we obtain E = C B . To- l 2x dx = 2 ln l .
gether with the equations form questions 1 and Alternatively, we can find it using the graph
2, A = 32 , C = F = , B = D = by determining the area S under the curve, from
E = 12 . r = r1 = 0, 01 m to r = r2 = 0, 04 m:
S 0, 28 mTm, further, = Sb = 280 Wb.
5) The larger the flux, the more difficult to remo-
ve a magnet, because the magnetic flux needs to2) After switching off the current, the flux th-
rough the tends to zero. From the Ohms law
go through the air gap which will be formed (en-
larging the magnetic energy), when starting theR dq d
dt = dt , hence Rdq = d, i.e. RQ = =
removal. So, the answer is A. . Finally, Q = /R = 280 C.
3) We calculate the force as difference between
7. Passive air-cooling (9 pts) the forces at the two loop segments parallel
1) Using = cp /cV and cp = cV + R we arrive to the straight line: F1 = biB1 and F2 =

at cp = 1 R. biB2 , where i = R1 d dt . So, dp = (F1

2) From the ideal gas equation, p0 = RT (the F2 )dt = bR1 (B1 B2 )d. Using B1 =
ESTONIAN FINNISH PHYSICS OLYMPIAD
PROBLEMS & SOLUTIONS (2005)
1. Rock Climber (6 points) 2. Magnetic break (12 points)  A and B is approximately given
istance between with water; there are holes of net cross-sectional
by R0 = R[ ( + 2) ] (2 pts). area S2 along the perimeter of the tank, which
A rock climber of mass m = 80 kg ascends along Foucault currents can be used to decelerate mov- are open for the operating regime of the pump.
Now suppose the system is rotating with an
a vertical rock. For self-protection, the climber ing metal objects, e.g. a rotating disc. Consider
angular speed = 1 rad/s, and the rails are The height of the tank from the free water sur-
uses the following method. One end of an elastic the following simple model. For the sake of sim- face of the reservoir is h (the height of the tank
uncut.
rope is anchored to the ground. The rope goes plicity, the disc is substituted by a circular rail- itself is small). An electric engine keeps the ves-
3) Sketch an equivalent DC circuit, so that the
through smooth protection loops (carabiners), way, see Figure. Plastic can of radius r =
currents through the resistors are equal to the sel rotating at the angular velocity . The water
which are anchored to the rock. The height of the 15 cm, mass m = 100 g, and height h = 1 cm density is , the air pressure p0 , and the sat-
currents in the respective railway elements:
last carabiner is H = 20 m. The other end of the consists of a homogeneous disc (the bottom of
sleepers and rail segments (between sub- urated vapour pressure pk . Assume the wa-
rope goes through a special braking clip which the can), and of a much thinner cylindrical wall. ter flow to be laminar (neglect the energy of vor-
sequent sleepers; 2 pts).
is tied to the harness of the climber. During the Along the edges of the cylinder, there are two tices) and neglect the friction. Inside the tank,
4) Using the above obtained results, prove that
climb, this clip keeps rope tight, but enables the wire rings (rails), which are connected with a
the (Joule) dissipation power is given by formula there are metal blades, which make the water ro-
climber to lengthen protective part of the rope. set of parallel wire bars (sleepers). Both the
P = kB 2 2 /R, and express the constant k tating together with the tank.
(Assume that the rope between the clip and ca- sleepers and rails are made of a copper wire
(3 pts).
rabiners is always tight) When falling, the max- of diameter = 0,2 mm; the distance between
5) Find the decelerating torque M (2 pts).
imum acceleration must not exceed amax = 5g the sleepers L = h, where = 0,3. Friction-
6) Prove that the angular speed will vanish as
(to protect from injuries). You may assume that less rotation of the system is decelerated with an
= 0 et/ , and determine the time constant
the rope is always vertical, the distance between homogeneous magnetic field (B = 1 T) in the
(2 pts).
the clip and the centre of mass of the climber is slit between the poles of a permanent magnet,
very small, and friction between the rope and ca- see Figure. Assume that the homogeneous field 3. Ballistic rocket (8 p)
rabiners is negligible. Relationship between the fills a region of rectangular cross-section, equal
A rocket is launched from a pole of the Earth
strain and stress of the rope is sketched on the to the area between three subsequent sleepers
with the first cosmic velocity (near-Earth or-
graph below. (i.e. of size h 2h). Outside of that region, the
bital velocity) in such a way that it lands at the
1) Assume that the distance between the climber field is negligible. The specific resistance of the Equator. The radius of the Earth R = 6400 km.
8
and the last carabiner is L (see Figure). If the copper = 1.724 10 m. 1) Find the longer semi-axes a of the rockets or-
climber happens to fall, the distance between the bit (1.5 pts).
highest carabiner and the climber will reach a 2) What is the maximal height of the rockets or-
h

maximal value l (afterwards, the elasticity of the bit h (from the Earths surface; 3.5 pts)?
rope starts lifting the climber). Which inequality N S 3) What is the rockets flight time (3 p)? 1) Calculate the pressure p2 at the perimeter of
should be satisfied for l ? (1.5 pts) h
Remark: The mechanical energy of a planet ro- the tank, when all the holes are closed (2 p).
2

2) Find the maximal safe length L between the tating around a star E = GM m/2a, where G
h

2) From now on, all the holes are open. Find the
climber and the last carabiner (upon reaching of B is the gravitation constant, M the star mass, velocity v2 of the water jets with respect to the
which he has to anchor a next carabiner; 4.5 pts). m the planet mass, and a the orbits longer ground (2 p).
(kN) semi-axes (zero potential energy corresponds to
3) If the tank rotates too fast, the pump eciency
an infinite departure). The surface area of an el-
6 2r A drops due to cavitation: the water starts boil-
L

lipse S = ab, where b is the shorter semi-axes.


ing in some parts of the pump. Find the highest
4 Suppose the system is motionless. 4. Water pump (10 p) cavitation-free angular speed max (3 pts).
1) Find the resistance R of a single sleeper Consider the following construction of a water 4) If the power of the electric engine is P ,
H

2 (1 pt). pump. A vertical tube of cross-sectional area S1 what is the theoretical upper limit of the volume
2) Suppose the rails are cut near the end-points leads from an open water reservoir to a cylindric productivity of the pump max (volume of the
10 20 30 (%)
of a certain sleeper A and B. Prove that the res- rotating tank of radius r. All the vessels are filled pumped water per unit time)?
5. Anemometer (6 points) 6. Mechano-electrical oscillator (7 points) source. We describe the situation in terms of P(W)
plate shift x and capacitors charge q. Derive ex-
Mechanical and electrical processes are some- 300
Anemometer is a device measuring flow rate of pressions for the accelerations x and q in terms
times strongly coupled. Very important ex-
a gas or a fluid. Let us look the construction of X0 , X1 , S, m, k, x and q. Which angular fre-
amples are systems containing piezoelectric ma- 200
of a simple laser-anemometer. In a rectangu- quencies of harmonic oscillation are possible in
terials, e.g. quartz resonator. Here we investigate
lar pipe with thin glass walls flows a fluid (re- the system? (2 pts)
a somewhat simpler situation.
fractive index n = 1,3), which contains light 100
There are two metal plates with area S and 7. Heat exchange (8 points)
dissipating particles. Two coherent plane waves o
mass m. One plate is situated atop of the other 200 400 600T( C)
with wavelength = 515 nm and angle = 1) Consider a simplified model of the air ventil-
one. Plates are connected to each other with
4 between their wave vectors, are incident on ation system of a house using a passive heat ex-
springs, whose total spring constant is k and 8. Balloon (8 points)
a plate so that (a) angle bisector of the angle changer. The exchanger consists of a metal plate
what are made of insulator. The lower plate is Find the mass of the balloon (including the
between wave vectors is normal to one wall of of length x and width y and thickness d dividing
mounted on a steady base. Equilibrium distance gas inside it). Equipment: balloon (floats in
the pipe and (b) pipe is parallel to the plane the air channel into two halves, one for incoming
between the plates is X0 . air), digital scales, rope, measuring tape, rope
defined by wave vectors. Behind the pipe is a cold air, and another for outgoing warm air. Both
photodetector, that measures the frequency of fasteners, dynamometer, paper sheets for folding
channels have constant thickness h, air flow ve-
changes in dissipated light intensity. and rough angle measurements, 100-g weight,
locity is v see Figure. Thermal conductance of
thread.
1) How long is the (spatial) period of the inter- the metal is (the heat flux through a unit area
Remark: You may find it useful to know that
ference pattern created along x-axis (see Figure; of the plate per unit time, assuming that the tem-
1) Let us assume that there is a small vertical if a rope is tied around a balloon so that the ten-
2 pts)? perature drops by one degree per unit thickness
shift x of the upper plate from its equilibrium po- sion of the rope is T , the excess pressure inside
of the plate). Specific heat capacity of the air by
2) Let the oscillation frequency of the photo- sition. Derive acceleration x of x in terms of sys- the balloon is p, the angle between the tan-
constant pressure is cp , air density is (neglect
meter signal be = 50 kHz. How large is the tem parameters. What is the angular frequency gents of the balloon envelope near the rope is 2
its temperature dependance). You may assume
fluids speed v? What can be said about the dir- 0 of the small vertical oscillations of the upper (average over the perimeter; see Figure), and the
that the air is turbulently mixed in the channel,
ection of the fluid flow (2 pts)? plate (1 pts)? radius of the circular loop formed by the rope is
so that the incoming and outgoing air temperat-
R, then p = T tan /R2 . The universal gas
3) Let us consider a situation, when the 2) Plates are now connected to a constant high ures Tin and Tout depend only on the coordinate
constant R = 8.31 J/Kmol, the molar mass of
wavelengths of the plane waves dier by = voltage source, so that they form a capacitor. x (the x-axes is taken parallel to the flow velo-
air = 29 g/mol.
4,4 fm (1 fm= 1015 m). What is the frequency Electrostatic force between the plates causes an city), i.e. Tin Tin (x) and Tout Tout (x). As-
of signal oscillations now (fluids speed is the additional shift of the upper plate. The equilib- suming that the inside and outside temperatures 2
same as in previous section)? Is it possible to de- rium distance between the plates is now X1 . De- are T0 and T1 , respectively, what is the temperat-
termine the flow direction with such a device (2 rive expressions of the electrical attractive force ure T2 of the incoming air at the entrance to the
pts)? Fe and voltage applied to the plates U in terms room (4 pts)?
of X0 , X1 , S, m and k (2 pts). cold air
photodetector
metal plate
3) System is set to oscillate again, keeping
x
flow

warm air
voltage U constant. Let x stand still for the shift
d from the equilibrium position. Derive an expres- 2) Attached is a plot of the heat exchange rate P
w a ve 1 re
a tte sion for the acceleration x of x in terms of X0 , of the wire of an electric heater as a function of 9. Mechanical black box (7 points)
sc ht
li g X 1 , S, m, k and shift x. What is the angular fre- temperature (assuming the room temperature is There is something small inside the cylindrical
quency 1 of upper plates small vertical oscilla- T0 = 20 C). The operating temperature of the black box. Find the mass of it, as well as the
2 tions (2 pts)? wire is T1 = 800 C. The heater is switched o; friction coecient between it and the inner sur-
wave
4) Let us modify the situation of the previous find the time after which the temperature of the face of the box. Equipment: black box, ruler, a
question and connect an inductor with induct- wire will drop down to T2 = 100 C. The heat ca- wooden plank, timer, scales.
ance L in series to the capacitor and voltage pacitance of the wire is C = 10 J/K (4 pts).
1. Rock Climber Notice also that we can take account symmetry and connect points B
1) In the case of falling, the acceleration should not exceed 5g , which with equal potential; this allows us later to simplify cyclic railway to O
previously solved infinite (actually, very long) railway. We can also see
m g < 5g . Maximum strain is the solution of the fol-
means that ()
that there is no current between the two sleepers residing in the mag- C
lowing equation () = 6gm = 69.8 sm2 80kg = 4.7kN . According
netic field (there is no potential dierence), hence we can disconnect
to the graph, = 0.315; hence, l < 0.315(L + H) + L
them. So, we can obtain two indipendent (almost) infinite railways and
2) In the case of falling, the climber reaches the lowest point, when its
both have their own source of elecromotive force. O
velocity become zero. This means that the energy absorbed by the rope
R
becomes equal to the change of the potential energy: A
E = mg(2L + x),
where x = l L. Energy absorbed by the rope is given by R
  
E= ()dx = ()(L + H)d = (L + H) ()d.
2) The ellipse has a property that the sum of lengths from each point on
We know that the maximal value is = 0.315, which makes it possible the orbit to the both foci of the orbit is constant (equals to 2a). Hence,
to calculate the integral numerically, as the area under the graph. 4) Electromotive force in the sleeper is E = Bvh. Energy is dissipated the other focus (i.e. which is not the centre of Earth) is at the distance R
 0.31 E2
into heat P = Rcircuit . where Rcircuit = 12 (RR + 2R + R), from both the launching point and landing point, see Fig. So, theheight
S() = ()d 564.8N
h = |CB| = |OB| R; since |OB| = R + 12 |OO | = R(1 + 22 ), we
0 1 
Thus, Rcircuit = R( (2 + ) + + 1). finally obtain h = R2 .
2
(L + H)S() = mg(2L + x) = mg(2L + (L + H)), 3) The ratio of the flight time to the period along the elliptic orbit
Consequently equals to the ratio of two surface areas: the one painted dark grey
hence
H(mg S()) 2B 2 2 r2 h2 in Fig, and the overall area of the ellipse. The rotation period is the
L= 5.08m. P = 
S() mg( + 2) R( (2 + ) + + 1) same as in the case of  near-Earth orbit (due to Keplers third law),
So, the new carabiner must be anchored within next L = 5.08 m. Eventually, T = 2R/v = 2 R/g . The dark gray surface area is calcula-
ted as the sum of half of the ellipse area, and a triangle area. So,
2. Magnetic brake 2r2 h2
k=  2.12 106 . = T ( 2 R R2 + R2 /2)/R R2 = ( + 2) R/g.
1) Sleeper is a simple cylindrical conductor: (2 + ) + + 1
h 4. Water pump
R = 2 5.59m. 5) Since the power equls to M = P , the torqe can be found as
(2) 1) Let us consider the process in the system, rotating together with the
2) Length of the railway element is R, hence the resistance is R2 = P 2B 2 r2 h2 tank. Then, there is a potential energy related to the centrifugal force:
R. Main ideas: first - we can imagine that railway is infinite; second M= =  0.39 mNm. r
Uc = 0 2 rdr = 12 2 r2 . So, the pressure p2 = p0 gh + 12 2 r2 .
R( (2 + ) + + 1)
the resistance (RR ) of this infinit array remain same even if we cut 2) From the Bernoulli formula, 12 u2 = p2 p0 = 12 2 r2 gh, hence
of one periodic element. Hence, 6) Disc has a momentum of inertia eual to I = 12 mr2 ; the angular the squared velocity in the rotating reference system u2 = 2 r2 2gh.
 laboratory speed v2 = u + r = 2( r gh), i.e. v2 =
R(2R2 + RR ) 2 2 2 2 2
RR = . acceleration = M d
I = dt . Consequently (using decelerating M),
The
2R2 + RR + R 2( r gh).
2 2
After solving the equation kB 2 d
  = . 3) The point of lowest pressure pm inside the pump is the upmost point
IR dt of the tube. Using the Bernoulli formula, p0 = pm + gh + 12 v12 ,
RR = R2 R22 + 2R2 R = R22 + 2R2 R R2
and noting that the negative solution of the equation has to be dropped If we group the variables I and t into dierent side of the equation, we where the velocity in the tube can be found from the continuity condi-
(it does not have physical meaning), we arrive at obtain tion: S1 v1 = S2 u = S2 2 r2 2gh. Therefore, pm = p0 gh
 kB 2 d 1
( 2 r2 2gh)( S 2 2
) . Notice that the boiling starts when pm = pk .
RR = R( ( + 2) ). dt = . 2 S1
IR
3) Important ideas: So, m r = 2gh + ( p0 p
2 2 k
gh)( SS21 )2 ; finally we obtain
Integrating the both sides of the equation yields
  
  2
electromotive force is generated when conductors move in mag-
t
kB 2
d kB 2 1 p0 pk S1
dt = t = ln , m = r 2gh + gh .
netic field; IR IR 0 S2
0 0
4) The maximal productivity is apparently achieved for the highest ef-
There is always two sleepers moving between magnets (in mag- kB 2
= 0 e IR t, and finally = IR
kB 2
2.9s. ficiency. The eciency is highest, when the residual velocity is lowest:
netic field);
u 0, and min . According to the results of the second ques-
Those sleepers act as a sources of electromotive force (like a bat- 3. Ballistic rocket tion, min = r1 2gh. So, the minimal residual velocity of the wa-
tery); ter streams is vmin = min r = 2gh. The associated lost power is
1 2
1) The net energy depends only on the longer semi-axes. Hence, the 2
vmin = gh. The useful power is associated with the potential ener-
those sleepers also have internal resistance R. longer semi-axes is the same as in the case of near-Earth orbit: a = R. gy increase (by gh),i.e. the total power P = 2gh. Hence, = P/2gh.
d 1
5. Anemometer Q = S0 U/X1 ; hence dX1 C = 1/S0 , and 8. Balloon
q x
1) First we need to find the angle after the refraction : For small inci- Lq = U . We can measure the lift of the ball by attaching a weight M = 100 g to it
dence angles we find approximately = /n. In the liquid, the wa- C X1 and taking the reading of the scales F/g = m+M V = 73.4 g, whe-
Here, the sign of the second term assumes that the x-axes is directed
velength is decreased n times:  = /n. The requested wavelength upwards (there is no current in the inductance and Lq = 0, if the vol- re = p0 /RT 1.2 g/l. Hence, V M = 27.6 g. So, we need to de-
can be found as the distance between the lines connecting the intersec- tage on the capacitor keeps constant; for increasing charge q > 0, this termine the volume of the ball. To that end, we tighten the rope around
tion points of the equal phase lines of the two beams. Alternatively (and assumes increasing capacitance, i.e. x < 0; in a full agreement with the the ball as tightly as the fasteners can hold (note that smaller tension
in a simpler way), it is found as the dierence of the two wavevectors: signs of the above expression). values would results in a too small volume decrease, and hence, in a lar-
k = k , where k = 2/ = 2n/ is the wavevector of the incident The second equation describes the Newton second law. First we no- ge uncertainty of the final answer). We weight the ball with rope (93.3
beams. So, = 2/k = / 7,4 m. te that the expression for Fe can be rewritten as Fe = Q2 /2S0 . So, g) and subtract the mass of the rope (19.1 g) to find V  M = 28.4 g,
2) The scattered light fluctuates due to the motion of the scattering if the charge on the plate does not change (q = 0), neither does chan- where V  is the ball volume, when the rope is tightened. Hence, the
particles; the frequency is = v/ = v/. There is no way to de- ge Fe . So, Fe = q dQ d
Q2 /2S0 = qQ/S0 . The infinitesimal force volume decrease V = V V  = (28.4 27.6)/1.2 l .67 l. La-
termine the direction of the flow, but the modulus is obtained easily: ter, we determine the maximal tension in rope (which can be hold by
changes (Fk and Fe ) can be simply added:
v = / 0.37 m/s. mx = kx qQ/S0 . fasteners) with dynamometer, T 30 N. We also estimate the average
3) The spatial structure of the interference pattern remains essentially Now, let us look for a sinusoidal solution of circular frequency . angle tan 1.4. The radius of the loop is calculated from the measu-
unchanged (the wavelength dierence is negligible). However, the pat- Then, x = 2 x and q = 2 q . Substituting this into the two above rement of the perimeter, R 15 cm. According to the given formula,
tern obtains temporal frequency = (c/) c/2 . The velocity obtained equations,
we find these data correspond to p 1900 Pa. Due to gas law, p p0
V
V
,
of the interference pattern u = = c . If the fluid speed is (L 2 C 1 )q = xU/X1 hence V p0 V 33 l. Therefore, M = V 27.6 g 12 g.
p
v 0.37 m/s, then the relative speed of the pattern and the fluid is ( 2 m k)x = qQ/S0
.
 = c v , depending on the direction of the flow (in both cases, 9. Mechanical black box
This has a non-zero solution for x and q only if
 740 kHz). So, the output frequency allows us to determine the (L 2 C 1 )( 2 m k) = U Q/X1 S0 . First we determine the coecient of friction as follows. We make sure
flow direction as long as we can be sure that the interference pattern Bearing in mind that U Q/X1 = 2k(X0 X1 ) and C = 0 S/X1 , we that the object is at the bottom of the box. We put the box on the plank
velocity is larger than the flow velocity. can rewrite the equation as so that the axes of the cylinder is parallel to the axes of the plank. We
(0 SL 2 X1 )( 2 m k) = 2k(X0 X1 ). start inclining the plank so that the bottom of the cylindrical box gets
6. Mechano-electrical oscillator higher. We determine the angle of the plank , when the object starts
Introducing 02 = k/m and 12 = X1 /0SL we canfurther rewrite as
1) From the Newtons second law, mx = kx, hence x = m k
x, hence X0 sliding: it hits the cover of the box. It is convenient to put the cover
 4 2 (12 + 02 ) + 02 12 3 2 = 0.
= k/m. X1 (and the rim of the cover) of the box hanging slightly over an end of the
2) From the Gauss law, the charge on the plate Q = S0 E = Therefore,  plank. In that case, object hiting the cover of the box results in box fal-
S0 U/X1 . The force acting on it Fe = k(X0 X1 ) = Q E, whe- 2 2 = 12 + 02 14 + 04 + 212 02 (X0 X11 5), ling down from the plank. We measure the tangent of the plank at that
re E is the average electric field (averaged over the charges). Let us i.e. this system has two eigenfrequencies, if X 3
X1 < 2 (and becomes uns-
0
moment: = tan = 0.17 0.2.
look at the charge layer (at the surface of the plate) with a high mag- table, otherwise). Now we turn the axes of the cylinder perpendicular to the axes of
nification: the electric field there depends linearly on the net charge the plank, but keep lying on its side. We start again inclining the plank
7. Heat exchange
inwards (in the plate) from the current point. Therefore, the average and determine the angle , at which the box starts rolling down. As-
field is just the arithmetic average of the fields on both sides of the layer: 1) It is easy to see that the temperature profile along the plate is linear, suming that the object is small (as compared to the radius of the box),
E = E/2. Finally, Fe = k(X0 X1 ) = QE/2 (this result could
and the temperature dierence T between the two plates is constant, the following relationship can be derived: M sin = m(sin sin ),
T T0 T2 . Indeed, then the heat exchange rate q (per unit plate where m is the mass of the object and M the mass of the empty box.
have been obtained from energetic considerations, using infinitesimal
area) is also constant, which in its turn corresponds to a linear tempe- Using sin = 605 253
virtual displacement of the plate andthe energy conservation law). So, 350 and sin = 350 we obtain M/m = 35/25 =
rature profile. Let us use a reference frame moving together with the 1.4 0.2. From the measurement of the net weight M + m = 10.4 g
Fe = S2 0 (U/X1 )2 , hence U = X1 2k(X0 X1 )/S0 . incoming air. Then, the temperature increase rate at a given point is
3) If the plates move by x, the change of the force due to electric field we find m = 10.4 g/2.4 = (4.3 0.4) g. Note that the actual mass was
T = v(T2 T1 )/x. Then, the heat balance for a air element of volume 4.5 g.
is Fe = x| dX d S
1 2
0 (U/X1 )2 | = Xx1 S0 (U/X1 )2 ; bearing in mind V = s h is written as shcp T = q = sT /d = s(T0 T2 )/d. The mass ratio can be, in principle, determined from the period of
that S2 0 (U/X1 )2 = k(X0 X1 ), we obtain Fe = 2 Xx1 k(X0 X1 ). So, shcp v(T2 T1 )/x = s(T0 T2 )/d, hence small oscillations, T 0.4 s. Then, if we estimate the moment of iner-
There is also force cahnge due to elasticity: Fk = kx; the two forces xT0 + hcp vdT1 tia of the system box+object as (M + m)r2 , where r is the radius of the
T2 = .
have opposite sign (while approaching the discs, Fk tries to push back, x + hcp vd cylinder, then I = mrg, i.e. 2 = m+M m g
r . Using r = 16 mm
and Fe tries to pull disks even closer). So, F = kx[1 2( X X1
0
2) Rewriting the heat balance equation P = C dT as M+m g 2 2
1
dt we obtain m = r T /4 = 3.9. This result, however, is rather
1)] = kx(3 2 X1 ). Finally, x = F/m = x m (3 2 X1 ), and
X0 k X0
dt = CP dT we conclude that time can be found
via the area S under the graph, where P 1 is plotted
approximate, because it is dicult to measure such a short oscillation
= k
2X period (at the high dissipation rate). Furthermore, the period is taken
m (3 X1 )
0
versus the temperature as t = SC . The graph data: to the second power, this explains the unrealistic result (so,it does not
4) Now we have two oscillating variables, x and q . First, we write down T (K) 100 200 300 400 500 600 700 800 make sense to try to improve the approach by taking account the geo-
the equation due to Kircho s laws: Lq = Cq xQ dX d
1
C 1 . He- P (W) 13 30 55 83 122 177 258 395 metric factors for the calculation of the moent of inertia of the box etc).
re, the second term describes the voltage change on the capacitor due 100P 1 7.7 3.3 1.8 1.2 .82 .57 .39 .25
to the change of the capacitance (we approximate the real change by Substituting the region with smooth boundaries with a superposition
dierential, valid for small shifts x). Note that C 1 = X1 /S0 and of trapezoids we find S 12 K/W. Consequently, t = 120 s.
ESTONIAN FINNISH PHYSICS OLYMPIAD
PROBLEMS & SOLUTIONS (2006)
1. Drying (12 p) r > 100% is formed)? through the LED, it emits light.
4) If you happened to obtain r > 100% then
According to the wide-spread belief, it is useful
the oversaturated vapour breaks down into a fog
to keep window open wen drying laundry even if
which contains tiny water droplets. In that case,
the relative humidity outside is 100%, because
what is the mass m of the condensed water (i.e.
the temperature of the incoming air rises and
the total mass of the water droplets)? Air dens-
thereby the relative humidity drops. Let us ana- 3
ity 0 = 1,189 Kg/m ; latent heat of vaporiza-
lyse, do these arguments hold, when heating is
tion for water q = 2500 kJ/kg.
switched o. If a capacitor of capacitance C and res-
Suppose that inside a room, the volume of 2. Photography (7 p) istor of resistance R are connected in series to
air V1 = 20 m3 from inside at the temperat- By taking measurements from the photo at the an electromotive force E then the capacitors
ure t1 = 25 C is mixed with the volume of end of the page, determine the diameter of the voltage will approach exponentially its asymp-
t/RC
air V2 = 10 m3 from outside at the temperat- camera lens used for this photo. You can as- totic value: U = E U0 e .
ure t2 = 1 C . The specic heat of the air (by sume that images created by this camera lens
xed pressure) cp = 1005 J/(kgK) can be as-
are identical to ones created by ideal thin lens
sumed to be constant for the given temperature
of matching focal length and diameter.
range; the heat exchange with the medium can
be neglected. For the time being, you may neg- 3. Sucking (7 p)
lect the possibility of (partial) condensation of Let a large vessel be lled with an incompress-
the vapour. ible dielectric liquid of density of mass density
1) Prove that the total volume of the air will m (the relative dielectric permeability 1).
not change i.e. that the volume of the mixed air This liquid carries homogeneous volume charge
V = V1 + V2 . of density e which is so small that the electric
2) What is the temperature of the mixed air T? eldE0 created by it is negligible: E0 e gm ,
(g/m3) where g is the free fall acceleration. Surface ten-
sion can be also neglected. All the heights will
be measured from the unperturbed height of the
liquid surface. A point charge q of opposite
2,0 sign is brought to the height H, due to which a
kink is formed on the liquid surface.
1) Determine the height of the kink a.
2) If the height of the charge is slowly decreased,
1,5 at which height h the liquid will start owing to
the point charge?

4. Electric experiment (12 p)

1,0 Find the capacitance of an unknown capa-


citor and estimate the experimental uncertainty.
Equipment: red light emitting diode (LED),
three resistors one of resistance R1 = 1.5k one
0
t ( C) of resistance R2 = 6.2k, and one of unknown
resistance; a battery of unknown electromotive
0 5 10 15 20 25 force (internal resistance is smaller than 500),
3) The graph below shows the dependence of the wires, timer, unknown capacitor.
saturated vapour density for water as a func- Remarks: below is provided a typical V I
tion of temperature. Before mixing, both the curve of a LED; during this experiment, the
interior and exterior air had relative humidity V I curve of the LED can be approximated
of r0 = 100%. What is the relative humidity with that of an ideal diode, cf. graph. The
r of the mixed air (if it happens to increase value of the opening voltage Uc of the LED is
then assume that an oversaturated vapour with not known. When there is a non-zero current
5. Empty bag (12 p) 6. Car (7 p) between the entry and exit points is l (see gure) the droplet after second refraction. Note that
the internal reection is only partial, and not
A car attempts driving over a road barrier, start-
A cylindrical bag is made from a freely deform- ing from rest, as shown in gure. The dia-
total, see gure. The exit angle has a max-
able fabric, impermeable for air, which has sur- imum as a function of the impact parameter
meter of its wheel d = 1 m, coecient of fric-
face mass density ; its perimeter L is much tion between the wheels and ground = 1. The
b; the angular radius of the rainbow equals to
less than its length l. If this bag is lled with the maximal exit angle [indeed, if light of in-
sketch is drawn using correct proportions, point
air, it resembles a sausage. The bag is laid on C marks the position of the car's centre of mass.
tensity I0 falls onto the droplet with all pos-
a horizontal smooth oor (coecient of friction Determine the maximal height of those road bar-
sible impact parameters b < r, the light en-
= 0). The excess pressure inside the bag is p, riers which can driven over with such a car, as-
b is 2I0 bb;
ergy per impact parameter range
the free fall acceleration  g. The density of air . hence, the energy per exit angle interval
suming that there is no drag in the non-driving
1
is negligible. 1) Express the mass M of those ions which hit is I/ = 2I0 bb// = 2I0 b(d/db) ,
axis (i.e. wheels rotate without friction), and the
the centre of the detector via the quantities B, which diverges near the maximum of the func-
1) Find the width of the contact surface between driving wheels are
tion (b).]
the bag and the oor c. 1) front wheels; l, U and e.
2) rear axis. 2) The entrance to the detector is a circle with
2) Prove that the tension in the bag's fabric radius r (r l). Because of the nite size of the
3) Suppose that the car has four-wheel-drive,
T = x + , where x is the height of the given detector entrance, ions within a mass range from b
and the road barrier is substituted with a wall.
point P above the oor, and nd the coecient M M to M + M will enter the detector;
Would it be possible to rise the front of the car
. Remark: fabric's tension is (loosely speak- nd the range width M .
by driving slowly against the wall?
ing) the force per unit length. Actually, fabric's
3) Under the assumptions of the previous ques-
tension is not as simple concept as a rope's ten-
tion, what is the width of the range of the exit
sion, because dierent force directions are pos-
angles of those ions which can still hit the
sible; here, however, we neglect the force com-
detector?
ponent along the sausage's axis. Thus, we can C 4) Now let us assume that r is negligibly small,
describe the fabric's tension here with a single
but we cannot neglect the thermal eects (still
number, the force per unit length T acting across
eU kT ). Due to dierence in energies, the
an horizontal cut of the sausage.
detector can be hit by ions of dierent masses,

Hint: consider the force balance for a small within the range from M M to M + M .
piece of fabric (using the vertical cut shown in What is the resolving power M of the mass-
7. Mass-spectrometer(9 p) spectrometer?
gure).
In the gure below, a simplied scheme of a
3) Let the highest point of the bag be at height 8. Optics experiment (10 p)
mass-spectrometer is given. It is a device for
a from the oor. What is the tension T1 at this
Equipment: a cylindrical bottle lled with wa-
measuring the masses of molecules. The sub-
highest point? Express your answer in terms of
stance under investigation is ionised by heat- ter and having a millimetre scale across half of
a, and p (or , if you were unable to answer
its perimeter (seen when looking through the
ing up to a temperature T on a hot lament
the previous question).
(molecules undergo a single-electron ionisation). bottle); measuring tape.
Hint: consider the force balance between two The ions are accelerated using voltage U. At 1) Determine, how long arc of the bottle's meas-
halves of the bag. rst, let us neglect the thermal energy of the uring scale could be seen simultaneously if look-
ions (eU kT , where e is elementary charge ing at it through the bottle from a very distant
4) Assuming that p g , determine the quant-
ba and k the Boltzmann's constant). A nar- point (much larger than the bottle's diameter),
ity = b+a , where b is the width of the bag.
row beam of accelerated ions enters a region assuming that the observation point lies at the
l with magnetic eld. For the sake of simpli- height of the millimetre scale.
city, let us assume that the region has a rect- 2) Using the results of the previous experiment
angular shape, and the magnetic eld is ho- determine the angular radius of a rainbow (the
T angle between a ray coming to the observer's eye
mogeneous inside it. The magnetic elds de-
P ects the ions and depending on their mass, they from a rainbow, and the axis of the cone formed
a
b may hit the detector. Let us assume that those by other such rays).
T x
ions which hit the centre of the detector enter Remark: Rainbow is formed due to those
and exit the region with magnetic eld perpen- rays which enter a spherical water droplet, re-
c
dicularly with its boundary, and the distance ect once from its surface internally, and exit
1 e q
1. Drying 4) In order to find the condensating mass, we write 40 m g
= A, the result can be written as Performing for both cases 35 measurements
down heat balance: cp 0 t = q[a a (t + t)], 1 p and finding the average (1 37 s, 2 32,4 s), we
1) Let the number of moles of cold and warm air be where t is the temperature change due to the con- 0 = (H H 2 4A). find C 13 F.
2
1 and 2 ; letCV designate the molar heat capaci- 2) It is clear that flowing starts at the point x =
densation. By designating t + t = we can rew-
tance at a fixed volume. Then the total change of in- 0, where the fluid surface is the highest. When
rite the balance as a ( ) = a cp 0 ( t )/q .
ternal energy is U = CV [1 (T T1 ) + 2 (T the flowing starts, this surface point [with coordina-
So, we need to find the intersection point E of the
T2 ) = (CV p0 /R)(V V1 V2 ) (using the ideal tes (0, 0 )] realizes the potential energy maximum,
curve a ( ) with the line a cp 0 ( t )/q =
gas law). Internal energy change must be equal to the
a 0,478 gm3 K1 ( t ) (line C 0 E in Fig.). when moving along the y -axes towards the charge.
work of the external pressure: (CV p0 /R)(V V1 Using the graph we find 0,25g/m3 this is So, the function (y) = m gy 4 1
e q/(h y)
V2 ) = p0 (V V1 V2 ), hence V V1 V2 (since 0
has a maximum at y = H0 . This gives us two equa-
the length of the line with arrows. So, the condensa-
CV /R 6= 1). tions:
ting mass m = (V1 + V2 ) 7,5g.
2) The molar amount of gas (p0 /R)(V1 /T1 + Thus, when meteorologists tell us that at the 1
V2 /T2 ) = (p0 /R)(V1 + V2 )/T , hence T = (V1 + m g0 4 e q/(h 0 ) = 0,
meeting point of cold and hot air, there are heavy 0
V2 )/(V1 T11 + V2 T21 ), i.e. t 16,5 C. rains, the phenomenon can be explained by this m g 40 e q/(h 0 )2 = 0.
1

3) The vapor mass ma = a (t1 )V1 + a (t2 )V2 , problem. Comparing these, we find h = 20 and 20 =
1
40 e q/m g , hence
the mass of saturating vapor at the given tempera-
ture mak = a (t )(V1 + V2 ). Relative humidity
2. Photographing p
r = ma /mak , because at the fixed temperatu- h= e q/0 m g.
re, the pressure is proportional to the density. So, Let us notice that at the lower part of the photo, the- 4. Electrical experiment
r = a /a (t ), where the weighted average of the re are few brighter spots of regular circular shape and
vapor a = [a (t1 )V1 + a (t2 )]/(V1 + V2 ) this clear edges unlike all the rest at the smudged (out We start with charging the capacitor (waiting long
value can be found from the graph as the coordinate of focus) part of the image. This can be only due to enough, to allow equalizing the voltages of the source
of the point C : we draw the line at + b, connecting the point sources in that far area. Let the distance and the capacitor, of the order of the discharge time
points A and B , and take the reading for the point C of the linear from the lens be l, and the distance be- below). The capacitor will be discharge on the diode
lying on the line at +b 1,68 g/m3 at t = 17 C tween the sensor and the focus x. Then, according and two resistances (the unknown one r is parallel
(this value divides the interval [t2 ; t1 ] in the propor- to the Newton formula, x(l f ) = f 2 , where f is the to the diode), using the scheme in the figure. We
tions V1 : V2 ). The saturating vapor pressure at the focal distance; hence lf = fx . Let the spot diame- perform two experiments using for the sequentially
f
given temperature is found as the coordinate of the connected resistor R the both supplied resistors with
ter be . Then the lens diameter d = fx = lf
f
. Let
point D: pa (t ) 1,38 g/m3 . Finally we obtain known resistance, R = R1 and R = R2 .
the size of the image of the linear be a, and the size
r 1,22 = 122%. of the linear itself A. Then A = a x+f l
. From the r
(g/m3) 1 lf lf
lens formula, x+f = f l , hence A = a f . Com-
R
B paring with the previous result we obtain d = A/a,
i.e. the lens diameter equals to the spot diameter, Initial voltage of the capacitor U0 = E ; the voltage
2,0 using the scale of the linear. From the figure, we find drop on the diode is constant (while emitting light)
d = 17 mm. exactly as on a voltage source. Therefore, the voltage
on the capacitor approaches that value exponential-
C
C 3. Sucking ly:
U Uc = (E Uc )et/RC .
1,5 1) Let x be the horizontal axes, and y the ver-
E Diode stops burning, when all the current I = (U
D tical axes. At the liquid surface, the potential energy Uc )/R goes through the unknown resistor, I =
of a unit volume is constant (so that the liquid will Uc /r. Thus, at the fading moment (t = ):
not flow towards the lower potential energy). So, the r(E Uc )e /RC = RUc .
1,0 formula for the height (x) of the liquid surface is
1
Rewriting the latter equality for the both experi-
given by vp = m g 4 0
e q/r = 0, where ments,
p
r = x2 + ( H)2 is the distance of the given r(E Uc )e1 /R1 C = R1 Uc .
A point from the charge. Let us designate 0 (0).
t (0C) r(E Uc )e2 /R2 C = R2 Uc .
From the previous formula we obtain (bearing in Dividing these and taking the logarithm results in
0 5 10 15 20 25 mind that for x = 0 we have r = H 0 ) the result 2 1 R1
0 (0 H) + 41 e q
= 0. Using the designation C=( )/ ln .
0 m g R2 R1 R2
5. Empty sack point Q intersection point of the lines of the case of very large distance (black line d). In the lat-
1) The pressure at the floor P = p + g , hence resultant forces applied to the touching points of ter case, the ray (in Fig, a) is refracted at the entrance
Lg = (p + g)c, from which c = L/( g p
+ 1). the front- and rear wheels with the wall and floor, to the bottle by a certain angle ; when observing from
2) Here we provide a solution departing from the respectively. Only the gravity force can contribute to smaller distances, one ray (b in Fig) is refracted by the
recommendations (finding the other solution is left the net torque; since Q lies leftwards to the center of same angle. These two rays coincide after rotation by
for the reader). Let the tension of the material at mass, this torque rotates car rising its front. So, the an angle around the center of the bottle. So, the
some contact point with floor P0 be T0 . Consider front will start rising. part of the scale, given by the gray line in Fig, is lon-
the energy balance of a piece of material between the ger than the black line at least by 2R . We should
points P and P0 for a tiny virtual displacement , perform the measurements with as large L as pos-
tangential everywhere to the material (thus, the sha- Q sible; the result of the measurement is to be adjusted
pe of the material is preserved). The potential energy O by subtracting 2R , where = arcsin(l/2L).
a
change (per unit length of the sack) is gx (because
the piece of material of length will get from the floor b
to the height x); the work done equals to (T T0 ) . C L c d
The energy balance yields T = gx + T0 , hence l
= g . R P
3) The force balance between the left and right halves
Alternatively, we can measure c by different values of
of the sack can be written as T1 + T0 = pa. Bearing
L, and present the results on graph. It makes sense
in mind that T0 = T1 ga, we find T1 = (p+g) a2 .
to use 1/L as the scale for the horizontal axes. Then,
4) The force balance between the lower and upper 7. Mass-spectrometer L = represents the origin, to which the curve can
halves of the sack: 2T2 + L1 g = pb, where T2 is
1) The trajectory of a charged particle be easily extrapolated).
the tension at the widest point, and L1 L/2 in the magne-
tic field is circle of radius R = l/ 2. Lorenz force The measurements yield d 22 mm (by R =
is the length of the upper half. The tension T grows
is responsible for the acceleration, Bev = M v 2 /R, 31 mm).
linearly with the height, and the widest point is app-
hence BeR = p. Substituting p2 = 2M U e = 2) Comparing the ray geometry for the previous
roximately at the half height; hence 2T2 T1 +T0 =
B 2 e2 R2 , we obtain problem (in connection to the piece scale c), and the
pa. Substituting it into the first equation, we obtain
M = B 2 l2 e/4U. ray geometry in the rainbow, it turns out that the geo-
p(b a) = Lg/2. Taking into account that the
2) Now, the radius can beR r. Approximate calcu- metry is actually identical, with d2 = R 2 , see Fig. So,
sack is almost of a circular cross-section, we write
lus yields R/R = r 2/l M/2M , hence = d/R. Using the data from the previous part,
(b + a) 2L; hence, we finally obtain g 4p .
M M r2 2/l, i.e. 41 .

6. Car M = B 2 lre/ 2U.
/2 /2
1) Let us consider the force balance projected to the 3) Ion leaves the magnetic field at the distance r be-
horizontal axes. The only force, which could create fore (or after) performing
a quarter of the circle. So, d
a non-zero projection, is the resultant of the friction r/R = r 2/l.
and reaction force, applied by the corner of the de- 4) Certain initial energy kT implies that the terminal
limiter. Due to the balance, this must be also zero, energy U e+kT = e(U +kT /e); this is equivalent to
i.e. this resultant
force is directed vertically, hence the change of the voltage by U = kT /e. Using app-
H = d4 (2 2) 15 cm. roximate calculus and the result of the first question,
2) Consider the torque balance with respect to the we obtain: M = dM dU kT /e, i.e.
point O the intersection point of the lines of the M = B 2 l2 kT /4U 2 .
resultant force applied to the rear wheel by ground,
and of the gravity force (vertical line through C ). At 8. Optical experiment
the equilibrium, the line of the reaction force applied 1) Looking at the bottle from a distance reveals that
to the front wheel by the delimiter must go through the central part of the scale is not reversed, unlike the
the same point. Thus, the intersection point of the image at the extreme edge of the bottle. The turning
line OP with the wheel gives us the corner of the de- point corresponds to an one end of the visible part of
limiter (P is the center of the front wheel). Using the the glued scale (the other end-point is symmetrical-
scale of the figure yields H 10 cm. ly situated). Looking from smaller distances results
3) Consider the torque balance with respect to the in large visible part (Gray line c in Fig.), than in the
ESTONIAN FINNISH PHYSICS OLYMPIAD
PROBLEMS & SOLUTIONS (2007)
1. Wire (7 pts) time are to be gathered together (focused) at the probe D, which is at
A conducting wire is formed of a cylindrical copper core with a diameter distance b from the gap C . In order to analyze this setup, answer the
of a = 2,5 mm. The core is wrapped in a concentric, cylindrical alumi- following questions.
nium coating, the total diameter of the wire being b = 4 mm. A current A e B U(t) D
of I = 2,4 A flows through the wire. The specific resistivity of copper is
c = 0,0168 106 m and of aluminium, a = 0,028 106 m. C
1) What are the current densities j in different parts of the wire (current a b
density is defined as the current per cross-section area)?
2) What is the magnetic inductance B1 at the distance c = 1 cm from 1) Assuming that U (t) 0, what is the time needed for the electrons
the axis of the wire? to travel from the gap B to the probe D?
3) What is the magnetic inductance B2 at the surface between the cop- 2) Find the same travel time assuming that the voltage U (t) U  U0
per and aluminium? R is constant (your approximating expression should be a linear function
Remark It may be useful to know the circulation theorem: B ~ =
~ dl of U ).
0 I , where the integral is taken along a closed trajectory (loop) and I is 3) What functional equation should be satisfied for the waveform U (t)
the net current flowing though that loop; 0 = 4 107 Hm1 . This in order to ensure the focusing of all the electrons at the probe D. Solve
formula is completely analogousRto the formula for the work done by a this equation by assuming a  b and |U (t)|  U0 .
force along a trajectory: F~ , A = F~ dl~. 4) The waveform generator yields a periodic signal of period T in such
a way that the profile U (t) is followed up to achieving some maximal
2. Pendulum (7 pts) value Um ; after that, the voltage drops immediately to 0 and the process
Consider an elastic rod, the mass and the compressibility of which (i.e. starts repeating. What is the fraction of electrons missing the time
the length change) can be neglected in this problem. It can be assumed focus at the probe D?
that if one end of the rod is firmly fixed, and a force F is applied to the
4. Coefficient of friction (12 pts)
other end of the rod, perpendicularly to the rod at the point of applica-
tion, then the rod takes a form of a circle segment. The radius of that Equipment: a wooden brick, a spherical ball, board and ruler (the mass
circle is inversely proportional to the force, R = k/F , where the factor ration of the brick and ball is provided).
k is a characteristic of the rod. 1) Determine the static coefficient of friction between the board and
the brick.
2) Determine the static coefficient of friction between the ball and the
F brick.
R

1) Let the rod be fixed vertically, at its bottom end, and a ball of mass m
be attached to its upper end. Knowing the factor k, the length of the rod
l, and the free fall acceleration g , find the period of small oscillations of
the ball. In this question, you may assume that gml  k.
2) What is the maximal mass M of the ball, which can be stably held on
such a vertical rod?
Remark: you may use approximate expressions sin x x x3 /6
and cos x 1 x2 /2 (for x  1).
3. Temporal focusing (10 pts)
Suppose that at the point A, there is a source of thermal electrons (of
negligible thermal energy), which are accelerated initially by the vol-
tage U0 = 36 V in horizontal direction (see figure). At the path of
the electrons, there are two voltage gaps B and C of negligible size,
at the distance a from each other. These gaps receive a voltage signal
UC (t) = UB (t) U (t) from a waveform generator. We can assu-
me that |U (t)|  U0 . The electrons starting at different moments of
5. Rotating disk (7 pts) Use the numerical value = 0,3. All the other friction forces acting 1) Describe the position and geometry of the diffraction pattern, which
upon the truck can be neglected. can be observed, when the laser beam falls onto the strip; use different
A lamp is attached to the edge of a disk, which moves (slides) rotating
4) How does the answer change, if the cross-section of the pole is not incidence angles.
on ice. The lamp emits light pulses: the duration of each pulse is negli-
circular, but instead, egg-shaped? Motivate your answer. 2) Provide a qualitative (approximate) explanation for the observed phe-
gible, the interval between two pulses is = 100 ms. The first pulse is
nomenon.
of orange light, the next one is blue, followed by red, green, yellow, and (a)
again orange (the process starts repeating periodically). The motion of (b) 3) Estimate the diameter of these glass spheres.
the disk is photographed using so long exposure time that exactly four T1 T
pulses are recorded on the photo (see figure). Due to the shortness of
the pulses and small size of the lamp, each pulse corresponds to a colo-
red dot on the photo. The colors of the dots are provided with lettering:
O orange, S blue, P red, R green, and K yellow). The friction
7. To the Mars (10 pts)
forces acting on the disk can be neglected.
1) Mark on the figure by numbers (14) the order of the pulses (dots). In this problem, we study a project for flying to Mars. At the first stage,
Motivate your answer. What can be said about the value of the exposure the space ship switches on the rocket engines and obtains an initial ve-
time? locity v0 . You may assume that during the first stage, the height of the
2) Using the provided figure, find the radius of the disk R, the velocity space ship (from the surface of the Earth) remains much smaller than
of the center of the disk v and the angular velocity (it is known that the radius of the Earth R0 = 6400 km. At the second stage, the space
< 60 rad/s). The scale of the figure is provided by the image of a line ship performs a ballistic motion in the gravity field of the Earth: upon
of length l = 10 cm; achieving a height, which is much larger than the radius of the Earth R0 ,
but much smaller than the orbital radius of the Earth, Re = 1.5108 km.
1) Find the relationship between the residual velocity v1 (with respect
to the Earth) at the end of the second stage, and the quantities v0 , R0 ,
and the free fall acceleration at the surface of the Earth g (for the sub-
sequent questions you may use the numeric value g 9.8 m/s2 ).
At the third stage, the space ship performs a ballistic motion in the
gravity field of the Sun, up to reaching an immediate neighborhood of
the Mars. The trajectory is chosen by minimizing the residual launch
velocity v1 (required for achieving the Mars).
2) Sketch the trajectory.
3) Find the flight time T . You may use the following numeric data: the
orbital velocity of the Earth ve = 30 km/s, orbital radius of the Mars
6. Truck (7 pts) Rm = 2.3 108 km.
4) Find the previously considered launch velocity v0 and the terminal
1) A rope is put over a pole so that the plane of the rope is perpendicular (i.e. at the end of the third stage) velocity vt of the space ship with res-
to the axes of the pole, and the length of that segment of the rope, which pect to the Mars.
touches the pole is l, much shorter than the radius of the pole R, see fi- 5) The required mass of the fuel M can be found from the formula
gure (a). To the one end of the rope, a force T is applied; the sliding of v = u ln[(M + m)/m], where u is the speed of the gas at the outlet
the rope can be prevented by applying a force T1 to the other end of the of the engine (with respect to the space ship), and m is the useful mass
rope. Express the ratio T1 /T via l, R, and , where is the coefficient of the ship (when all the fuel is exhausted). You may assume m  M
of friction between the rope and the pole. and use the numerical value u = 1 km/s. Find, how much more fuel
2) Answer the first question, if l is not small (i.e. without the assumption is needed for the Mars flight, as compared to simply escaping the Earth
l  R). gravity field, if the useful space ship mass is equal in both cases.
Remark: you may use equality
8. Laser (12 pts)
lim (1 + nx)1/x = en .
x0 Apparatus: Laser (wavelength = 650 nm), ruler, stand, a strip of ref-
3) The rope makes exactly n = 2 winds around the pole. One end of the lecting material, a sheet of paper with a circular hole (in your pack of pa-
rope is attached to a truck standing on a slope (slanting angle = 10 ); per sheets), pencil. Note that the reflecting material provided to you is
the mass of the truck m = 20 t, see figure (b). Find the force F , needed coated with a layer of densely packed tiny glass spheres of equal diame-
to apply to the other end of the rope, in order to keep the truck at rest. ter.
1. Wire (7 pts) 3) For the changing voltage, we can use the result of the previous ques- from where
1) Using the Ohms law in differential form, E = j , and noting that the tion, but the voltage value should be taken at the moment of electron m
+ 1 2 cot
M
passing the gap. Also, the terminal velocity is changed, v1 v0 is rela- = m .
electric field must be the same both in the core and in the coating, we 2 (cot + M )1
conclude jc c = ja a jc = a a /c . On the other hand, the net ted to the change of the kinetic energy e[U (t + ta ) U (t)]. The travel
current I = 4 [a2 jc + (b2 a2 )ja ] = 4 ja (b2 a2 + a2 a /c ), hence time of the later electrons passing B at a certain moment of time t > 0 Using the measured values we get 0.2.
4 I should as much shorter as it was delayed, i.e.
ja = 0.15A/mm2 ; jc 0.25A/mm2 .
r
b2 + a2 (a c )/c m
t1 t2 = {a[U (0) U (t)] + b[U (t + ta ) U (t)]} = t,
2) Writing down the circulation theorem for a circular concentric loop 8U03 e
of radius c around the straight wire, 2cB1 = 0 I , we obtain B1 = where the flight time ta a/v0 [since U  U0 ]. Using the suggested 5. Rotating disk (7 pts)
0 I/2c = 2 Ic 107 Hm1 = 4.8 105 T . approximation, we obtain
r
3) Using the technique as before, but noting that the current flowing m 1) We notice that there is no image of the orange pulse, hence it must
b[U (t + a/v0 ) U (t)]} = t.
through the smaller loop of radius a/2 is Ic = jc 4 a2 , we obtain 8U03 e have taken place immediately before the shutter release. So the blue
B2 = 0 Ic /a = 0 jc a/4 = jc a 107 Hm1 = 2.0 104 T . If we seek a quadratic solution U (t) = At2 + Bt, we get pulse is first, red the second etc. The exposure time must have been
triple and quadruple flash interval, 300 ms < t < 500 ms.
r
2. Pendulum (7 pts) t 8U03 e
2Ata (t + ta /2) + Bta = .
1) The smallness of the oscillations means that the angle of the arc for- b m 2) The displacement of the lamp between two subsequent pulses can be
med by the rod is small, = l/R  1. Therefore, the force returning So, r r p represented as the sum of two components: ~ri = ~v +2R sin( /2)~ei ,
the ball is almost horizontal, F = k/R = k/l, and the horizontal 8U03 e 8U03 e 2eU0 /m 4U02 e where each next unit vector ~ei+1 is rotated with respect to the previous
A= /2ta b = = ,
displacement of the ball is x = R(1 cos ) R2 /2 = l/2, hence m m ab mab one (~ei ) by angle . So, if the starting points of the displacement vec-
= 2x/l. and B = Ata . Finally we obtain tors ~ri coincide, then the end-points must be on a circle, at equal angu-
4U02 e
 
For horizontal equation of motion, we can neglect the gravity force, lar distances from each other, see figure.
p
U (t) = t t a m/2U0 e .
which is of the order of mg  k/l; the right-hand side here hap- mab
to be the elastic force. So, mx = k/l 2 4) Lost are those electrons, which are in the interval between B and C , In our case we redraw the displacement vectors 1, 2 and 3 as vectors
pens p p = 2kx/l , hence when the voltage drops to 0. The duration of the time interval, when with common origin, P~A, P~B , and P~C . Since the starting points of the
= 2k/m/l; the period T = 2/ = l 2m/k .
these werepemitted, is = a/v0 . So, the asked fraction is given by vectors 2R sin( /2)~ei are brought together to the point O, their end-
2) Elastic energy of the rod depends apparently only on its shape, i.e.
/T = Ta m/2U0 e. points lay on the circle, the center of which can be found as the center
on the curvature radius R, or, equivalently, on the binding R x angle . of the circle drawn around the triangle ABC .
First, we derive the expression for that energy: r = 0 F dx = 4. Coefficient of friction (12 pts) B
(2kx/l2 )dx = kx2 /l2 = k2 /4. This is to be compared with the
R
1) This is a straightforward question: we measure the tangent of the
change of the gravitational potential energy g = mg(l R sin ) = 2
slope of the board, when the brick starts sliding, 1 = tan 1 , for the
mgl(1 sin /) mgl2 /6. So, the vertical position is stable, if given setup, 1 0.24.
k2 /4 mgl2 /6 > 0, i.e. 3k > 2mgl. 2) We put the ball and brick together on the slope, the ball touching 3
Remark: a simple-minded force balance of the ball to the direction, both the brick and board and being upwards on the slope. So, the ball
perpendicular to the rod leads to another result k > mgl. This is not pushes the brick down and the sliding is expected to take place for so-
correct, because if we bind the rod with a force applied to its tip, small 2
mewhat smaller sloping angles. For the given setup, that critical angle
displacements of the tip are not perpendicular to the rod (as one might turns out to give tan 0.12. O
think). In particular, this means that if there are both perpendicular Now we need to express knowing 2 0.24 and = P
force F and tangential tension T in the rod, the curvature radius R will 1
arctan 0.12. First we write the torque balance for the ball with res- 1
depend also on the tension T . In our solution, we avoided such kind pect to the touching pint with the board: mgR sin = NR + NR,
of complications by noting that the elastic energy depends only on the A
where N is the pressure force between the ball and the brick. Hence, 3
shape of the rod (if we bind the rod with some force F while keeping mg L
N = +1 sin .
T = 0, we dont need to bother about this effect). Now, we use the normal (to the board) force balance for the brick, C
3. Temporal focusing (10 pts) to find the pressure force between the board and the brick: N2 =
p M g cos + mg +1 sin . Finally, we have the tangential force balance
1) Apparently, t0 = (a + b)/v0 = (a + b) m/2U0 e. The velocity of center of the disk is found as the ratio of the
for the brick:
2) Apparently, t = a/va + b/vb t0 [a(va v0 ) + b(vb v0 )]/v02 .   length P O and the interval : v 65 cm/s. The angular velocity
mg m
On the other hand, for constant voltage U (t), vb = v0 . For the interval sin + M g sin = 2 g M cos + sin . is found as the ratio of the angle 6 AOB = 6 BOC and the inter-
+1 +1
between B and C , we have (mv 2 /2) mv0 (va v0 ) = U e. So, val : 23 rad/s. Radius of the disk is found from the length
r This equation can be simplified to
U a m m m |OA| = 2R sin( /2) = 2R sin 6 BOC 1.5R; using the scale of
t = t0 + U ea/mv03 = t0 +
h i
. + + 1 = 2 ( + 1) cot + ,
U0 2 2U0 e M M the figure, 1.5R 8 cm and R 5 cm.
6. Truck (7 pts) the observed data. First, the diffraction pattern is not where the reflec-
1) First, since l is small, T1 T . From the radial force balance, N tion form the stripe would be; so, it doesnt work as a reflecting diffrac-
T = T l/R, where is the angle, by which the direction of the rope is tion grating. If the packing were regular, and the optical path difference
changed. From the tangential balance, T1 = T N = T (1 ). (required for the diffraction) were gathered due to scattering on neig-
2) If the angle is not small, we divide the touching segment into M hboring spheres, there would be a pattern, characteristic to diffraction
small segments and use the previous result: Ti+1 = Ti (1 /M ). So, on crystal lattices (regularly positioned spots). If the optical path diffe-
TM = T (1 /M )M T e (as M ). So, T 0 = T el/R . rence were gathered due to scattering on neighboring spheres, with ir-
3) Using the previous formula and the provided numerical values ( = regular packing, the path difference would be random, and hence, the
2 ), we obtain diffraction pattern would be also random. This would not explain the
T = M g sin e 800 N. regular circular pattern.
4) If we re-examine the solution to the second question, we notice that On the other hand, if the optical path difference were gathered on
we havent used the assumption of cylindrical shape. What matters, is a single sphere, the pattern would be axially symmetric, as is the sphere
just the rotation angle of the tangent to the rope. So, the answer does itself. So, this fits well with all the experimental observations (except
not change for egg-like cross-section. for slight elongation of the circles for large incidence angles, which may,
perhaps, be explained by the refraction in the coating layer).
7. To the Mars (10 pts) On a single sphere, the light is reflected partially back on the front
1) Apparently v12 = v02 2gR0 . surface, and partially refracts into the sphere, reflects internally and ref-
2) The trajectory is ellipse touching Earth orbit at its perihelion and racts back into the air (in our rough analysis, we neglect multiple partial
Mars orbit at its apohelion.. reflections). For beams falling close to the axes of the sphere, the out-
3) According to the Keplers third law, the time T = 21 T0 (a/Re )3/2 , going beams are reflected almost in the opposite direction. So, in its
where T0 = 1 year is the period of Earth and a = (Re + Rm )/2 is the central part, the sphere acts almost like a reflecting disk.
longer semi-axes of the trajectory. Numerically, T = 0, 70 years. In our very rough model, we substitute the spheres by disks of app-
4) The full energy (kinetic plus potential) of the space ship is E = roximately the same diameter as the spheres, oriented perpendicularly
Gm/2a = Gm/(Re + Rm ) = Gm/Re + m 2
(v1 + ve )2 . So, to the incident beam. Then, all the microspheres provide maxima and
1 2 Rm 2 Rm minima in the same directions.
2 (v1 + ve ) = G Re (Re +Rm ) = ve Re +Rm and
r ! 3) We measure the diameter of the first dark ring d 1.1 cm, and the
2Rm distance between the laser and the strip l 80 cm. Then, the diameter
v1 = ve 1 0.095ve 2.86 km/s.
Re + Rm of the spheres can be estimated as the diameter of the effective light-
reflecting disk, D l/d 50 m.
Hence, v0 = 11.22 + 2.862 = 11.6 km/s.
The relative speed near the Mars can be written using the approp-
riate change of indices in the expression for v1 (alternatively, it can be
found analogously to v1 ):
we first find the speed with respect to Sun from the expression of
the total energy: v3 =
r ! r
2Re Re
v3 = vm 1 0.106ve 2.59 km/s.
Re + Rm Rm
5) Apparently Mi = mevi /u , so that Ma /Mb = e(va vb )/u = e0.4
1.5.
8. Laser (12 pts)
1) These are concentric circles reflected back towards the laser. They
can be seen, if laser light is directed through the hole in the paper and
the paper is used as the screen. The position and size of the circles is in-
dependent of the incidence angle (for large incidence angles, the circles
will be somewhat elongated along the direction of slanting).
2) The exact calculation of the diffraction pattern on such microspheres
is very difficult task, evidently beyond the possibilities of an olympiad
problem. So, we have to work as a detective, drawing conclusions from
ESTONIAN FINNISH PHYSICS OLYMPIAD
PROBLEMS & SOLUTIONS (2008)
Eestonian-Finnish Olympiad - 2008
(a)
1. Dumbbell (6 points)

Two perfectly elastic identic balls of mass m are connected by


a spring of stiness k so that a dumbbell-like system is formed.
That dumbbell lays at rest on a slippery horizontal surface (all
the friction forces can be neglected). Third ball (identic to the (b)
ones making up the dumbbell) approaches coaxially the dumbbell
from the left side with velocity v (see Figure). Fourth ball (identic
to the other ones) lays coaxially rightwards to the dumbbell. Ti nano-disk heater thermal
temperature sensor bridge
1) Find the velocity of the centre of mass of the dumbbell after
being hit by the ball approaching from the left. 1) Find the thermal resistance R between the microcalorimeter
and the wafer (i.e. the ratio of the temperature dierence and
2) For which distances L between the dumbbell and the right- heat ux).
most ball, the nal velocity of the latter will be exactly the same, 4. Magnetic eld (6 points)
Magnetic eld with inductance B (parallel to the z-axis) lls the
as the initial velocity v of the leftmost ball? For questions (ii) and (iii), use quantity R, without substi-
tuting it via the answer of question (i). region x + y < R . Let us consider an electron of velocity
2 2 2

v L v = RBe/m (where e is its charge and m  its mass).


2) Write down the heat balance equation for the microcalorimeter 1) Sketch the trajectory of the particle, if initially, it moves along
and nd the temperature of the microcalorimeter as a function of the line y = 0 towards the region lled with magnetic eld.
time T (t) [you may seek it in the form T = T + T sin(t + )]. 2) How long time does the electron spend inside the magnetic
eld?
0

3) In order to study the thermal properties of the Ti-nanodisks,


3) Now, let us consider the situation, when initially the electron
the amplitude of the sinusoidal oscillations of T (t) should change moves along the line y = a (a < R). Find the angle , by which
by as large as possible value, as a response to a small change of the electron is inclined after passing through magnetic eld.
C (which is caused by the Ti-disks). Find the optimal circular
2. Microcalorimeter (9 points)

frequency . 5. Ball
A microcalorimeter is a thin circular silicon nitride membrane, 4) We have assumed that the temperature prole along the
0 (9 points)

thermally isolated from the surroundings, except that it is bridges is linear, i.e. their heat capacity can be neglected. For ruler, a glass ball, sheet of paper, marker. Find the
coecient of friction between the glass ball and the ruler. Estim-
Equipment:

thermally connected to the wafer by four thin and narrow thermal high frequencies & , this is not the case. Estimate the crit- ate the uncertainty of you result.
bridges (see Figure). The microcalorimeter is equipped with a ical frequency in terms of , l, specic heat c and density of
c

small heater in the middle of the membrane and a similar struc- the bridge material. c

ture on the edge of the membrane working as a thermometer.


This micro calorimeter is used to study the thermal properties of
nanoscale Ti disks (light tiny dots in Fig). The thermal power of
the heater depends sinusoidally on time, P = P cos(t) (negat-
ive power implies a withdrawal of heat). The circular frequency 3. Tractor
0

is suciently low, so that for any moment of time t, the temper- (6 points)

ature of the microcalorimeter T (t) can be considered constant Provided sketches (a) and (b) are made on the basis of satel-
across its entire surface, and the temperature prole along the lite images, preserving proportions. They represent tractors, to-
thermal bridges can be considered linear. The wafer, to which gether their smoke trails. The tractors were moving along the
the bridges are connected, is large and thick enough, so that its roads in the direction indicated by the arrows. The velocity of
temperature T can be considered to be constant all the time. the tractors was v = 30 km/h. For sketch (a), the direction
Each of the four bridges have length L and cross sectional area of all
0

S ; the thermal conductance of them is . Thermal conductance of wind is indicated by another arrow.
0

is dened as the heat ux (measured in Watts) per surface area, 1) Using the provided sketch, nd the wind speed for case (a).
assuming that the temperature drop is 1 C per 1 m. The heat

capacity of the microcalorimeter (with Ti-disks) is C . 2) Using the provided sketch, nd the wind speed for case (b).
6. Rectier (8 points) 9. Asteroid (7 points)

A voltage rectier is made according to the circuit depicted in It is believed that the impacts of the Earth with asteroids have
Figure. The load R = 10 k is fed with DC, equal to I = 2 mA. played an important role in the history of Earth. In this prob-
In what follows we approximate the U-I characteristic of the di- lem, you are required to study such an impact. As an ex-
ode with the curve depicted in Figure. The relative variation of ample, let us use the orbital data of the Apollo asteroid. Its
the current at the load has to satisfy the condition I/I < 1%. perihelion is 0.65 AU, i.e. its closest distance from the Sun is
r = R, where = 0.65 and R denotes the radius of the
Earth's orbit; its aphelion is 2.3 AU, i.e. its farthest distance
1

from the Sun is r = R, where = 2.3. In your calcula-


tions, you may use the orbital velocity of Earth, v = 30 km/h,
2

Earth radius R = 6400 km, and the free fall acceleration at the
0

Earth's surface, g = 9.8 m/s . You may also use the formula
0
2

E = GmM/2a, expressing the total energy of a body of mass


m, which moves along an elliptic orbit of longer semiaxis a, in
1) Find the average power dissipation at the diode at the working the gravity eld of a much heavier body of mass M . You may
regime of such a circuit. assume that the orbital planes of the Earth and of the asteroid
2) Determine the amplitude of the AC voltage (with frequency coincide, and that they rotate in the same direction around the
= 50 Hz), which has to be applied at the input of the circuit. Sun.
3) Find the required capacitance C . 1) Find the velocity v of the asteroid in the vicinity of the Earth,
4) Find the average power dissipation at the diode during the in the Sun's system of reference, neglecting the eect of Earth's
rst period (of AC input voltage) immediately following the ap- attraction.
plication of AC voltage to the input of the circuit. 2) Find the radial and tangential components v and v of that
velocity (i.e. the components, respectively parallel and perpen-
r t

7. Fire dicular to the vector, drawn from the centre of the Sun to current
(6 points)

There is wet wood burning in a replace on the ground. Seven position of the asteroid).
3) Find the same velocity components u and u in the Earth's
meters above ground, the smoke is at a temperature of t = 40 C.
system of reference.
r t

Disregard the exchange of heat with the surrounding air and as- 4) Find the velocity of the asteroid w, immediately before enter-
7

sume that the atmospheric pressure at the ground is constant in ing the Earth's atmosphere (at the height h = 100 km from the
time and equal to p = 1000 hPa; the air temperature t = 20 C
Earth's surface).
is independent of height . Assume that the smoke represents an
0 0
1

ideal gas of a molar mass = 29 g/mol (i.e. equal to the molar 10. Glass plate (10 points)
mass of the air), and of a molar specic heat at constant volume Laser ( = 650 nm), thin glass plate, lens, ruler.
C = 2.5R; universal gas constant R = 8.31 J/kg K. How high NB! glass plate is xed to a stand; avoid touching the glass itself
Equipment:

will the smoke column rise? (because its edges are sharp, and because it can break easily).
V

Find the thickness of the plate and estimate the uncertainty


8. Electron (5 points) of the result. Draw the scheme of your experimental setup.
Electron rests at the origin. At the moment of time t = 0, an
electric eld is switched in: its modulus is constant and equal to
E , but its direction rotates with a constant circular velocity
in the x y plane. At the moment t = 0, it is directed along the
0

x-axes.
1) Find the average velocity of the electron over a long time
interval for t > 0.
2) Sketch the trajectory of the electron and calculate the geo-
metrical characteristics of it.
1 Actually, during day time, this is not the case: air temperature decreases with height. However, during evening and night, due to heat radiation, the lower layers of air cool more rapidly than upper layers, and it

may easily happen that the temperature is roughly independent of height.


1. Dumbbell (6 points) The heat contained in the bridge must be comparable with the 2) Circular part of the trajectory is a quarter of the full circle,
4)
1) During the rst collision, we can neglect the eect of the heat, which ows through it during one half-period (if it is much so t = R/2v.
spring, because during the collision time, the balls almost don't smaller, the stationary linear prole will develop very soon). So, 3) Let O be the centre of the circular orbit of the electron and B

move, hence the spring doesn't deform. Two absolutely elastic AcSL AS/(L ); hence,  the intersection point of the trajectory with the region bound-
ary. The polygon COBO is rhomb, because all the sides are
identic balls exchange velocity during a central collision. So, the
c

/cL . 2

rst ball will remain at rest, and the second one will obtain the
c
equal to R. So, the line BO is vertical (because O C is vertical

velocity v. So, the velocity of the centre of mass of the dumbbell 3. Tractor and BO is parallel to it). Hence, the inclination angle of the
is v/2. (6 points)
electron is
2) After the impact, the dumbbell will oscillate in the sys-
a
= CO B = AOB + AOC = arcsin .
tem of reference of its centre of mass with circular frequency 2 R
= 2k/m (balls oscillate so that the middlepoint of the spring 5. Ball (9 points)

is at rest; twice shorter spring has a twice larger stiness).


Due to the energy conservation law, the only way for the
fourth ball to acquire the velocity v is such that all the other
balls remain at a complete rest after the interaction. Therefore,
before the impact of the third and fourth balls, the third ball must
have velocity v (and the second ball must be at rest). This is the
opposite phase of the moment, when the dumbbell started its
motion. Hence, the travel time t of the dumbbell must be a half-
integer multiple of the period T = 2m/2k. For that phase
of oscillation, the spring is, again, undeformed, i.e. the travel We lay one of the rulers horizontally on the table. Then, we put
distance of the centre of mass is also L. So, 2L/v = T (n + ), 1
the ball on that ruler, and the other ruler laying on the ball. With
hence ( )
1
2

nger, we keep one end of the second ruler in contact with the
L = v n + m/2k. rst ruler and nd the closest stable position of the ball (resulting
2
1) Let us draw from an arbitrary point B on the road a line par-
in the largest inclination angle of the second ruler), see Figure.
2. Microcalorimeter (9 points)
allel to the direction of the wind, and let it intersect the smoke Now, we consider the torque balance (for the ball) with respect to
1) Every bridge has thermal resistance L/S ; so, the overall res-
trail at point C . Then, the smoke emitted by the tractor at B because and
the ball ruler touching point B. Gravity force has no torque,
istance is R = L/4S. has travelled the distance |BC| = ut, where u is the wind speed. vector of the applied
it is to the centre of the ball O. So, the resultant
2) The power dissipation P results in an heat ux through the
bridges, = T /R, and in the change of the heat contained in measureTractor itself has travelled the distance |AB| = v t. So, we can torque, i.e. it has to goandthrough
friction reaction forces at C must have also zero
B . At the threshold of sliding, the
the microcalorimeter, Q = C T = CT (here, dot denotes the late the distances |AB| and |BC from the gure and calcu- angle between this vector and the surface normal CO is arctan .
0

time derivative). So, |BC| So, = tan BCO = tan OAB = |OB|/|AB| = R/|AB|. The
radius of the ball R 40 mm can be measured by rolling the ball
18 mm
u=v = 30 km/h 13 km/h.
0
|AC|
Now, we can search the solution as T = A cos(t + ), and 2) If the second tractor (at the right-hand-side) had started some- on the ruler by angle 2. The distance |AB| can be measured
P cos(t) = CT + T /R.
0 42 mm

denote = arcsin(C/ C + R ). Then, 2 what earlier, the two tractors had been at the crossroad simul- directly using the ruler. Several measurements are needed, to
taneously. Now, the tractors would be at the same distance from nd the critical position of the ball more accurately.
2 2

P cos(t) = A C + R cos(t + ).
the crossroad, i.e. for the current position of the second tractor We have used one ruler as the basis, because if the surface has
2 2 2

So, we must have = ( and A = P / C + R , i.e.)


0

C , |OC| = |AO| = v t (this is how we nd the point C ). Its smaller coecient of friction than the ruler, the sliding starts at
2 2 2
0

smoke trail can be found as a line, parallel to its smoke trail at the point B, hence we are not able to obtain the required result.
P cos t + arcsin(C/ C + R ) 2 2 2 0
0
T =T + .
its actual position B. Such a meeting of the tractors would have 6. Rectier
0
2

3) The amplitude of the oscillations is A = ; it been resulted


C +R 2 2

must be as sensitive with respect to the small changes of C , i.e. now in positionin Dthe, with
crossing of the smoke trails. which would be 1) Since none of the DC current through the load can come from
P0
(8 points)
C 2 2 +R2

OD = ut. So, we nd


dA/dC must be maximal by modulus. dA/dC = P (C + 2 2
the capacitor, all must come through the diode. Hence, the aver-
C ; if we denote x = (C) , we need to minimize the age current through the diode is also I = 2 mA, and the average
0 |OD| 27 mm
2 3/2 2 2 u=v = 30 km/h 21 km/h.
following function of :
R )
power dissipation is obtained by multiplying it with the diode
0
|AO| 39 mm
x
[ ]
ln (dC/dA) = 3 ln(x + R ) ln x + ln C.
2 2 4. Magnetic eld (6 points) voltage u = 1 V: P = 2 mW.
Upon taking derivative and putting it equal to 0, we obtain 1) Since the radius of the cyclotron orbit is equal to the radius 2) If the diode is open, Uload(t) = U cos(t) u. If the di-
3x = x + R , from where x = R /2, i.e. of the region R, the trajectory is given by the curve DABE in ode is closed [i.e. U cos(t) < Uload(t) + u], the capacitor dis-
0
2 2

the Figure (AB is a circle fragment). charges through the load. However, the relative change of the
0
= 1/ 2CR.
voltage of the capacitor has to be small (otherwise I/I would We can integrate these equations over time (bearing in mind that 10. Glass plate (10 points)
not be small). The respective load voltage as a function of time initial velocity is zero):
is sketched in the Figure. So, we can use the above written 1
Kircho's law with Uload(t) IR, hence U = IR + u = 21 V.
0
mx = eE sin t,
my = eE (1 cos t). 1
0

Now, we can integrate once more, bearing in mind that the initial
0

coordinates are zero:


eE0
x= (cos t 1),
m 2
There are two possible setups. First,
we consider the interference of the
eE0 eE0
y= cos t + t.

The change of the voltage of the capacitor during the discharge So, the electron performs circular motion in the system of ref-
m 2 m
beams, reected from the upper and
3)
cycle can be estimated as U = Q/C , where the capacitor's erence, moving with velocity (parallel to the y-axis) u = . eE0
lower surfaces of the glass plate, see
charge drop Q = It, and t is the discharge time. Since the dis- The radius of the orbit is R = . In the laboratory system,
eE0
m
Figure, upper drawing. Second, we
charge cycle occupies almost all the period (see Figure), we can this is a cycloid (the curve drawn by a point on the edge of
2 m
direct the beam on the edge of the
use t 1/ . Further, I/I = U/U = Q/CU = Q/CIR = a rolling disk); the distance between the neighbouring loops is plate. As a result, on the screen, there
= 2u/ = 2R. will be almost the same diraction
1/CR . Hence, C 100/R = 200 F.
4) Initially, the capacitor is empty, so that the charge owing pattern, as from a single slit (lower
through the capacitor during the rst cycle is Q = CIR. Hence, 9. Asteroid (7 points)
drawing in the Figure).
the average power P = Qu = CIRu = 200 mW. 1) The longer semiaxis is a = (r + r ) = ( + )R. So, the
1 1
In the rst case, we need to calculate the optical path dif-
full energy of the asteroid at the Earth's location is ference, see Figure. l = 2(n|CD| |AB|) = 2(nd/ cos
1 2 1 2 2

7. Fire (6 points) d sin sin ) = 2d(n/ cos sin /n); we keep in mind that
2

sin = sin /n. We need to nd such a change in ,


E M 1 M
The smoke will rise until its density becomes equal to the density = = v2 .
which gives rise to the change of l by (this corresponds
of the air at the same height. Since the molar masses and pres-
m 2a 2 R
Bearing in mind that v02 = M , we can rewrite this as to a transition from one diraction minimum to another one):
sures of the smoke and air are equal, this implies also equal tem- R

= . Then, we can relate the measured quant-


peratures (pressures are equal, because otherwise, there would be v02
= v02
1 2 d(l)

ity, the distance between the minima on the screen a = L


no mechanical equilibrium). Temperature of the smoke will drop
v . d
+ 2
with increasing height due to adiabatic expansion. If we com- So, we obtain (where L is the path length |AB| + |BC|) to the plate thick-
bine the law of the adiabatic process pV = Const with the ideal
ness. d(l) 2 2
= 2d(sin / cos sin 2/n) = 2d sin (cos
2 cos /n). So, L = 2ad sin (cos 2 cos /n); hence,

2[1 ( + )1 ] 34.5 km/h.
gas law (pV /T ) = Const, we obtain p /T = Const. Tak-
v = v0 d

Tangential component can be found from the angular mo-


2

d = L/2a sin (cos 2 cos /n). We can easily measure


1

ing a logarithm and dierential from this equation, we obtain 2)


mentum conservation law: v = v , where the velocity at the
2

and calculate ; for n, we can use typical value n 1.4, or use


= 0, hence we can use approximate expression
perihelion canbe found analogously to v:
r p
( 1)
the Brewster angle measurement to nd n = tan . For the
dp dT

for the require temperature change


p T

( )
precise measurement of a, we count several, e.g. 10, inter-minima
B B

20 K = T = T
1 p
. vp = v0 2
1

1
= v0
2
37.5 km/h. intervals, and divide the distance between the farthest minima by
We can use , where
p
is the air So,
+ +
10.
p = gh = p/RT 1.2 kg/m3
density. Also, we can substitute . So, In the second case, the angular distance between the minima
we obtain
= cp /cV = (cV + R)/cV 2 is given by = 2/d, so that a = 2L/d and d = 2L/a.
T =
R gh
,
v =v
Radial component
t
+
24.4 km/h.
0
Numerically, the thickness was d 0.20 mm.
hence (
cV + R R
cV ) T R
vr = v 2 vt2 24.4 km/h.
h= 1+ 2040 m. 3) The required components can be found by subtracting the
R g
Earth's orbital velocity. Apparently u = v , and r r

8. Electron (5 points) ut = vt v0 5.6 km/h.


Let us write the Newton's II law for x- and y-components of the 4) When the asteroid approaches Earth's surface along the para-
electrons coordinates: bolic orbit, the energy due to the Earth gravity force gR is added
to its kinetic energyin the Earth's system of reference:
0

mx = eE cos t,
0

my = eE0 sin t. w= u2t + u2r + 2gR0 27.4 km/h


ESTONIAN FINNISH PHYSICS OLYMPIAD
PROBLEMS & SOLUTIONS (2009)
Estonian-Finnish Olympiad - 2009 i) Find the moment of time t, when the ball stops sliding, i.e. starts
1. Boat (9 points) Consider a boat of effective mass m, which rolling without sliding.
moves along the (horizontal) x-axes, and for which the friction ii) Calculate the angular velocity of the ball and its total
force of the water is given by Ff = v , where v is the velocity mechanical energy E at the moment when it stops sliding. In the
of the boat. (Effective mass is somewhat larger than the real mass, case of a hollow sphere, would the energy E be larger or smaller
because any change of the boats velocity incurs a velocity change than in the case of a homogeneous ball?
of those regions of water, which are close to the boat; hence, the iii) Now, assume that the horizontal surface is treated so that
boats inertia is complemented by the inertia of the surrounding the coefficient of kinetic friction depends on the horizontal
water.) coordinate x as = a + b cos x (with a > b). Find the
i) Prove that during this motion, expression for the terminal mechanical energy E in this new case.
v + kx = Const, iv) Now, let us return to the case of constant kinetic friction
coefficient . However, let us assume that the surface is not
and find the factor k .
perfectly rigid (e.g. covered by a felt cloth). This gives rise to a
ii) Now, a boy of mass M moves inside the boat, back and forth,
second friction force the rolling friction force Fr = r mg .
with velocity u(t) (also, along the x-axis). What term must be
Unlike the kinetic friction force, it is not tangential to the touching
added to the conservation law v + kx = Const, to describe this
point of the ball and the surface. Instead, it can be interpreted as
situation?
the horizontal component of the surface reaction force (the entire
iii) Now, let the boat rest initially close to the coast and ready to
reaction force is, of course, normal to the surface), see Fig. Find
depart. The boy jumps from the coast into the boat, turns around
the expression for the terminal mechanical energy E in this case.
and jumps back to the coast. The horizontal component of the
What is the most important (qualitative) difference between the
boys velocity during the first jump was u1 , and during the second
expressions for E and E ?
jump u2 . What is the maximum distance s travelled by the boat?
Neglect the effect of a vertical motion on the horizontal friction
force of the water.
2. Nanoclock (10 points)
Nanotechnology allows controllable fabrication of very small
structures. Let us consider a tiny homogeneously charged thin
ring, having a radius R and carrying a positive net charge Q.
i) Find the electric potential at a point P , which is on the axes Hint: The moment of inertia of a ball is I = 25 M R2 .
of the ring, at a distance z from the centre of the ring. 4. Black box (9 points) Equipment: electrical black box with three
ii) Find the electric field E at the point P . outlets, battery, voltmeter.
iii) Show that the force acting on an electron moving along the It is known that inside the black box, there are three resistors
symmetry axes in the vicinity of the centre of the ring (|z| R) (connected with wires in an unknown manner), the smallest of
is harmonic (i.e. depends linearly on z ). which is R1 = 100 . Find the value of the largest resistor R3 .
iv) Define the frequency of oscillations of such an electron. Use What can be said about the middle-valued resistor R2 ? Estimate
numerical values R = 1m and Q = 1.0 1013 C. the uncertainties of your results.
v) Now, let us assume that the electron can perform also off-axis
movements. Is the position at the symmetry centre of the ring (on-
axis, z = 0) stable or unstable? Motivate your answer.
Hint: You may use the approximate formula
(1 + x) 1 + x + 12 ( 1)x2 .
3. Ball (8 points) A homogeneous ball of radius R and mass m is
thrown horizontally onto a table at the moment of time t = 0. Its
initial velocity before contact is purely horizontal and equal to v ;
it is non-rotating. Coefficient of kinetic friction between the table
and the ball is .
5. Pencil (6 points) Equipment: pencil, paper, ruler. i) Find the current through the resistors R1 and R2 at the moment
Determine the coefficient of friction of the pencils graphite of time t1 = 5 ms.
core against the paper. Estimate the uncertainty. ii) Find the current through the resistors R1 and R2 at the
6. Spring (7 points) Equipment: helical spring of known mass moment of time t2 = 15 ms.
m = 19 0.5 g, measuring tape, a load of unknown mass. iii) What is the net charge passing through the resistor R2 ?
Determine the mass of the load. Estimate the uncertainty. 9. Stratostat (5 points)
7. Soap film (6 points) Lord Rayleigh had in 1891 a lecture about i) Show that the pressure of an isothermal gas of molar mass
taking photos of physical processes. Among others, he showed a follows the law p = p0 ez , where p0 is the pressure at the origin,
photo of a soap film, which is falling apart (see Fig.). Instead of aand z is the height. Find the constant . The temperature is T , the
flash, he used an electric spark (well, nowadays the flashes are alsofree fall acceleration is g .
based on electric sparks). Estimate, how precise must have been ii) Consider a stratostat, the envelope of which (a freely
the timing, i.e. estimate the time for a soap film to fall apart. Letdeformable non-elastic sack) is filled at the Earths surface by
the thickness of the soap film be h = 1 m, the ring diameter helium to the volume fraction of = 10%. At which height h
D = 10 cm and the surface tension = 0.025 N/m. does the helium expand so that it fills the entire volume of the
stratostat? The molar masses of the air and helium are a =
29 g/mol and He = 4 g/mol, respectively. You may neglect
the temperature variations of the atmosphere, and use the value
T = 250 K.
10. Wedge (5 points) A wedge of mass M is kept at rest on an
horizontal surface, and a block of mass m is kept on the wedge at
the height h from the surface. The angle of the wedge is , see
Fig. There is no friction neither between the block and wedge nor
Hint: you may use a model, according to which the already between the surface and the wedge. The system is released into
broken part of the soap film gathers into a single front and moves a free motion. Find the time t needed for the block to reach the
all together towards the still preserved part of the film. surface.
8. Magnetic pulse (7 points) Consider an electric circuit
consisting of a coil of negligibly small inductance, consisting of
N = 10 turns and with the surface area of a single loop S =
10 cm2 ), resistors R1 = R2 = 3 , capacitor C = 0.2 F,
and an inductance L = 1H , connected as shown in Fig. At the
moment of time t = 0, a magnetic field, parallel to the axis of
the coil is switched on. The inductance of the magnetic field starts
growing linearly, starting from B = 0 until the maximal value
B = 1 T is achieved at t = 10 ms. Further, the inductance of
the magnetic field remains constant (and equal to 1T).
Estonian-Finnish Olympiad - 2009 field E = kQz/R3 ). Hence there must be field lines entering triangular connection,
1. Boat (9 points) the cylinder through its side surface. This implies a radial repelling R3 V12
force for an electron situated at that surface, i.e. instability. = ,
i) From the Newton II law, m dv dt
+ dx
dt
= 0. Multiplying this R1 V23
equation by dt, we obtain dv + m
dx = 0. Integrating (i.e. Remark: In the same way, one can prove a theorem, where Ri denotes the resistor farthest away from the i-th outlet;
summing over all the small increments dx and dv ) this equation electrostatic equlibria are always unstable. for a triangular connection,
leads us to 3. Ball (8 points) A straightforward way to solve parts i), ii), In such a way we find
and iii) is to use conservation of angular momentum with respect
v + x = Const, R3 R3 R2
m to any axis laying on the surface (there is no torque with respect 4,7, 3,3, and 1,4,
i.e. k = m
. R1 R2 R1
to these axes). Angular momentum at the beginning: L0 = implying R1 = 100 , R2 = 140 and R3 = 470 ; or
ii) We proceed in the same way as before, but we need to add the
mvr . Rolling ball is always rotating around the touching point
interaction force between the boat and the boy: m dv + dx = R1 R2 R1
du
dt dt with the ground (although the location of that point is constantly 4,7, 3,3, and 1,4,
M dt . [Note that since the right-hand-side of this equation is changing). From Steiners theorem we know that for a rotation R3 R3 R2
the interaction force, it goes to zero, if the boy leaves the boat. axis that is located at the distance r form the center of mass, the R3 = 100 , R2 = 330 and R1 = 470 . So, we can
Therefore, if we want to keep this equation correct even after the moment of inertia is I = I + mr 2 . Therefore the angular conclude that the largest resistance is 470 , and the middle-
boy leaves the boat at the moment of time t = t , we must assume momentum for rotating ball is L = I and clearly L0 = L , valued resistance is either 330 or 140 .
u(t) u(t ) for t > t .] Similarly to the previous section, we therefore mvr = (I + mr 2 ), = I+mr mvr
2.
obtain
M i) The torque with respect to center of mass: M = mgr . Since
v+ u(t) + x = Const. M t = I , and the sliding stops when angular speed has
m m I
iii) We use the conservation law of the previous section, and reached value , we obtain the corresponding time: t = mgr =
compare the value of the left-hand-side immediately before the Iv
.
g(I+mr 2 )
boy lands into the boat with its value after a very long time. mvr
Bearing in mind that we need to substitute u(t ) = u2 ii) Clearly, = I+mr 2
and E = 12 (mv2 + I2 ) =
2 2 2
1 m v r
(see above), we obtain 2
(mr 2 2 + I2 )= 2(I+mr 2 ) . If I increases then E decreases.
M M iii) Since we didnt make any assumptions about the functional
0+ u1 + 0 = 0 u2 + s.
m m m form of while deriving w , the results w and E of ii) are still
So, valid.
M
s= (u1 + u2 ), iv) The net force F = F +Fr causes the decrease of translational
velocity: mv = F t, v (t) = v ( + r )gt. The sliding
i.e. the result is independent of how long time did the boy spend
in the boat. stops when (t) = v (t)/r . Note that the torque with respect to
2. Nanoclock (10 points) center of mass is only caused by kinetic frictional force, therefore
i) All thecharges of the ring are at the same distance from the point (t) = mgr I
t. We get an equation for the time of the termination
mgr 2 Iv
P , l = R2 + z 2 . So, according to the superposition principle, of sliding: I t = v ( + r )gt, t = g(I+mr 2 )+g I .
r
mvr
the potential is the sum of potential of all the charges, The corresponding angular speed = I+mr2 + r I , and energy

= kQ/ R2 + z 2 . E = 12 (I + mr 2 )2 . Clearly, E < E .


ii) E = d dz
= kQz/(R2 + z 2 )3/2 . 4. Black box (9 points) Independetly of whether there is a
iii) For |z| R we may approximate (R2 + z 2 )3/2 R3 , so triangular or a star connection, one can measure the ratio of the
that E = kQz/R3 . resistances a pair of resistors by connecting the battery to two
3 2 outlets let these be the outlets 1 and 3, and measuring the
iv) From the Newton II law, q mz = ekQz/R , i.e. = voltages between the outlets 1 and 2, and between the outlets 2
1 ekQ
ekQ/mR3 and f = 2R mR
5.6 1012 Hz. and 3. For a star connection,
v) At the origin, there are no charges; hence, the electric field
R3 V23
lines cannot neither start nor end there. Consider a tiny coaxial = ,
cylinder embracing the origin. The field lines exit the cylinder R1 V12
through its bottom and top surfaces (because there is an electric where Ri denotes the resistor closest to the i-th outlet; for a
m
5. Pencil (6 points) We put the pencil with its graphite end against RC and LC cirquits are 1 = R1 C = 0.6 s and 2 = L/R2 horisontal velocity v = vh + u = vh (1 + M ). Since the
a horizontal sheet of paper, and push the other end with a fingertip 0.3 s. So, for both cirquits, the processes are very fast, i.e. the block is sliding down the wedge, the vertical component of the
(without holding between the fingers and thereby possibly giving capacitor is effectively short-circuited, and almost all the voltage block has to be v = v tan . The total kinetic energy in the lab
a torque to it). Then, there will be a purely longitudinal stress falls on the inductance. frame EK = 12 (M u2 + mvh2 + mv2 ). Substituting the relevant
inside the pencil (there is no bending of the pencil). If we push i) According to the considerations given above, I1 = U/R1 quantities and simplifying, we obtain
strongly enough, we may neglect the weight of the pancil in the 0.33 A. As for I2 , it starts growing from 0 A at t = 0 at a rate, m M
balance of torque with respect to the fingertip. So, equilibrium given by L dI = U , i.e. I2 = U t1 /L = 5 mA. Ek = v2 , where cot2 + 1.
dt 2 M +m
implies that the resultant force of the friction and reaction forces at ii) When the voltage U is switched off (at t = = 10 ms), The kinetic energy equals to the change in potential energy:
the graphite end are along the axis of the pencil. This is possible, if the capacitor will (almost completely, because t2 R1 C ) m
v 2
2 = EP = mg(h h ). By differentiating and noting
tan , where is the angle between the pencil and a vertical retain the charge it has accumulated, Q = I1 . All the voltage that dh = dt v , we get dv = gdt, t = g vmax . From the
line. So, we increase wile pushing the pencil, and determine of the capacitor (Q/C ) will fall on the resistor R1 , so that I1 = q
the position 0 , when thepencil starts sliding; tan 0 can be Q/R1 C = U /R12 C 5.6 mA. As for the inductance, it will above written energy conservation law we get vmax = 2gh
; so
calculated as tan 0 = x/ l2 x2 , where l is the length of the retain (almost completely, because t2 L/R2 ) the current q r  
pencil, and x is the length of the projection of the pencil onto a it has acquired during the first 10 ms, I2 = U /L = 10 mA. we finally have t = 2h g
= 2h g
M
M+m
cot2 + 1 .
horizontal plane. iii) Since the current in R2 will decay very slowly, as compared
6. Spring (7 points) First, we measure the lengthening of the to its growth during the first 10 ms, we can neglect the charge
spring under its own weight. The relative lengthening of each loop passing thorugh it during t < . Then we can write the Kirhoff s
is propotional to the order number of the loop. So, the average law in the form L dI +R2 dq = 0, from where LdI +R2 dq = 0,
dt dt
value of those legth increments is half of the largest increment (for and LI = R2 q . Since I = I2 , we obtain q =
the topmost loop). The topmost loop is deformed by the weight LI2 /R2 = 3.3 mC.
of the whole spring, hence the average deformation corresponds 9. Stratostat (5 points)
to the half-weight of the spring,
i) Consider the pressure difference at heights z + dz and z : dp =
kx1 = mg/2. gdz (the difference is simply due to the weight of the layer
Now we add a load to the lower end of the spring and measure dz ). The density can be found using the state equation of ideal
the new deformation x2 . Since the additional weight of the load gas: pV = m RT, = m = RT p
. Therefore p1 dp g
= RT .
V dz
increases the stress of all the loops by the same value M g , each
loop is deformed additionally by the same length increment. The Note that we have a derivative of a logarithm: p1 dp
dz
= d lndzp(z) .
g
sum of those additional increments is x2 x1 ; according to the Therefore ln(p/p0 ) = z, p = p0 ez , where = RT .
Hooks law, k(x2 x1 ) = M g . So, xx12 1 = 2M m
, and Alternatively, we can derive this law from the Boltzmann
m x2 x1 distribution for the particle density n = n0 eU/kT , where U
M= 6 g. is the potential energy of a molecule. Bearing in mind that for a
2 x1
7. Soap film (6 points) Suppose the area of a broken part of the constant temperature, the pressure is propotional to the density,
soap film is S . The corresponding surface energy is ES = 2S . we obtain p = p0 eU/kT . Substituting U = mgz = NA gz and
The mass of the soap water that was previously located at the R = kNA , we obtain the same result as above.
broken part is m = Sh, where = 103 kg/m3 . Assuming ii) Clearly, the pressures inside and outside of the stratostat are
that the surface energy is transformed to the kinetic energy of equal and depending on the height as p = p0 ez , where
the moving front, we get an equation for the speed of the = RT ag
. For helium inside the sack pV = const. Let the
q front:
volume of the stratostat be VS , therefore p0 VS = pVS , where
2S = 12 mv 2 , 4S = Shv 2 . Therefore v = 4
h
=
p is the pressure at the height, where helium has filled the entire
4 0.025
q
106 103
m/s = 10m/s and we can estimate the time of volume. Since p = p0 , we obtain RT ag
h = ln , h =
RT 1
breakup of the soap film to be t = D/v = 100 1
s. a g
ln , h = 17 km.
8. Magnetic pulse (7 points) Since we can neglect the inductance 10. Wedge (5 points) The center of mass of the system doesnt
of the coil, it performes as a voltage source, which outputs U = move, therefore M u = mvh , where u and vh are the horisontal
N SB/ = 1 V during the time period between t = 0 ms and components of the velocities of the wedge and block in the lab
10 ms, and 0 V otherwise. The characteristic time scales of the frame at some instant of time. In the wedges frame, the block has
ESTONIAN FINNISH PHYSICS OLYMPIAD
PROBLEMS & SOLUTIONS (2010)
Estonian-Finnish Olympiad - 2010 is related to).
i) (3 pt) Knowing that the chicken leg is approximately l =
1. Charges in E (8 points) Two particles (the blue and the red)
15 cm tall, estimate the length of a tyrannosaur leg L. You may
of mass m are connected with a spring, the stress-free length of
assume that the length of a leg scales as the length of the whole
which is L and stiffness k ; the blue carries charge q (q > 0)
animal, and that the bone stress (force per area) is the same for
and the red is chargeless. In the region x > 0, there is an
both animals. Is your result consistent with the step length A?
homogeneous electric field E , antiparallel to the x-axis; In the
ii) (3 pt) Estimate the natural walking speed of the tyrannosaur by
region x < 0, there is no electric field. Initially, the dumbbell
approximating the walking motion of a leg with a freely oscillating
of charges moves in region x < 0 with velocity v , parallel to the
pendulum motion. State clearly all the assumptions you make.
x-axis; the dumbbells axis is also parallel to the x-axis and the
4. Ball (6 points) Massive spherical ball has a mass M = 100 kg;
spring is stress-free. It is known that after a while, the dumbbell
an attempt is made to roll the ball upwards, along a vertical wall,
moves in the region x < 0 with velocity v , and that the red
by applying a force F to some point P on the ball. The coefficient
particle never enters the region x > 0. Also, the springs length
of friction between the wall and the ball is = 0.7.
achieves minimum only once.
i) (5 pt) What is the minimal force Fmin required to achieve this
i) (2.5 pt) How long time does the blue particle spend in the
goal?
region x > 0?
ii) (1 pt) On a side view of the ball and the wall, construct
The process takes place exactly as described, if one equality geometrically the point P , where the force has to be applied to,
and one inequality are satisfied for the quantities m, v , k , q , E together with the direction of the applied force.
and L. 5. Elastic thread (10 points) Equipment: ruler, tape, an elastic
ii) (3 pt) Which equality must be satisfied? thread, a wooden rod, marker, a known weight.
iii) (2.5 pt) Which inequality must be satisfied? The purpose of this problem is to study the elastic properties
2. Thermos bottle (6 points) In order to study the thermal of an elastic thread for large relative deformations = (l
properties of a thermos bottle, let us model it as two concentric l0 )/l0 , where l0 and l are the lengths in initial and stretched states,
spherical vessels, with radii R1 = 7 cm and R2 = 10 cm. The respectively. If the Hooks law were valid, the ratio F/ of the
gap between the walls of the vessels contains vacuum (hence, the elastic force F and would be constant: F/ = SE , where S is
heat conductivity can be neglected). the cross-section area of the thread and E the Young modulus
i) (3,5 pt) Find the radiative heat flux (i.e. transmitted heat per of the thread material.
unit time) between the walls of the bottle, assuming that the i) Collect the data needed to plot the ratio F/ as a function
ambient temperature is T2 = 293 K and the inner sphere is , up to 4. Plot the appropriate graph, and indicate the
filled with liquid nitrogen at the boiling temperature T1 = 77 K. uncertainties.
The emissivities of all the surfaces are equal to that of stainless ii) By making assumption that the Young modulus E = F/S
steel: = 0.1. Remark: The emitted heat flux per unit area remains constant, study, how does the volume of the thread
is given Stefan-Boltzmanns law P = T 4 , where = depend on . Plot the appropriate graph.
5.67 108 W/m2 K4 (assuming that is independent of the
wavelength.
ii) (2,5 pt) Estimate, how long time does it take for a full
evaporization of the liquid nitrogen (the vapor escapes through an
over pressure valve). For the liquid nitrogen, density = 810 g/l
and latent heat for vaporization = 5.580 kJ/mol). NB! If you
were unable to find P (for question i), express the evaporization time
symbolically (i.e. using the symbol P ).
3. Tyrannosaur (T. Rex) (6 points) Paleontologists have
discovered tracks of a tyrannosaur where the footprints of the
same leg are A = 4.0 meters apart. They have also recovered
a piece of a tyrannosaur leg bone that has bone cross-sectionalo
area N = 10000 times that of a chicken (which the tyrannosaur
6. Charges in B (5 points) There is an homogeneous, parallel to sprinkler. i) (2 pt) What is the coefficient of friction, assuming a four-wheel
the z -axis magnetic field of inductance B in region x > 0. In ii) (1.5 pt) At which distance from the sprinkler is the watering drive?
region x < 0, there is no field. There are two particles of mass m intensity (mm/h) the highest? Because of a manual gear change, there is time period of
and charge q . Initially, the particles have coordinates y = z = 0, 9. Power supply (6 points) 1 = 0.5 s, during which there is no driving force (so that the
and respectively x = L0 and x = 2L0 (with L0 > 0). car decelerates due to air friction). Except for that period, the
Initial velocity of both particles is v , along the x-axis, towards the acceleration follows the law given by the graph. As a result, the
magnetic field. Neglect the electrostatic repulsion force of the two terminal velocity vt = 40 m/s is reached 2 = 1.0 s later than it
charges. would have been reached, if there were no delay caused by the gear
i) (1,5 pt) Sketch the trajectory of the first particle, and the change. Upon reaching the terminal velocity, the car continues
dependance of its y -coordinate on time. moving at constant speed. In your calculations, you can assume
ii) (3,5 pt) Sketch the distance L between the particles as a i) (2 pt) Consider the cirquit given in Fig (a), where the diode that the air friction was constant during the gear change period.
function of time t, assuming that mv/Bq > L. What is the can be assumed to be ideal (i.e. having zero resistance for forward ii) (2,5 pt) At which speed the gear was changed?
minimal distance Lmin ? current and infinite resistance for reverse current. The key is iii) (2,5 pt) How many meters shorter distance will be covered
7. Satellite (5 points) switched on for a time c and then switched off, again. The input during the first 100 seconds, as compared to ideal acceleration (i.e.
i) (3 pt) A large ball of mass m1 is kept at the height h from the and output voltages are during the whole process constant and without the delay due to the gear change)?
floor (so that the center of the ball is at the height h + d/2, where equal to Ui and Uo , respectively (2Ui < Uo ). Plot the graphs 11. Black box (10 points) Equipment: a black box, multimeter,
d is its diameter). A small ball of mass m2 is placed upon the of input and output currents as functions of time. battery, timer (on the screen).
large one, and the system is released (so that it starts falling). To ii) (2 pt) Now, the key is switched on and off periodically; each Determine the electrical scheme inside the black box, and the
which height (from the floor) will the small ball rise, assuming that time, the key is kept closed for time interval c and open also values of all the resistors inside it. Estimate the characteristics of
the collision between the lower ball and ground, and the collision for c . Find the average output current. other electrical components. It is known that apart from the wires,
between the balls are absolutely elastic, and m1 m2 ? the total number of components is three.
ii) (2 pt) Consider the following satellite launching project. There
are N absolutely elastic balls of masses m1 m2 ...
mN : the first ball (the heaviest) is the lowest; the second ball is
placed on top of the first; the third on top of the second etc.
The upmost ball is supposed to become a satellite, i.e. to obtain iii) (2 pt) Now, cirquit (a) is substituted by cirquit (b); the switch
the velocity vN = 7.8 km/s). The lowest ball is at the height is switched on and off as in part ii. What will be the voltage on
h = 1 m from the floor, and the system is released. What should the load R, when a stationary working regime has been reached?
be the number of balls N ? What should be the mass of the lowest You may assume that c RC , i.e. the voltage variation on the
ball, if mi /mi+1 = 10, and the mass of the satellite MN = 1 kg? load (and capacitor) is negligible during the whole period (i.e. the
8. Sprinkler (3 points) charge on the capacitor has no time to change significantly).
A sprinkler has a shape of hemisphere, which has small holes 10. Ice-rally (7 points) The car accelerates on a slippery ground
drilled into the spherical part of its surface. From these small so that the wheels are always at the limit of slipping (e.g. via using
holes, water flows out with velocity v = 10 m/s. Near an electronic traction control). Such an acceleration would result
the sprinkler, the water flow is distributed evenly over all the in the velocity vs time graph as given in the Figure.
directions of the upper half-space. The sprinkler is installed at the
ground level so that its axis is vertical. In what follows, the air
resistance can be neglected, and the dimensions of the sprinkler
can be assumed to be very small.

i) (1.5 pt) Find the surface area of the ground watered by the
ESTONIAN FINNISH PHYSICS OLYMPIAD
PROBLEMS & SOLUTIONS (2011)
Estonian-Finnish Olympiad - 2011 plates equals still to E . What is the hydrostatic pressure between
1. Spool (12 points) A spool with inner radius r and outer radius the plates if the atmospheric pressure is p0 and the pressure of the
R lies on a horizontal table; the axis of the spool is horizontal. A water column can be neglected?
weightless rope is wound around the inner part as shown in the 3. Charged cylinder (8 points) Dielectric cylinder of radius r
picture. The loose end of the rope makes an angle with the hori- carries a charge of surface density on its cylindrical surface and
zont (the angle can be also negative). The moment of inertia of rotates with angular velocity .
the spool is J and mass M . In what follows you may assume i) (3 pts) Determine the magnetic induction B inside the cylin-
that the spool rolls on the table without slipping. der. Remark: if you wish, you can use the expression for the in-
i) (2 pts ) We pull the loose end of the rope with velocity u (par- ductance L = 0 N 2 S/l of a solenoidal coil of radius r , length
allel to the loose part of the rope; that loose part can be thought l r , area of cross-section S and number of loops N .
to be very long). What is the velocity of the spool? ii) (3 pts) A radial conducting wire connects the axis of the cylin-
ii) (3 pts) Suppose now that the spool is at rest, and we apply force der with the cylindrical surface (it rotates together with the cylin-
F to the loose end of the rope (parallel to the loose part of the der). Find the electromotive force (voltage) E between the ends
rope). What is the acceleration of axis of the spool? of the wire.
iii) (2 pts) How large does the coefficient of friction need to be iii) (2 pts) Suppose that the wire connecting the axis of the cyl-
(as a function of ) to ensure that there is no slipping between the inder with the cylindrical surface is not radial and has an arbitary
spool and the table? shape (still, there are no segments protruding outside the cylin-
iv) (1.5 pts) Now the spool rolls, again, with velocity u; however der). Show that E does not depend on the shape of the wire.
there is no rope. The spool hits a threshold of height H (see Fig- 4. Black box (10 points) Equipment: a black box with three ter-
ure); the impact is perfectly inelastic. What is the speed v of the minals, voltmeter, timer. Inside the black box, there are two capa-
spool immediately after the impact? citors and a battery, connected as shown in Figure. The capacit-
v) (1.5 pts) What is the speed w of the spool after rolling over the ance C1 = (3400 400)F; you are asked to determine the
threshold? Assume that u is such that the spool will roll over the capacitance C2 and estimate the ucertainty. Remark: the terminal
threshold without losing contact with its edge. + is a wire, long enough to be conected to either terminal A or
vi) (2 pts) If the speed u is too large, u > u0 , the spool will jump terminal B.
up and lose contact with the edge of the threshold. Determine u0 .

2. Capacitor (6 points) An ideal plate capacitor has plates with


area A and separation d and is charged so that the electric field
between the plates equals to E .
i) (2 pts) Find the energy density of the electric field inside the
capacitor and the total energy of the field.
ii) (1.5 pts) What is the force required to keep the plates separ-
ated?
iii) (2.5 pts) Now, this capacitor is submerged into distilled water
of dielectric permittivity = 80; the electric field between the
5. Plutonium decay (3 points) Plutonium is an unstable element, 7. Vacuum bulb (8 points) Let us study how a vacuum can be i) (2 pts) The electronic component has been dissipating energy
a Pu239 atom decays with a half-life of 1/2 = 24 000 years created inside a bulb by pumping. Let the volume of the bulb be with a constant power of P = 35 W for a long time, and the av-
by creating smaller nuclei, including an -particle. Find the - V , and the pump consist in a piston moving inside a cylinder of erage plate temperature has stabilized at the value T0 = 49 C.
particle flux density (i.e the number of passing nuclei per unit volume V , where 1. The pumping cycles starts with piston Now, the component is switched off, and the average plate tem-
time and per unit cross-sectional area) near the surface of a plate being pulled up; when the pressure inside the cylinder becomes perature starts dropping; it takes = 10 s to reach the value
of Pu239 . The plate has thickness d = 1 mm; its width and smaller than inside the bulb, a valve VA (connecting the cylinder T1 = 48 C. Determine the heat capacity C (units J/ C) of the
length are much larger than that. The density of plutonium = and the bulb) opens and remains open as long as the piston moves plate. The capacities of the electronic component and the temper-
19 800 kg/m3 . Remark: half-life is the period of time it takes up. When piston is released, it starts moving down, at that mo- ature sensor are negligible.
for a substance undergoing decay to decrease in size (in the num- ment, the valve VA closes. As long as the valve VA is open, the ii) (4 pts) Now, the electronic component has been switched off
ber of particles) by half. The mass of an atom of Pu239 is m0 = pressures of the bulb and the cylinder can be considered as equal for a long time; at the moment t = 0, a certain amount of heat
3.84 1025 kg. to each other. When the piston moves down, the pressure in the Q is dissipated at it during a very short time. In the Figure and
6. Violin string (9 points) The motion of a bow puts a violin cylinder increases adiabatically until becoming equal to the out- Table, the temperature is given as a function of time, as recorded
string into a periodic motion. Let us make a simplified model of side pressure p0 = 105 Pa; at that moment, another valve VB by the sensor. Determine the dissipated heat amount Q.
this process. The string has elasticity and inertia, so we substitute opens leting the gas out of the cylinder. When the piston reaches
it by a block of mass m, fixed via a spring of stiffness k to a mo- the bottommost position, there is no residual air left inside the
tionless wall and laying on a frictionless horizontal surface. The cylinder. Now, the piston is ready for being lift up: the valve VB
bow is substituted by a horizontal plate, which is pressed with closes and VA opens, marking the beginning of the next pumping
constant force N downwards, and which moves with a constant cycle. The air inside the bulb can be considered isothermal, with
velocity u, parallel to the axis of the spring, see Figure. The static the temperature being equal to the surrondin temperature T0 . The
coefficient of friction between the plate and the block is 1 , and adiabatic exponent of air = cp /cV = 1.4.
the kinetic coefficient of friction is 2 < 1 . So, as long as the i) (2 pts) How many pumping cycles N needs to be done to re-
plate does not slide with respect to the block, the coefficient of duce the pressure inside bulb from p = p0 down to p = p0 ,
friction equals to 1 ; as soon as there is some slip, it decreases where 1?
down to 2 . ii) (2 pts) What is the net mechanical work done during such a
pumping (covering all the N cycles)?
iii) (2 pts) What is the temperature of the air released from the
cylinder to the surroundings at the end of the pumping process t (s) 0 20 30 100 200 300
(when the pressure inside the bulb has become equal to p0 )? T ( C) 20.0 20.0 20.4 32.9 41.6 42.2
iv) (2 pts) According to the above described pumping scheme, t (s) 400 600 800 1000 1200 1400
there is a considerable loss of mechanical work during the period T ( C) 39.9 33.4 27.9 24.4 22.3 21.2
i) (2 pts) For questions (i) and (ii), let us assume that the speed when the piston is released and moves down. Such a loss can 9. Coefficient of refraction (10 points) Equipment: A thick glass
of the plate u is very small as compared to the maximal velocity of be avoided if there is another pump, which moves in an oppos- plate having the shape of a half-cylinder, a glass prism, a container
the block. What is the maximal velocity of the block vmax (max- ite phase: the force due to outside air pressure pushing the piston with an unknown liquid, a laser pointer, graph paper, ruler.
imized over time)? down can be transmitted to the other pump for lifting the piston i) (5 pts) Determine the coefficient of refraction of the half-
ii) (2 pts) Sketch qualitatively the graph of the displacement of the up. What is the net mechanical work done when such a pumping cylindrical glass plate and estimate the uncertainty of the result.
block as a function of time and indicate on the graph the durations scheme is used? ii) (5 pts) Determine the coefficient of refraction of the liquid and
of the prominent stages of the block motion (graph segments). 8. Heat sink (6 points) Consider a heat sink in the form of a cop- estimate the uncertainty of the result.
iii) (1,5 pts) Now, let us abandon the assumption about the small- per plate of a constant thickness (much smaller than the diameter
ness of u. Sketch qualitatively the graph of the velocity of the block d of the plate). An electronic component is fixed to the plate, and a
as a function of time. temperature sensor is fixed to the plate at some distance from that
iv) (2,5 pts) Determine the amplitude A of the blocks oscilla- compnent. You may assume that the heat flux (i.e. power per unit
tions. area) from the plate to the surrounding air is proportional to the
v) (1 pt) Which condition (strong inequality, or ) must be difference of the plate temperature at the given point (the coeffi-
satisfied for u in order to ensure that the oscillations will be almost cient of proportionality is constant over the entire plate, including
harmonic? the site of the electronic component).
Estonian-Finnish Olympiad - 2011: solutions Using the result of the previous task, we can use this equation dir- iii) Let us push away part of the water from the inter-plate space
1. Spool (12 points) i) First solution. The momentary rotation ectly to obtain an expression for the minimal allowed value of the so that there will be a small region of plate area dA, where there is
centre of the spool is the contact point P with the floor (since coefficient of friction: r no water between the plates (here, p is the pressure difference
J 2 cos

this point is at rest). So, the velocity of the spool is u = R , R MR
between the inter-plate space and the outside regions). By doing
min =  Mg
so, we perform work d A p, and increase the capacitors en-
where is the angular velocity. Consider triangle P OA, where A 1 + MRJ
F sin

is defined as the point where the loose end of the rope meets the
2
ergy:
inner part of the spool at the current moment of time, but which is iv) The angular moment of the spool with respect to the edge of Q2 d 1 1
 
2
W = (Q /2C) = .
actually a point of the spool, i.e. it rolls together with the spool); the threshold conserves during the impact (since the impact force 20 A (A A) + A
O is the centre of the spool. Let us denote 6 P AO = ; it is easy has zero arm): u  v So,
to see that 6 AOP = . The velocity ~vA of the point A M u(R H) + J = J + M R2 Q2 d( 1) A 1
is perpendicular to P A and, hence, forms angle with the loose R R W = = 0 E 2 d( 1) A;
H/R
! 20 A
2 2 2
end of the rope. Its projection to the rope equals to u, therefore
v =u 1 J comparing this with the pressure work d A p we conclude
vA = u/ cos . Further, = vA /l,where l = |AP | can be 1 + MR
2 2
2
that
found from the cosine theorem: l = R + r + 2Rr cos . v) From the energy conservation law we obtain immediately 1 1
The angle can be found using the sine theorem for the triangle 2 2 p = 0 E 2 ( 1) p = p0 + 0 E 2 ( 1).
2 v 2 w 2 2
AOP : sin = R sin
. Combining everything together we end (J + M R ) = (J + M R ) + 2M gH
3. Charged cylinder (8 points)
up with
l R2 s R2
2gH i) Moving surface charge creates a solenoidal surface current with
uR uR w = v2
u = 2 = . J . the surface density j = v = r . From the circulation the-
R cos2 + r 2 + 2Rr cos |R cos + r| 1 + MR 2
orem for a rectangular loop embracing a segment of surface cur-
Second solution. Let us decompose the velocity ~vA into two vi) The spool is the most prone to jumping immediately after the rent we obtain Bl = jl, where l is the length of the surface current
0
components: the tangential component (parallel to the rope) impact; the gravity force needs to be large enough to bind the
segment (so that jl gives the current flowing through the loop).
equals (by modulus) to u; let us denote the radial component as centre of mass to the rotational motion around the edge of the
Hence, B = 0 j = 0 r .
threshold:
ur . Since the distance between O and A is constant, the projec- ii) Using formula E = d = B dS , where S is the area covered
tion of the velocities of O and A to the line OA are equal: M v2 RH g dt dt
g v2 (R H) by the wire, we obtain E = Br 2 /2. Indeed, during a small time
ur = v sin v = ur / sin = ur /R sin . R R M interval dt, the wire covers a equilateral triangle of side lengths r ,
J
g 1 + MR
r
The vertical component of the velocity of the point A remains un- u0 = (R H)
2 r , and rdt; its area is apparently r 2 dt/2. By using the earlier
J H .
changed if we switch the laboratory system of reference with the M 1 + MR2
R
obtained expression for B we end up with
system associated with point O ; hence, 2. Capacitor (6 points) E = 0 2 r 3 /2.
u sin ur cos = r sin = ur r/R i) The energy is W = CU 2 /2 = 12 0 Ad E 2 d2 = 12 0 AdE 2 ; iii) We need to show that from the previous task, dS is independ-
dt
ur uR hence, the energy density w = W/Ad = 12 0 E 2 . ent of the wire shape. First we note that due to rotational sym-
v= = . ii) There are two ways to calculate the force. First, we notice that metry, dS , it cannot depend on the rotation angle, i.e. dS S =
sin R cos + r the innermost charges q at the capacitor plates are affected by the dt dt
ii) (2 pts) The easiest way to solve this part is to use the energy Const. Further we note that regardless of the wire shape, during
electric field E, therefore there is a force qE acting upon these. The the entire rotation period 2/ , the whole circle area is covered;
balance for infinitesimal displacement of the cylinder and apply
outermost charges, however, have no electric field around them S 2/ = r 2 S = r 2 /2.
the answer to the previous question:
 (because outside the inter-plate space, there is no electric field).
M 2 J J 4. Black box (10 points) There are several ways to perform this
   
F u dt = d v 1+ = M vdv 1 + Due to the Gauss law, the electric field decreases linearly with the task. First one can notice that if two capacitors discharge at the
2 M R2 M R2 net charge left below the level of the current point (i.e. towards same resistor, starting with equal voltages and ending also with
dv Fu F cos + Rr the inter-plate space). Therefore, the electric field averaged over
a= =  = . equal voltages, the ratio of the discharge times equals to the ratio
dt M v 1 + MR J
M 1 + J the charges is just half of the maximal value E : hEi = 12 E , and
2 MR2 of the capacitances (because for each given voltage, the discharge
iii) Let us write the force balance projection to the horizontal axis the net force acting on the plate is F = Q hEi = CEd hEi =
1 2 currents are the same, but larger capacitor has more charge pro-
assuming that the spool is at the edge of slipping, i.e. the friction 2 0 AE .
portionally to the capacitance). Therefore we can first charge the
force Ff = min N , where N = mg F sin is the normal The second way includes writing the energy balance for a small known capacitor (using the battery), and let it discharge on the
2
force: displacement of a plate: F d = (Q2 /2C) = 2Q0 A d = voltmeter (which has some finite resistance), measuring the time
M a = F cos + min N = F cos + min (M g F sin ). 12 C 2 E 2 d d F = 12 0 AE 2 . t1 required for it to reach a pre-defined final voltage. Then we need
to repeat the procedure with the other capacitor and measure the straight line is tangent to the sinusoid. The length of a straight seg- iii) Due to adiabatic law, pV = Const; when combined with
time t2 and calculate
 C2 = C1 t2 /t1; the uncertainty is estimated ment can be calculated as the gas law pV T we obtain p1 T . During the last
as C1 = C1 t1 + tt1 2
+ C 1
. T1 = 20 /u = 2(1 2 )N/ku; downwards motion of the piston, the pressure inside the cylinder
t2 C1 1

It is recommended to check the negligibility of the leak current the sinusoidal segment p corresponds to a half-period and therefore is increased by a factor of 1/ ; thus, T = T0 1 .
across the plates of the capacitor. To this end, one can charge a ca- has a length of T2 = m/k . iv) According to the modified pumping scheme, the work/energy
pacitor, measure the voltage, remove the voltmeter and wait for loss is only due to the release of the hot air. Note that if we had
some time (of the order t1 and t2 ), and check again the voltage. a cylinder of volume V , we could be able to create vacuum inside
Another way is to discharge completely one capacitor by there using only one pumping motion, i.e. by performing work
short-circuiting its terminals and charge the other capacitor up to A = p0 V and without any energy loss. Now, we perform an ex-
cess work, which is converted into internal energy of the released
the voltage of the battery. Further, we connect the terminals A iii) The speed v(t) = dx is the derivative of x(t); therefore, the
and B so that the capacitors re-distribute the charge Q = EC1
dt hot air, which needs to be calculated. Let = pp0 be an inter-
sinusoidal segment of x(t) will correspond to a sinusoidal seg-
and take the same voltage: Q1 /C1 = (Q Q1 )/C2 Q1 = ment of v(t), and a straight segment of x(t) to a horizontal mediate rarefaction factor; then, we can apply the previous result
QC1 /(C1 + C2 ) = EC12 /(C1 + C2 ). Consequently, the new segment of v(t). The resulting graph is depicted below. to calculate the internal energy of released air, if its quantity is d
1
voltage (which we measure) is U = Q1 /C1 = EC1 /(C1 +C2 ), moles: dU = T0 ( 1 1)cV d . Let us note that the num-
from where C2 = ( UE 1)C1 . ber of moles inside the bulb is = pRT 0 V
0
d = RT p0 V
0
d . So,
5. Plutonium decay (3 points) cV
R 1 1
1
U = p0 V R 0 ( 1)d = ( 1)p0 V R . Now, recall c V

Let the number of Pu239 -atoms be reduced during time interval that = cp /cV = 1 + cRV , hence cRV = 1 1
and U = p0 V .
t = 1 s by a factor of 1 (with 1). Then, during the
This gives us the energy loss due to heating the released air; an-
time period of 1/2 , it is reduced by a factor of (1 )1/2 /t
other p0 V is required for loss-free creation of the vacuum. Hence,
e1/2 /t = 12 = t ln 2/1/2 . Therefore, the number of the total required work is A = 2p0 V .
atom decay events is Nd = N t ln 2/1/2 , where N = dS/m0 8. Heat sink (6 points)
is the number of atoms, i.e. the -particle flux is = Nd /2St iv) Let the amplitude of the oscillations be A, i.e. the sinusoidal
p i) When the average temperature is stable at T0 , all the power dis-
(where the factor 2 accounts for the fact that the particles are emit- segments follow the law (t) = A cos(t), where = k/m.
sipated at the electronic component is eventually given to the air:
ted towards the both sides of the plate). Upon bringing all the ex- Correspondingly, v(t) = A sin(t) A sin(t) =
the air is being heated with power P . As the heat flux depends lin-
pressions together, we obtain v(t)/ ; hence, for any point at a sinusoidal segment, 2 +
early on the temperature difference between a point on the plate
d ln 2 v 2 / 2 = A2 . At a point, where a sinusoid and a straight line
= 2.36 1013 m2 s1 . and the air, the average heat flux and therefore the net power dis-
meet, the straight line and sinusoid have equal values for =
21/2 m0 0 = (1 2 )N/k and v = u. Consequently, sipated into the air depends linearly on the average temperature of
6. Violin string (9 points) the plate. The average temperature determines the radiated power.
(1 2 )2 N 2 /k2 + u2 / 2 = A2
i) When the plate slides, there is a constant friction force 2 N 1q Now consider the situation after the heating has ended. The
acting upon the block, which means that the equilibrium deform- A= (1 2 )2 N 2 + u2 mk. average temperature is initially the same, so the radiated heat
k power is initially still P . By the definition of heat capacity, an
ation of the spring is x0 = 2 N/k ; the net force acting upon the v) The oscillations will be almost harmonic when the straight seg-
infinitesimal heat amount given to the surroundings is dQ =
block (due to spring and friction) is given by F = k , where ments are very short,i.e. when u/ (1 2 )N/k C dTavg with the minus sign encoding the direction of the heat
we have defined = x x0 . Therefore, while sliding, the block u (1 2 )N/ mk.
oscillates harmonically around the point = 0. Slipping starts 7. Vacuum bulb (8 points) flow. Thus, at the first moment, P = dQ dt
= C dTdtavg . Assum-
when the static friction will be unable to keep equilibrium, i.e. i) Each pumping cycle reduces the number of molecules inside ing that during the average temperature depends approximately
at kx = 1 N , which corresponds to 0 = (1 2 )N/k . the bulb by a factor of (1 ); therefore, after N cycles, the linearly on time (because T0 T1 = 1 C is much less than the
dT
If the plate moves slowly, the block is released with essentially number of molecules (and hence, the pressure) by a factor of usual ambient temperature), dtavg T1 T
0
and C T0PT
1
=
missing kinetic energy, and the energy conservation law yields = (1 )N eN
350 J/ C. Actually the graph of Tavg (t) is slightly curved down-
1 1
2 2
p
2
k 0 = 2
mv max vmax = 0 k/m . ln wards (as it is an exponential eventually stabilizing at the ambient
ii) As mentioned, when the plate slides, the motion of the block is N = . temperature) and initially somewhat steeper, so C is a bit smaller.

harmonic, i.e. the graph of x(t) is a segment of a sinusoid; when ii) Majority of the pumping cycles are done when the pressure in- ii) The average temperature of the heat sink falls off exponentially,
there is no sliding, the block moves together with the plate, i.e. the side the bulb is negligible as compared to the outside pressure. therefore, if the tail of the given graph turns out to be expo-
graph of x(t) is a straight line. At the moment when slipping starts During such a cycle, a work equal to p0 V is done. Therefore, nential, we can presume the tail depicts the situation where the
or stops, the oscillatory speed is equal to the speed of plate, i.e. the A N p0 V = p0 V | ln |. sensor is sensing the average temperature and the initial bump
in the temperature distribution has evened out. Extrapolating the but not too much: Tc is exponentially sensitive to the T -intercept
exponential to t = 0 we get the initial average temperature Tavg,0 of the straight line fitted to the tail (its crossing point with the
(immediately after the Q has been dissipated into the sink) and, T -axis) on the logarithmic plot. The bump has still not yet disap-
by Q = C(Tavg,0 Tamb ), the heat Q. The ambient temper- peared completely enough.
ature Tamb can be read off from the beginning of the given graph 9. Coefficient of refraction (10 points)
where the sensors surroundings have not yet heated up. This is i) We direct the laser beam radially into the semi-cylinder: per-
furthermore a check for the assumption T0 T1 Tamb made pendicularly through its cylindrical surface. The beam enters the
in the first part of the solution. From the table, Tamb = 20.0 C. plate without refraction and reaches the opposing flat face at the
Let us analyse the (yet hypothetical) exponential Tavg Tamb axis of the cylinder. Depending on the angle between that face and
ought to obey, so that eventually we expect T Tavg = the beam, there may or may not be a refracting beam, but there is
t
Tamb + Tc e tc where Tc and tc are, respectively, a characteristic always a reflecting (from the flat face) beam . We rotate the semi-
cylinder around its axis to find the position, when the refracting
temperature and a characteristic time. (The means is asymp-
beam appears/disappears; the angle between the flat face and
totical to or approaches.) We plot ln(T Tamb ) using the data
the incident beam correspond to the angle of complete internal
from the table. Then approximate the tail linearly (valuing the
reflection, i.e. n = 1/ cos . We can measure cos using the
end of it most) to get ln[(T Tamb )/ C] 4.89 300t s .
graph paper: we draw the beam as a segment AO and the flat face
Therefore Tc e4.89 C 133 C. On the other hand,
of the semi-cylinder as a line BO so that 6 ABO = /2; then,
plugging t = 0 into our exponential function shows that
n = |AO|/|BO|. The uncertainty can be found using the for-
Tavg,0 Tamb = Tc and, finally, Q = CTc 46 700 J.
mula n = n( |AO| |AO|
+ |BO|
|BO|
) and by estimating the uncer-
tainties of the direct length measurements |AO| and |BO|.
ii) We drop the liquid on the prism and press it against the flat face
of the semi-cylindrical plate. Further we study the complete in-
ternal reflection at the boundary between the semi-cylinder and
prism (which is filled with the liquid) by repeating the above
described experiment. Thereby we measure new lengths A O
and B O ; the condition of complete internal reflection is now
n/nl = |A O|/|B O| nl = n|B O|/|A O|, where nl
stands for the coefficient of refraction of the liquid. The uncer-

O| O|
tainty is now calculated as nl = nl ( |A |A O|
+ |B
|B O|
+ n
n
).

Actually, quite a good result can be obtained without re-


plotting anything, by just considering the last three datapoints
of the table. Denote Ti Ti Tamb . If the times t3
t2 = t2 t1 , then with an exponential we should observe
that T3 /T2 = T2 /T1 . The last three timepoints are
good indeed, so we check T1 = 4.4 C, T2 = 2.3 C
and T3 = 1.2 C. Their ratios are T3 /T2 0.522 and
T2 /T1 0.523, a splendid match. This confirms the ex-
ponential tail. As in every equal time interval the T is mul-
tiplied by the same number (that is the essence of exponentials),
  t tt
3

Tc = Tavg,0 = T3 T 2
T3
3 2
114 C. From this,
Q 39 900 J. This is discrepant from our previous calculation,
ESTONIAN FINNISH PHYSICS OLYMPIAD
PROBLEMS & SOLUTIONS (2013)
Estonian-Finnish Olympiad 2013 The focal length of its camera f = 4.3 mm been bent into the shape of a square with
and the diameter of the lens D = 1.8 mm. The side length a. At some point on the wire is
1. PRISM (8 points) sensor is w = 4.6 mm wide corresponding to a small ideal current source that keeps cur-
N = 3264 pixels. rent I flowing in the circuit in all situations.
i) (4 points) A right prism that has an
3. MISSION TO MARS (7 points) A crew of The magnetic moment m of a planar circuit
equilateral triangular base with length a is
astronauts is going to be sent to explore the is given by the relation m = I A , with vector
placed in a horizontal slit between two tables,
polar region of Mars and search for buried ~ pointing in the normal direction of the cir-
m
so that one of the side faces is vertical. How
water ice. Their spaceship will travel from cuit according to the right hand rule ( A is the
small can the width d of the slit be made be-
Earth to Mars along an elliptic transfer orbit area bounded by the circuit).
fore the prism falls out of the slit? There is no
friction between the prism and the tables and tangential to the orbits of both planets. Des- i) (3 points) The dipole is placed inside a
the prism is made of a homogeneous material. pite its shortcomings, this orbit is commonly homogeneous magnetic field B ~ , so that the
The edges of the slit are parallel. used in space travel due to its relatively good angle between m ~
~ and B is . Find the angles
fuel economy. Future manned missions to s and u that correspond to stable and un-
ii) (4 points) Now the prism is placed in the
Mars are very likely based on this kind of stable equilibria, respectively. Calculate the
slit so that one of its side faces is horizontal.
transfer. In this problem you will examine amount of work (w) needed to rotate the di-
How small can the width l of the slit be made
some aspects of this orbit. pole from s to u . Give your answer in terms
before that position becomes unstable?
The mean orbital radius of Mars is R a = of m and B.
i) ii) a 1. 52 AU. The mean orbital radius of Earth We can use this model to calculate the
is R g = 1 AU = 149 600 000 km. Mars has a magnetic properties of materials containing
mean radius of r a = 3397 km and surface unpaired electrons that have negligibly weak
a
gravity g a = 3.71 m/s2 . Earth has a mean interactions with one another. Let us con-
radius of r g = 6371 km. sider a sample of material with n such un-
i) (1 point) Find the orbital period T a of paired electrons per unit volume, placed in-
l ~
Mars, i.e. find the length of Martian year side a homogeneous magnetic field B. Due
d

in Earth years. to spin, each unpaired electron acts as a


2. CELLPHONE CAMERA (6 points) A pho- small magnetic dipole. However, owing to the
tographer focussed his camera to distance L ii) (1.5 points) How long ( t t ) does a one-way quantum nature the electron, the projection
and took a photo. On the photo, all farther trip to Mars take? of its magnetic moment along B ~ can only be
objects (up to infinity) turned also out to be iii) (1.5 points) The spaceship is put into this B or B (B is called the Bohr magneton).
sharp. Additionally, all closer objects down orbit by using a powerful rocket. It is more ii) (4 points) Calculate M , the magnetic mo-
to distance s were sharp. efficient to burn fuel as a short burst when ment per unit volume of the sample, if the
i) (4 points) What is the minimum pos- the spaceship is still near Earth. How much temperature of the material is T and the ex-
sible L? additional speed (v1 ) does the booster have ternal magnetic field is B.
to be able to give to the spaceship to enter the
ii) (2 points) Find the corresponding s. transfer orbit, starting from the north pole? 5. FRICTION OF A STRING (8 points)
Measure the dynamic coefficient of friction
Background. We consider the image of Neglect the air resistance.
1 between the ballpoint pen and the string.
a pointlike object to be sharp if its image is iv) (1.5 points) Estimate the v needed to
2 Estimate the uncertainty. It might help that
smaller than one pixel on the sensor. Oth- enter a circular orbit close to Mars.
the dynamic coefficient of friction between
erwise the image is blurry. The lens of the
v) (1.5 points) What is the minimal duration the pencil and the same string was measured
camera may be viewed as a convex lens. The
of the trip to Mars and back? beforehand and 2 = 0.20 0.01 was obtained.
camera is focussed by changing the distance
between the sensor and the lens. 4. MAGNETIC DIPOLES (7 points) Let us Equipment: dynamometer, string, ball-
consider the following model for a magnetic point pen, pencil and weight.
Parameters. Calculate the answer for a
cellphone made by a well-known company. dipole. Some wire with no resistance has
Estonian-Finnish Olympiad 2013 8. ZENER DIODE (7 points) An inductance come to a halt. 10. RESISTIVE HEATING (8 points) Meas-
L and a capacitor C are connected in series iv) (2 points) Find the decrease q in the ure the resistor. You are not asked to estim-
6. SPHERE AND CYLINDER (7 points) A with a switch. Initially the switch is open ate the uncertainty.
maximum positive value of the capacitors
sphere and a cylinder are lying on an inclined and the capacitor is given a charge q 0 . Now charge q after one full oscillation. How long Equipment: resistor, voltage source (bat-
surface with inclination angle . Both have the switch is closed. does it take before oscillation halts? teries), ammeter, calorimeter, thermometer,
mass m and radius r . The bodies are released
i) (1 point) What are the charge q on the 9. GLASS CYLINDER (7 points) In the fol- stopwatch.
from equal initial heights H . The moments of
capacitor and the current I in the circuit as lowing figure, there is a half-cylinder, made The calorimeter has V = 0.80 dl of water
inertia of the sphere and the cylinder are, re-
functions of time? Draw the phase diagram of glass and put on a paper with stripes (the and m = 27 g of aluminum, specific heat ca-
spectively, I sph = 52 mr 2 and I cyl = 21 mr 2 . The a
of the system the evolution of the system inter-stripe distance is everywhere the same). pacity of water c = 4.2 J/(K g) and of alu-
coefficient of friction between the surface and w
on a I q graph and note the curves para- Find the coefficient of refraction of the glass. minum c = 0.90 J/(K g). Internal resistance
the bodies is . a
meters. Note the direction of the systems of the batteries will vary. Your set of batteries
i) (2 points) Which of the bodies comes down evolution with arrow(s).
may become depleted, spares are available.
faster? What was the relative lag of the
A Zener diode is a non-linear circuit ele-
slower body = ( t 2 t 1 )/ t 1 ? The times t 1 and
ment that acts as a bi-directional diode: it
t 2 , respectively, denote the traveling times of
allows the current to flow in the positive dir-
the faster or the slower body. Assume that
ection when a forward voltage on it exceeds
the rolling occurs without slipping.
a certain threshold value, but it also allows
ii) (2.5 points) Find the minimal angle of a current to flow in the opposite direction
inclination 0 for which the cylinder starts when exposed to sufficiently large negative
to slide in addition to rolling. voltage. Normally the two voltage scales are
iii) (2.5 points) If 90 , the bodies obvi- quite different, but for our purposes we will
ously lose contact with the surface and fall take a Zener diode with the following volt-
down in free fall with equal times. What ampere characteristics: for forward currents,
is the minimal angle of inclination m , for the voltage on the diode is Vd , for reverse
which both the sphere and the cylinder come currents, the voltage on the diode is Vd ,
down with equal times? for zero current the voltage on the diode is
Vd < V < Vd .
7. BURNING WITH A LENS (7 points) Sun-
rays are focused with a lens of diameter Now we connect the inductance L, the ca-
d = 10 cm and focal length of f = 7 cm to a pacitor C all in series with a switch and a
black thin plate. Behind the plate is a mirror. Zener diode. The switch is initially open. The
Angular diameter of the Sun is = 320 and capacitor is again given the charge q 0 > CVd
its intensity on the surface of the Earth is and the switch is then closed.
I = 1000 W/m2 , Stefan-Boltzmann constant ii) (2 points) Make a drawing of the phase
= 5.670 108 W/(m2 K4 ). diagram for the system. Note the direction of
i) (4 points) Find the temperature of the the systems evolution with arrow(s).
heated point of the plate. iii) (2 points) Does the evolution of the sys-
ii) (3 points) Using thermodynamic argu- tem only necessarily stop for q = 0? Find the
ments, estimate the maximal diameter of the range of values of q on the capacitor for which
lens for which this model can be used. the evolution of the system will necessarily
Estonian-Finnish Olympiad 2013 ii) (4 points) Let the corners be denoted the values of a and b gives
Solutions by A and B, and the tip of the prism (at
f2
its equilibrium position) by C . Let us con- f2 f+ L 1 + /D 2
1. PRISM (8 points) sider a small rotation of the prism (assum- f+ = =f f 1+
s 1 /D 1 /D D
i) (4 points) The prism is acted on by three ing it remains in contact with the corners).
2

~ l from the left-hand The trajectory of the tip is a circle ascribed f 1+ .
forces: reaction force R D
table, directed perpendicularly to the prisms around the triangle ABC (it follows from the
face; reaction force R~ r from the right-hand property of the inscribed angles because the Finally, f 2 / s = 2 f /D , or s = 12 D f / = L/2
6 ACB remains equal to 60 ). The radius of Lets consider that the camera is focused
table, directed horizontally (with its point p 2.75 m.
that circle r = l / 3; its centre will be denoted to distance L and the image is formed exactly
of action to be determined yet); and gravit- on the sensors plane. The objects distance L 3. MISSION TO MARS (7 points)
ational force mg, directed vertically and ap- by O . Once the prism rotates by angle , so
that the new position of the tip will be D , the and its images distance a (corresponds to p0 i) (1 point) We can find the orbital period
plied at the triangles centre. (Considering a on the figure) are related by the lens formula
central angle 6 COD = 2. Hence, the tip is of Mars from Keplers third law R a 3 /R g 3 =
planar triangular cross-section of the prism 1 1 1
raised by r r cos(2) 2 r 2 . The height of L + a = f , thus T a 2 /T g 2 , giving t t 1.87 yr.
is enough.)
the centre of mass P of the prism is raised ii) (1.5 points) Again, we can use Keplers
On the verge of falling out, the force R~r
because the tip is raised, and lowered be- third law to calculate half of the orbital
is applied at the lower corner of the triangle. Lf Lf Lf f f2

cause the vertical projection of the segment a= = 1+ = f+ , period.
If a body in equilibrium is acted on by three CP is reduced p by |CP |(1 cos ) |CP |2 /2. L f L(1 f /L) L L L
forces, then their lines of action must inter- Here, |CP | = a/ 3. So, the original position T g (R a + R g )3/2
sect at one point. This is because otherwise is stable if pa 2 /2 < 2 pl 2 , hence l > 14 a. tt = 0.707 yr.
3 3 2 (2R g )3/2
the torque of one of those forces would not be where the approximation (1 + x)1 1 x (for
a
zero with respect to the intersection of the small x) was used. Images distance exceeds
lines of action of the two other forces. iii) (1.5 points) Background. v is import-
the focal length by a = a f = f 2 /L.
ant, because the sum of all v determines
As the distance betweenp
the triangles i) (4 points) The light coming from an in- how much fuel is needed for a given mission.
centre and its side is 63 a, the distance P P0 finitely far away object will pass the focal The fuel needed is exponential of total v and
between ~ ~
the points of action of R l and Rpr is point F and form a cone which is cut by the is described by Tsiolkovsky rocket equation.
p
3 1 1 3 sensors plane. The diameter d of the cut on
6 a cos 30 = 4 a. Thus, d = 4 a cos 30 = 8 a. the sensors plane can be found from similar Kinetic energy per unit mass of such a
triangles d /D = a/ f , thus d = D f /L. Taking transfer orbit where it intersects the Earths
GM GM
into account the sharpness condition d , orbit is R g +Rs a + R g s . Using the orbital
A 2 O B
where = w/ N is the size of a single element angular speed of Earth we can substitute
42 R 3g
of the sensor, we find that the limiting value GM s = T . The speed at the beginning of
g
p of L is L = D f / = D f N /w 5.5 m.
3 DC the transfer orbit becomes
6 a
ii) (2 points) Well now find the shortest dis-
a
s
l 1 1

tance s satisfying the sharpness condition. v t0 = 2GM s 32.7 km/s
m~ g ~ Object at distance s will have an image at R g R g + Ra
Rl
2. CELLPHONE CAMERA (6 points) The distance b = f + f 2 /s and the light passing the
distance L is often called hyperfocal distance lens will converge behind the sensors plane . 0 The speed in Earths inertial frame is
in photography and it was calculated more forming a cone. The diameter d 2 of the cones v t0 = v t0 v g 2.94 km/h. To achive that, we
1
4 a
than one hundred years ago by Louis Derr cut with the sensors plane can be calculated first need to escape Earths gravity, so
~r (the figure is taken from his book Photo- from similar triangles: d 2 /D = ( b a)/ b. Ac-
R
v
u v0t 2 GM g
u !
graphy for students of physics and chemistry, counting for sharpness condition d 2 = , we 0
v1 = 2 t + .
d published in 1906). can express b = a/(1 /D ), and substituting 2 rg

1
Using the surface gravity of Earth we can u , we can again keep two sides perpendicu- dial component of T d2 (where is measured stant acceleration a directed parallel to the
GM ~ - the answer cannot depend on the
substitute r g g = g g r g so v1 11.2 km/s. lar to B in radians). Therefore, we get a differen- surface. Lets express a from the equation
path, so we choose the simplest one. Integ- tial equation: dT = T d or d ln T = d , v2 = 2a x:
iv) (1.5 points) We can calculate the speed rating = Bm sin from = 0 to gives us whence T = T0 e .
of the transfer orbit where it intersects the a = g sin /(1 + k).
w = 2Bm. As a solution to the problem, we can meas-
orbit of Mars from Keplers second law v t1 = The times are now easy to calculate as t =
v t0 /1.52 21.5 km/s. The speed of the space- ii) (4 points) Let us denote the number ure the change of the tension force for differ-
3 5 v/a , giving
craft relative to Mars is v0t1 3.25 km/s. The of electrons (per unit volume) with mag- ent angles (for example, 2 , , 2 , 2, 2

speed of the spacecraft once near Mars sur- netic moment projection +B as n + and the etc. for several turns; however, keeping the p

2H
1
2
ones with B as n . Their sum is always strings vertical offers better precision) and t = 1+k .
face is 2
g sin
v the same, n + + n = n. Also, in thermal plot ln T with respect to . The slope of the
u v0t 2 equilibrium, their ratio is given by nn+ = graph is the to be measured. Replacing k s = 25 for sphere and k c = 12 for
u !
1
vt m = 2 + r a g a 5.98 km/s
Extra solution (not as exact). Those who cylinder, we find that the sphere is faster by
t
2 B

2 exp k BT , where k B is Boltzmanns con-
cannot derive the necessary formula can still a relativesfactor
B

Since the speed of low Mars orbit is v e a = stant. Solving the equations, we can find
p n + and n . The total magnetic moment per do the experiment by doing the same meas- 1 + kc 15
r
r a g a 3.55 km/s, we need to brake for = 1 = 1 0.035.
unit volume (in the direction of B ~ ) is given by urements and noting from the plot that the 1 + ks 14
v2 2.43 km/s.
M = B ( n + n ). After substituting, relationship between and T looks expo-
v) (1.5 points) The EarthSunMars angle nential. Thus, we can make an ansatz that ii) (2.5 points) As found in previous subpart,
2 B

at the launch of the mission needs to 1 exp k BT T = T0 X : as = 0 must correspond to the accelerations parallel component to the
B B

B
be = wa t t 0.77 for the spacecraft to M = B N = B n tanh . T = T0 , we cannot reasonably write the slope a is smaller than the contribution by
2B B kB T

reach Mars. Likewise for the return trip 1 + exp kB T anywhere else without over-complicating the gravity g sin . The difference is contributed
= w g t t 1.301. If we go to the coro- formula. Now, we can re-measure the given by the friction force F f = mg sin ma . Slid-
tating frame of referense with earth, we can Additional comments. We see B and M pencil (it may be reasonably enough approx- ing starts, if the necessary friction reaches
see that the minimal time between those two always have the same sign, therefore M ~ is imated with a cylinder here; more exact ap- the maximal value Fmax = N = mg cos .
2+
angles is wa w g 1.96 yr. The minimal dura- ~
parallel with B. This makes sense, as we proaches exist) and conclude that X 2.7. Equating the two expressions gives
tion of the trip is therefore longer by t t , giving saw that = 0 ( m ~ ) orientation
~ parallel to B From there on, the calculation is the same.
had lowest energy. The graph of M vs B goes mg sin mg sin /(1 + k) = mg cos ,
2.67 yr. 6. SPHERE AND CYLINDER (7 points)
to B n for very large B or to B n for very 1+k
4. MAGNETIC DIPOLES (7 points) tan = .
small B (all spins aligned with B ~ ). At B = 0, i) (2 points) Since no energy is lost due to k
i) (3 points) There is no torque on the square M = 0 as well, since both spin orientations friction on sliding, the change in potential For the cylinder the limiting angle is =
0
if = 0 or = , so one of them is stable have the same energy. Around zero, the curve energy E p = mgH is transformed to kinetic arctan(3).
and the other unstable. If we start from is linear, as tanh x x for small x gives us energy consisting of both translational and
= 0 and turn the square to some , but 2B nB rotational motion. Taking into account the iii) (2.5 points) When the maximal friction
keep two sides of the square perpendicular
M kB T . rolling condition v = r , we have force is reached, the motion goes into rolling
to B~ , Lorentz forces on these two sides give a 5. FRICTION OF A STRING (8 points) and sliding mode, where the total force com-
1 1
torque = BIa a sin = Bm sin towards Lets first calculate the difference of tension E p = E k = mv2 + I 2 ponent along the surface is given by the dif-
2 2 ference of gravity and friction:
decreasing . By symmetry, we get the same force T between two ends of a sliding string 1 1 1
result if we keep the other keep two sides arced over a cylinder by an angle . Further- = mv2 + kmv2 = (1 + k) mv2 , F = mg sin Fmax = mg sin mg cos .
2 2 2
of the square perpendicular to B ~ . It is pos- more, lets look at a short piece of the arc
sible to conclude that the torque depends that subtends an angle d . On one hand, where general expression I = kmr 2 for mo- We note that the acceleration in this mode
only on (at least near = 0), not on the dT = dR is the friction force acting on the ment of inertia is used. Therefore, v2 = does not depend on the moment of inertia any
exact orientation of the square. Since torque piece, where dR is the reaction force. On the 2 gH /(1 + k). more.
acts to restore = 0, we find that s = 0 and other hand, dR T d , because both ends On the other hand, the bodies travel dis- Calculating the limiting angle of slipping
u = . To find the work to get from s to of the piece are pulled by a force with a ra- tance x = H / sin along the slope with a con- mode also for the sphere sph = arctan( 72 ) >

2
0 shows that for all angles larger than the frequency = p 1 and we can immedi- summarize the equations as follows: (for I = 0). If a trajectory reaches any of
LC
m = sph both bodies are in the slipping ately write q( t) = q 0 cos t, while I ( t) = q( t) = q the points in that segment, it will stay there
mode and thus have equal accelerations and q 0 sin t. L q + = Vd if q < 0 forever. The extent of that region is 2CVd .
C
arrival times. q iv) (2 points) Lets use the phase diagram
Note that L q + = Vd if q > 0
7. BURNING WITH A LENS (7 points) The C to figure this out. Suppose the capacitor
solar energy flux which is focused by the lens 1 initially has the charge q 0 CVd . Then
q2 + 2 I 2 = q20 (sin2 t + cos2 t) = q20 ,
to the image of the Sun can be calculated Let us introduce the new variables q 1,2 the charge will first swing to the other way
as P = 4 d 2 I ; the image of the Sun radiates such that q 1 = q CVd and q 2 = q + CVd . of CVd and will become q T /2 = CVd ( q 0
and therefore the phase diagram of the sys- Then we can rewrite the two equations above
according to the Stefan-Boltzmann law with CVd ) = 2CVd q 0 . Then it will perform the
tem is an ellipse centred at the origin, with in a more familiar form:
the total power P = 4 ( f )2 T 4 . From the other half-oscillation around CVd and the
semi-axes q 0 and q 0 . Alternatively, this re-
heat balance we obtain 4 d 2 I = 4 ( f )2 T 4 , q1 charge at the end of that is q T = CVd +
lation comes directly from the conservation L q 1 + =0 if q < 0
hence C (CVd (2CVd q 0 )) = q 0 4CVd , and there-
v of energy: q2 fore q = 4CVd .
u s
u d I L q 2 + =0 if q > 0
T= 4500 K. q20 C
t
f LI 2 q2 Note that we have the right to talk about
+ = E0 = . Thus the introduction of the diode only serves half- and full periods because the oscilla-
2 2C 2C
Due to the second law of thermodynamics, to shift the equilibrium points for the oth- tions still happen at the immutable frequency
it is impossible to direct heat energy from a By looking at q and I a quarter-period erwise simple harmonic orbits. For q > 0, = p 1 . Therefore the time between the
LC
lower temperature body to a higher temper- later from t = 0, say, its not hard to see that the equilibrium point is q 2 = 0 or q = CVd , two maxima is just a full period of oscillation,
ature body. Hence, the image temperature the system must evolve in a clockwise sense while for q < 0 it is q = CVd . So the orbit will T = 2 .
cannot exceed the temperature of the Sun. on the phase diagram. Note that in this in- consist of half-ellipses in the upper and the Once q( t) has a zero derivative inside the
Now we can use the known temperature of stance, only q = 0 is an equilibrium point: lower parts of the I q diagram, centred at region bounded by CVd , it will remain at
Sun T0 = 6000 K, but it is better to use the for all non-zero q there will be never-ending q = CVd for the upper half and at q = CVd that particular value forever. For a large ini-
Stefan-Boltzmann law for solar radiation flux oscillations in the circuit. for the lower half. As the evolution is continu-
tial
q 0 , we expect there to be approximately
density: near the Suns surface, I 0 = T04 , I ous, these half-ellipses will join up at I = 0. q0 | q0 |
with the total flux of P t = 4R 2s I 0 . Near q = 4CV total oscillations.
I d
the Earth, the total flux P t = 4L2 I ; here, More exactly, the distance from the dead
R s is the Suns radius, and L the orbital zone is initially | q 0 | CVd and decreases dur-
radius of the Earth. From here we obtain ing each half-oscillation by 2CVd .jThe total
I = I 0 R 2s /L2 = T04 R 2s /L2 ; using the previous | q 0 |CVd
k
number of half-oscillations is N = 2CVd
result, p
s and the total time t = N T2 = N
= N LC .
d Rs
T = T0 . q 9. GLASS CYLINDER (7 points) The axis
f L
q of the half-cylinder is where the stripe and
Let us note that L = 2R s , hence CVd CVd
its image coincide (form a straight line). The
front edge of the half-cylinder is at the 28th
s
d
T = T0 T0 , line, counting from the axis, hence the radius
2f
of the cylinder R 28. Let us consider the
which means that d 2 f . refracting ray s which is very close to a total
internal reflection. One can see the images
8. ZENER DIODE (7 points)
of 20 lines (ca 20.2, to be more precise), when
i) (1 point) Kirchoffs 2nd law gives L I + ii) (2 points) Now the sign of the voltage on counting from the central line upwards; the
1
q/C = 0 or q + LC q = 0. This is the equa- the diode depends on the direction of the cur- iii) (2 points) We can see on the diagram upper edge of the half-cylinder coincides with
q
tion of a simple harmonic oscillator with rent, giving either of L q + C Vd = 0. We can that there is a dead zone between CVd

3
the 42nd line at the background. the perpendicular of the paper surface. The Measure the temperature at the end, after The resistance used was R = 0.47 5%.

So, the ray s arrives at the camera at incidence angle of the ray s is + 45.9 , waiting a bit or stirring the calorimeter. We In the described circuit the batteries were
the angle = arcsin(28/42) 41.8 with re- hence n = 1/ sin( + ) 1.39. want to get maximum temperature difference
2
depleted in 10 to 15 minutes and the tem-
spect to the plane of the paper. The pro- 10. RESISTIVE HEATING (8 points) After for precise measurement. Since P = R I , perature of the calorimeter rose by 7 to 10
jection of the refraction point to the paper noting the temperature of the calorimeter, degrees.
Q X
surface lies at the distance a = 28 sin = connect the batteries, resistor (in the calor- t( I n /2 + I n+1 /2)2 ,
2 R n
28 /42 18.7 lines from the axis. There- imeter) and ammeter in series. Choose a
fore, before refraction, ray s forms an angle convenient time interval t and note the am- ( c a m a + c w m w ) T
R= P 2
.

= arcsin[(20.2 18.7)/(28 cos )] 4.1 with meter reading until batteries are depleted. n t( I n /2 + I n+1 /2)

4
ESTONIAN FINNISH PHYSICS OLYMPIAD
PROBLEMS & SOLUTIONS (2014)
Estonian-Finnish Olympiad 2014 iv) (2 points) Assuming that Vmax V0 , Sun? that ball fly compared to the initial drop
what is the average power dissipation on the iii) (1.5 points) What is the speed of the height h? Take f = 0.5 and n = 10.
1. DC-DC CONVERTER (8 points) In order diode? spaceship in the Earths frame of reference It maybe useful that that sequence a 0 = 1,
to obtain high voltage supply using a battery, v) (2 points) Now, let the key K 2 be closed, when the distance from the Earth is much a k+1 = a k + has a general term a n = n +
n
the following circuit is used. and let us assume simplifyingly
p that V0 = 0; larger than the Earths radius, but still much 11 , where and are constants.
also, RC L and K > LC . Suppose that shorter than the distance to the Sun? 5. PLANCKS CONSTANT (8 points) In a
the circuit has been operated for a very long iv) (2.5 points) Answer the previous question simplistic model, light emitting diodes can be
time. Find the average voltage on the res- without making the approximation 0. considered to only pass current when lit, and
istor. then they have a constant voltage drop Vt =
v) (1.5 points) What is the speed of the space- E
vi) (1 point) Find the amplitude of voltage ship in the Earths frame of reference when e across them. E = h f is the energy of the
variations on the resistor. the distance from the Earth is much smaller light quanta emitted and e = 1.60 1019 C
2. WASTE PROJECT (8 points) In 2114, than the Earths radius? is the elementary charge. Speed of light in
Europarliament decided that all radioact- 3. MAGNETS (6 points) To explore the force vacuum c = 3.00 108 m/s.
ive wastes need to be sent to the Sun, so between two small magnets, the following You have a assorted light emitting diodes
An electromagnetic switch K 1 connects a bat- as to avoid contamination of Earth and or- experiment is performed. One of the mag- numbered 16, each with a R = 680 series
tery of electromotive force E to an inductor bital space. In what follows, you can use the nets is hanged from a thread with length resistor. From the datasheets it is known
of inductance L: it is closed if there is no cur- following numerical data: duration of one l = 1 m. Other magnet is moved slowly closer for the peak wavelengths of the diodes to
rent in the inductor (a spring keeps it closed), year T = 365.25 days, orbital speed on Earth while keeping the axes of the magnets al- be 940 nm, 620 nm, 590 nm, 525 nm, 470 nm,
but if the inductor current reaches a critical v0 = 29.8 km/s, angular diameter of Sun as ways on the same horizontal line. At the and 450 nm.
value I 0 , magnetic field created by the in- seen from the Earth = 0.5, radius of the moment when the distance between the mag-
i) (2 points) Find out which wavelength cor-
ductor pulls it open. Due to inertia, once the Earth R = 6400 km, free fall acceleration at nets is d = 4 cm and the hanged magnet has
1 responds to which diode.
key is open, it takes a certain time K to close the Earths surface g = 9.81 m/s2 . moved x = 1 cm from initial position, bal-
1
ii) (4 points) Measure the Plancks con-
again even if the current falls to zero. According to the project, the waste is sent ance is lost and the magnets pull together. By
stant h that corresponds to our simplistic
For the diode D you may assume that to the Sun using ballistic spaceships: the en- making the assumption that the pulling force
model. This does not have to correspond to
its current is zero for any reverse voltage gine operates only during a short period of between the magnets Fm depends on the dis-
real Plancks constant.
(VD < 0), and also for any forward voltage time during which the displacement of the tance d according to the relation Fm d n ,
spaceship remains much shorter than the find the value of the exponent n. iii) (2 points) Estimate the uncertainty.
smaller than the opening voltage V0 (i.e. for
0 < VD < V0 ). For any non-zero forward cur- radius of Earth. In the Earths frame of refer- 4. SUPERBALLS (5 points) n + 1 elastic Equipment: voltage source (batteries)
rent, the diode voltage VD remains equal to ence, the spaceship obtains a velocity oppos- balls are dropped so that they are exactly with an unknown voltage, ammeter, assor-
V0 . ite to the orbital velocity of Earth in the Suns above each other, with a very small gap ted light emitting diodes with series resistor.
frame of reference. Further, the ship moves between each. Bottom ball has a mass of m 0 , Take care not to short the battery with the
You may express your answers in terms along a ballistic trajectory until it hits the
the one above has a mass of f m 0 , next f 2 m 0 ammeter. You may ignore the internal resist-
of L, E , I 0 , V0 , R , and the capacitance C (see Sun. The trajectory is such as to minimize
and so on, until the topmost ball with mass ances of the batteries and the ammeter.
figure). the consumption of fuel. f n m 0 , where f < 1. At the moment when the
i) (1 point) At first, let the key K 2 be open. If i) (1 point) Sketch the trajectory of the space- bottom ball touches the ground, all the balls
the initial inductor current is zero, how long ship. are moving with the speed v.
time L will it take to open the key K 1 ?
As a first approximation, calculations can i) (1 point) After the collision between the
ii) (1 point) Assuming (here and in what be done when neglecting the angular size of two bottommost balls, what is the speed v1
follows) that L/R K L , plot the in- the Sun (i.e. by putting 0); you can use of the second ball from the bottom?
ductor current as a function of time t (for this approach for the next two questions.
0 t < 3L ). ii) (3 points) What is the speed of the top-
ii) (1.5 points) How long will it take for the most ball vn after all collisions?
iii) (1 point) What is the maximal voltage spaceship to travel from the Earth to the
Vmax on the resistor R ? iii) (1 point) How many times higher would
placed at a distance l = N (where N is a Cv
3R
6. RUNNING ON ICE (4 points) A boy is is sparse, so that the mean free path of the
running on a large field of ice with velocity large integer) from the point source, and the molecules is much larger than a. Assume
v = 5 m/s toward the north. The coefficient of interference pattern is observed on a screen that v2 c s where c s is the speed sound in
friction between his feet and the ice is = 0.1. which is placed to a distance L l from the 2R the atmosphere surrounding the cube.
Assume as a simplification that the reaction point source (see figure). 10. YOUNGS MODULUS OF RUBBER
force between the boy and the ice stays con-
stant (in reality it varies with every push,
R (12 points) The linear Hookes law for a
rope made from an elastic material is sup-
but the assumption is justified by the fact
T(K) posed to held for small relative deformations
0 100 200 300
that the value averaged over one step stays = x/L (which is also called strain), where
constant).
i) (2 points) What is the minimum time ne-
i) (1 point) What is the total heat energy of
L is the undeformed length of the rope, and x
is the deformation. Once becomes too large,
cessary for him to change his moving direc- such a cube at the initial temperature T0 ? the force-deformation relationship F = kx is
tion to point towards the east so that the final ii) (3 points) Now, the cube has 5 no longer linear; what is too large depends
speed is also v = 5 m/s? faces painted in white (reflects all relevant on the material. For very elastic materials
ii) (2 points) What is the shape of the optimal wavelengths) and one face painted in black which can reach relative deformations con-
trajectory called? (absorbs all these waves). The cube is sur- siderably large than one, it may happen that
rounded by vacuum at a very low temperat- the linear Hookes law with a constant stiff-
7. SPIN SYSTEM (8 points) Let us consider ure (near absolute zero); there is no gravity ness k fails, but if we take into account the
a system of N independent magnetic dipoles In what follows we use the x, y, and z co-
field. Initially, the cube is at rest; as it cools change of the cross-sectional area S of the
(spins) in a magnetic field B and temperature ordinates as defined in the figure. The screen down due to heat radiation, it starts slowly rope with k = ES /L, where E is the Youngs
T . Our goal is to determine some properties is parallel to the mirror and lies in the y z- moving. Estimate its terminal speed v . modulus of the elastic material, such a non-
1
of this system by using statistical physics. It plane. linear Hookes law remains valid. In that
is known that the energy of a single spin is i) (2 points) At which values of the y- iii) (2 points) At very low temperatures, case we can say that there is still a linear
E = m, where m = 21 and = B. the heat capacitance of aluminium is pro-
coordinate (for z = 0) are the interference stress-strain relationship = E , where the
portional to T 3 , where T is its temperat-
i) (2 points) What is the probability p for maxima observed on the screen? You may ure. Which functional dependance f ( t) de- stress = F /S .
a spin to be in exited state, i.e. have positive assume that y L. scribes the temperature as a function of time i) (7 points) Measure the relationship
energy? ii) (1 point) Sketch the shape of few smallest- [T = A f (Bt), where A and B are constants] between the stress and strain in a rubber
ii) (2 points) What is the average value of sized interference maxima on the screen (in for such very low temperatures under the string and plot it.
the total energy E s of the spin system as a y z-plane). assumptions of the previous question? ii) (5 points) From your plot determine the
function of B and T ? iii) (2 points) Now the flat screen is replaced iv) (3 points) Now, the cube has 5 faces Youngs modulus E with its uncertainty, and
iii) (2 points) Using high temperature ap- with a spherical screen of radius L, centred covered with a thermal insulation layer (you the maximum strain m until which it ap-
proximation T Bm around the point source. How many interfer- may neglect heat transfer through these plies.
k , simplify the expres-
sion of E . ence maxima can be observed? faces). One face is left uncovered. The cube
s Note: the diameter of the thread is to be
is surrounded by hydrogen atmosphere at
iv) (2 points) Using high temperature ap- 9. THERMAL ACCELERATION (9 points) measured using the diffraction of laser light.
Consider a cube of side length a = 1 cm, made a very low temperature (molar mass of hy-
proximation T Bmk , find the heat capacity Equipment: rubber thread, stand, meas-
C of the spin system. of aluminium (density = 2.7 g/cm3 , molar drogen molecules M H = 2 g/mol). The cube
starts cooling down due to heat transfer to uring tape, 15 hex nuts with a known mass,
mass M A = 23 g/mol). The heat capacitance
8. MIRROR INTERFERENCE (5 points) of one mole of aluminium is given as a func- the surrounding gas; you may neglect the a plastic bag for hanging a set of nuts to the
A point source S emits coherent light of tion of temperature in the graph below. The heat radiation. Initially, the cube is at rest; thread, a green laser ( = 532 nm), a screen.
wavelength isotropically in all directions; speed of light c = 3 108 m/s, universal gas as it cools down, it starts slowly moving. Es- WARNING: AVOID LOOKING INTO
thus, the wavefronts are concentric spheres. constant R = 8.31 J/(kg K). The initial tem- timate the order of magnitude of its terminal A LASER BEAM, THIS MAY DAMAGE
The waves reflect from a dielectric surface perature of the cube is T = 300 K. speed v2 . Assume that the surrounding gas YOUR EYES!
0
1. DC-DC converter interval when the diode is open, we can also neg- hence its trajectory is an ellips with longer semi- proportionality coecient. Then

1) lect the presence of the diode since V0  Vmax . axis equal to RE /2.
According to the Kepler's kn
(1 pt) From the Kircho 's voltage law for
Hence, the current i in the LC -loop changes in 3/2 Fm = Fm
0
(d)d = x,
the loop consisting ofL and E , E = L dI , hence III law, the period on such an orbit is 2 dn+1
dt
time sinusoidally, starting from i = I0 and end- times shorter than the Earth's orbital period T. because d = x. Therefore
I = Et/L. From I0 = EL /L we obtain
ing when i = 0 (then the diode will close discon- The travel time t is half of the period, so that

kn mg

L = LI0 /E. necting the LC -loop). During that process, the 5/2 F = x.
t=2 T 64.6 days. dn+1 l
2) (1 pts) Once the current I0 is reached, the key 1 2
magnetic energy of the inductor 2 LI0 is conver- 3) In the Sun's frame of reference, the speed At the limit case of the loss of stability (which
is opened; the current trough L cannot change ted into the electric eld energy of the capacitor, needs to be zero; hence, in the Earth's frame is described by this problem) F = 0. Thus we
instantaneously and therefore is forced to ow which is later released as heat on the resistor. of reference, it is opposite to the orbital ve- have two equations with two unknowns (n and
through the resistor R. Since the characteristic When the stationary regime is achieved, the en- locity of the Earth and by modulus equal to k):
time of this current loop (consisting of L and R ergy lost by the capacitor during one period (of v0 = 29.8 km/s. k xmg kn mg
is very short (L/R  K ), the current decays duration L ) as the heat dissipation on the res- 4) The speed vS in the Sun's frame of reference = 0, = 0;
very fast and becomes essentially equal to zero 2 dn l dn+1 l
istor Q = Vav L /R must be equal to the energy is found from the expression for the total energy, this can be rewritten as
while the key is still open. Now there is no cur- received from the inductor; so, GM m GM m GM m mvS2 k xmg kn mg
rent through the inductor, so that the key will 2
r r = = + , = , = .
Vav L 1 LR EI0 R 2a RE + rS Re 2 dn l dn+1 l
close again and the process will start repeating = LI02 Vav = I0 = . hence If we divide the corresponding sides of the two
from the beginning. As a result we'll have a R 2 2L 2 r
GM 2rS equations we obtain d/n = x, hence
periodic graph as shown in gure. 6) (1 pt) The charge which ows away from vS = .
I the capacitor when the diode is closed can be
R E R E + rS n = d/x = 4.
I0 found as qC = L Vav /R (owing to RC  LC ,
This expressing can be rewritten by using equal-
2 GM 4. Superballs
ity v0 = R as
the relative change of the capacitor's voltage rE r 1) During the collision with the oor, the
is small). Hence, the voltage drop is found as 2rS
 
vS = v0 = v0 2 sin v0 . bottom-most ball will retain its speed and
t V = qC /C = L Vav /(RC). The amplitude is R E + rS 2 change the direction of the velocity; its upwards
half of the dierence between the minimal and
Numerically this yields vS 2.8 km/s; the speed speed v0 = v . Let the k-th ball move up with a
3) (1 pt) The voltage through the resistor is maximal values, so that the amplitude
in the Earth's frame of reference vE = v0 vS velocity vk ; we'll consider the collision between
maximal when the current is maximal, which r
happens immediately after the switch is opened;
V L Vav I0 L I0 27.0 km/s. this and (k + 1)-st ball, which moves down with
U0 = = = . 5) Part of the initial kinetic energy in the
the maximal current is I0 so that Vmax = RI0 . 2 2RC 2C 2RE the velocity v . In the frame of reference of the
Earth's frame of reference goes to the change vk f v
4) (2 pts) Due to Vmax  V0 , we can neglect centre of mass, u = 1+f
; hence, after the
2. Waste project of the potential energy due to the gravitational
the eect of the diode; so we have the Kircho 's collision the upwards velocity equals to vk+1 =
GME m
dI dq
voltage law L dt = RI = R dt (here we ex-
1) The trajectory is a very elliptical ellipse, peri- pull of the Earth, = R
= gmR hence f v
v + 2 vk1+f 2
= 1+f vk + 1f v . With v0 = v , we
2 1+f
helion of which is within the Sun. In order to vE 2
u 3f

can conclude that v1 = 1+f v = 1+f 1 v . 2)
4
pressed the current via the charge q which ows gR + 2 = 2 . Here, ME is the Earth's mass
minimize the fuel consumption, the speed near
through the resistor). Integration over a single and u is the launching speed. So, One can see that if we apply the recurrent for-
the Earth's orbit needs to be as large as pos-
cycle (when the inductor current drops from I0
p
u= 2
vE + 2gR 29.2 km/s. mula repetitively, the result at the n-th step will
sible (we need to decelerate the ship to bring n
2
 
down to 0) yields LI0 = Rq , hence the charge be vn = 2 1+f 1 v.
it to the elliptical orbit), hence the full orbital 3. Magnets
own through the resistor (and through the di- GM m 3) Now we need to relate the speeds to the jump-
energy of the ship E = 2a needs to be as
ode) q = I0 L/R. During that cycle, the diode There three forces acting on the hanging mag-
2 2
ing heights via v = 2gh0 and vn = 2ghn ; hence,
small as possible. Here, M is the mass of the
had a constant voltage V0 , so the electric eld net: the downwards directed gravity force m~g , s  n
A = V0 q Sun, m  the mass of the space ship, and a  2
performed work which was released T~ ,
p
the tension force which is directed along the hn /h0 = vn /v0 = 2 1.
the longer semiaxis. So, a needs to be as large as
as heat in the diode. So, the average power dis-
thread, and the horizontal magnetic force ~m .
F 1+f
possible, which means that the perihelion needs
sipation
Since the thread's angle is very small, the mod- For f = 0.5 and n = 10 we obtain that the nal
to lie at the surface of the Sun, in which case
A V0 I0 L V0 E ulus of T~ is almost equal to mg , so that its height will be ca 1200 larger than the initial one.
P = = = . 2a = RE + rS , where RE is the orbital radius of
L RL R the Earth and rS  the radius of the Sun. The horizontal projection is expressed as (x/l)mg ,
5. Planck's constant
5) (2 pts) Now, since the characteristic time of resulting trajectory is depicted below. where l is the length of the thread and x = 1 cm
the RC -loop is very large, the capacitor main- (the displacement from the initial position). The 1) When we connect each of the diodes to the
Earths o

tains its charge (and voltage) during that period net horizontal force F = Fm (x/l)mg . At the battery, we can observe the light of the emit-
of time when the diode is closed. Once the key equilibrium point F = 0. The equilibrium point ted light; the mapping is as follows: 940 nm 
Sun
K1 opens, the diode will open, and the capacitor is stable if a small (virtual) displacement x invisible (infrared), 620 nm  red, 590 nm  or-
rbit

is connected to the inductor so that a LC -loop is gives rise to a returning force ange, 525 nm  green; 470 nm  blue; 450 nm
formed. That
loop admits oscillations of period x  violet.
TLC = 2 LC and as the current to the resistor 2) If we neglect the radius of the Sun, the space
F = Fm mg 2) We can measure the current I through the
l
can be neglected (since RC  TLC ), we can neg- ship needs to fall directly to the Sun, which which needs to push towards the equilibrium diode (which is also the current through the res-
lect the presence of the resistor. During the time means that its initial orbital speed must be zero, point. Let F m
= kdn , where k is an unknown istor R), so that the voltage on the diode would
be Vd = E IR, but we don't know the battery 3) For small values of the argument of the hy- over all the photons, the perpendicular to the those molecules which collide with the cube at
voltage. However, we do know that the diode's perbolic tangent, the last expression can be ap- surface normal components cancel out (photons low temperatures to the overall momentum re-
2
voltage equals approximately to the photon's proximated as E N  /4kT . go to all the directions). The average value of mains still small.
energy Ep expressed in electron volts, Ep = 4) According to the denition of the heat capa- the parallel component can be estimated just as
10. Young's modulus of rubber
hc/(e). Since we expect that IR = E Vd , dE 2
city, C = dT = N  /4kT .
2
pk Ec .
if we plot IR versus 1/, we should obtain a If we want to obtain an exact result, we need 1) The setup is as follows. The rubber thread

straight line
8. Mirror interference
to integrate over all the directions while keeping is xed to the stand, and the plastic bag is xed

IR = E
1 hc
.
1) For a position y, the arriving rays form an in mind that the light intensity is proportional to to the free end of the thread. The hex nuts are

e angle = y/L (we use the small-angle approx- cos . So, the momentumR 2 averaged over all the
added, one-by-one, into the bag, starting with
We can measure the slope of the straight line imation; the angle is in radians). Then, the op- directions pk = Ec 2 1
cos d, where the solid zero and ending with 15. The laser light is dir-
A = hc/e, which allows us to calculate h = tical path dierence between the reected and angle dierential d = 2 sin d. Therefore, pk = ected to the thread and the diraction pattern
eA/c. 2 is observed on the screen (which is a vertically
direct rays is = 2l cos 2N N . .
R /2 R /2 2
E
sin cos2 d = E E
cos d cos = 3c
3) The major source of the uncertainty is not Since there is an additional phase shift for the
c 0 c 0 xed sheet of graphic paper on a support). The
Since the momentum-energy ratio is the
the instrument uncertainties, but the departure reected rays at the reection from optically diraction pattern from a wire is the same as
same for all the photons, equal to 1/c, the overall
of the real diode data from the simplistic model. denser dielectric material, the total phase shift from a single slit (the superposition of the Huy-
momentum equals to Q/c. Thus, a3 v Q/c,
Therefore we can try to t the data points with 2
is = 2/ = 4N (2N 1). At the gens sources from those two cases gives a full set
hence
dierent straight lines making the slope A as maxima, this equals to 2(2N n), where n Q of sources on a at wave front, hence the electric
steep as possible (while still keeping a reason- v 3 0.67 mm/s. elds from those two cases must provide equal
is an integer. Therefore, the condition for the a c
able t with the data points, and also as at maxima is written as 1 and opposite wave amplitudes and equal intens-
If we apply the exact factor 3 (obtained above
as possible; the uncertainty of A is found as
r r
via integration), we end up with v 0.22 mm/s.
ities). So, if we measure on the screen the dis-
n + 0.5 n + 0.5
A = 21 (Amax Amin ), and h = hA/A. = yn = L , 3) The heat balance at very low temperatures
tance a between such maxima which are separ-
N N 3 4 ated by n (e.g. n = 10) periods of the diraction
6. Running on ice can be written as AT dT = ST dt, where
where n = 0, 1, . . .  N . pattern then using small-angle-approximation,
1)
A is a constant, is the Stefan-Boltzmann con-
During the process, the velocity vector needs 2) Since the rays of a given order number n n/d = a/L, where L is the distance from the
stant, and S  the radiating area. This simpli-
to change its direction by 90 degrees. Let us form a xed angle with the x-axis, the max- thread to the screen and d  the thread dia-
es to
consider the this graphically using the vx vy - ima form on the screen concentric circles; the meter. So,
dT
plane: we need to move from the point A with pitch between the neighbouring circles becomes = Bt T = A eBt . d = nL/a.
coordinates (0, v) to a point B with coordinates smaller as the order number n grows (using the
T
The strain is calculated by making markings on
4) In the case of a hydrogen atmosphere, the
(v, 0) while having a constant speed. Indeed, length unit dened by the
smallestradius, the the thread and measuring the distance b between
momentum is given to the cube due to the fact
the velocity of a point in the vx vy -plane is radii
form a sequence 1 = 1, 3 1.73, these in a stretched state,
that the molecules colliding with the coated
the acceleration of the body, which has here a 5 2.23, etc). b b0
faces bounce back with the same speed as the = ,
constant modulus g . Obviously,the fastest 3) Since the reected rays can reach the screen b0
they came, but uncoated face gives away heat where b0 is the length in a non-strained state.
path
is a straight line of length v 2, so that only within a hemisphere, the phase shift varies
energy, and the molecules leave at higher tem- The stress is calculated as
t = v 2/g 7.2 s. between max = 4N + and min = . The
4N mg
perature. If we assume that the departing
2) Since the direction of the acceleration is con- number of maxima = ,
molecules have the same temperature as the d2
stant, the trajectory is the same as for a body m = (max min )/2 = 2N. where N is the number of hex nuts in the bag
cube (which serves us only as an estimate 
in the Earth's eld of gravity  a parabola. and m  the mass of a single nut. The data
when particles of dierent masses collide, only
9. Thermal acceleration
are plotted in a graph; linear relationship cor-
7. Spin system a part of the energy is transferred), then the
1) For the heat energy of one mole of material
momentum-to-heat ratio is estimated as 1/vT , responds to a straight line. The uncertainties
1) According to Boltzmann's distribution, p =
dq = Cv dT .R There is no heat energy by T = 0, p are calculated using the rule of relative uncer-
A em , where the constant A can be found T where vT = RT /MH is the thermal speed of
tainties, either using Pythagorean or simple ad-
hence q = Cv dT . Using the graph we nd
from the condition that the probability of hav- 0 the molecules after the collision with the cube
this as the area under the curve, q R560 J/K. dition, e.g.
ing either up or down orientation is one: A 3
for the motion along the surface normal. So we

2 1

The number of moles = a /MA 0.117 mol, 3 = b + ,
e/2 + A e/2 = 1, hence estimate a v Q/vT , hence
b b0 b0
1 1 hence the total heat energyQ = q 546 J. r where b 0.5 mm is the length measurement
A = /2kT = . 2) Each photon of frequency radiated by the Q MH
e + e/2kT 2 cosh(/2kT ) v 180 m/s. uncertainty. Similarly,
Thus, cube carries away heat energy equal to E = h , a3 RT d
and carries momentum p = h/ = h/c = E/c. It should be noted that in fact, one should have
= 2 .
e/2kT d
p = /2kT . If the photon departs at the angle with re- been careful with such an estimate, because the These uncertainties are marked in the graph as
e + e/2kT error bars.
2) The average energy is the weighted average spect to the surface normal then the compon- thermal speed is at the denominator. This will
of up- and down-state energies for a single spin,
E
ent parallel to the surface normal pk = c cos . increase the relative contribution of the heat ra- 2) Using the plot, we need to determine such a

multiplied by the number of spins: The total momentum given by the photons to diated at low temperatures. However, the re- value of = that for 1 > , it is impossible
/2kT /2kT the cube equals by modulus to the total mo- maining heat at low temperatures is propor- to draw a straight line intersecting the error bars
N e e N
E= = tanh(/2kT ). mentum carried by the photons; when averaged
4
tional to T , and therefore the contribution of of all the data points with 1 < .
2 e/2kT + e/2kT 2

You might also like